Sunteți pe pagina 1din 190

Aritmetica II

PROFMAT
Fabio E. Brochero Martinez
Carlos Gustavo T. de A. Moreira
Nicolau C. Saldanha
16 de Marco de 2012

Conte
udo
0 Fundamentos
0.1 Princpio da Inducao Finita . . . . . . . . . . . .
0.2 Princpio da Casa dos Pombos . . . . . . . . . .
0.3 Divisibilidade . . . . . . . . . . . . . . . . . . . .
0.4 mdc, mmc e Algoritmo de Euclides . . . . . . . .
0.5 O Teorema Fundamental da Aritmetica . . . . .
0.6 Congruencias . . . . . . . . . . . . . . . . . . . .
0.7 Bases . . . . . . . . . . . . . . . . . . . . . . . .
0.8 O Anel de Inteiros M
odulo n . . . . . . . . . . .
0.9 A Funcao de Euler e o Teorema de Euler-Fermat
0.10 Equacoes Lineares M
odulo m . . . . . . . . . . .
1 Pot
encias e Congru
encias
1.1 Polin
omios . . . . . . . . . . . . . . . . . . . . .
1.2 Ordem e Razes Primitivas . . . . . . . . . . . . .
1.3 Resduos Quadraticos e Smbolo de Legendre . .
1.3.1 Lema de Gau . . . . . . . . . . . . . . .
1.4 Lei de Reciprocidade Quadratica . . . . . . . . .
1.4.1 Uma demonstracao combinat
oria . . . . .
1.4.2 Uma demonstracao trigonometrica . . . .
1.4.3 Uma demonstracao usando corpos finitos

.
.
.
.
.
.
.
.
.
.

.
.
.
.
.
.
.
.

.
.
.
.
.
.
.
.
.
.

.
.
.
.
.
.
.
.

.
.
.
.
.
.
.
.
.
.

.
.
.
.
.
.
.
.

.
.
.
.
.
.
.
.
.
.

.
.
.
.
.
.
.
.

.
.
.
.
.
.
.
.
.
.

2
2
9
13
16
24
31
34
37
43
53

.
.
.
.
.
.
.
.

61
61
73
81
84
88
90
92
94

2 Fun
co
es Multiplicativas e as f
ormulas de invers
ao de M
obius
99
2.1 As funcoes d, e . . . . . . . . . . . . . . . . . . . . . . 102
2.2 Funcao de M
obius e Formula de Inversao . . . . . . . . . 110
3 Fra
co
es Contnuas
3.1 Reduzidas e Boas Aproximacoes .
3.2 Boas Aproximacoes s
ao Reduzidas
3.3 Fracoes Contnuas Peri
odicas . . .
3.4 O espectro de Lagrange . . . . . .

.
.
.
.

.
.
.
.

.
.
.
.

.
.
.
.

.
.
.
.

.
.
.
.

.
.
.
.

.
.
.
.

.
.
.
.

.
.
.
.

.
.
.
.

.
.
.
.

118
. 127
. 129
. 134
. 135


CONTEUDO
4 Equa
co
es diofantinas n
ao lineares
4.1 Teorema de Pit
agoras e triplas Pitagoricas . . . . . . . .
4.2 Tri
angulos ret
angulos de Pit
agoras e Platao . . . . . . .
4.3 Triplas Pitagoricas Primitivas . . . . . . . . . . . . . . .
4.4 Tri
angulos pitagoricos e o metodo geometrico . . . . . .
4.5 Tri
angulos com lados inteiros e angulos em progress
ao
aritmetica . . . . . . . . . . . . . . . . . . . . . . . . . .
4.6 Outra relacao de
angulos . . . . . . . . . . . . . . . . . .
4.7 Contando tri
angulos pitagoricos com um cateto dado . .
4.8 N
umeros que s
ao somas de dois quadrados . . . . . . . .
4.9 Tri
angulos pitagoricos com catetos consecutivos . . . . .
4.9.1 Equacao de Pell . . . . . . . . . . . . . . . . . .
4.10 Solucao fundamental da equacao de Pell . . . . . . . . .
4.11 Outras equacoes do tipo x2 Ay 2 = c . . . . . . . . . .
4.12 Contando tri
angulos pitagoricos com hipotenusa fixada .
4.12.1 Inteiros de Gau . . . . . . . . . . . . . . . . . .
4.13 Descenso Infinito de Fermat . . . . . . . . . . . . . . . .
Bibliografia

.
.
.
.

139
139
141
143
146

.
.
.
.
.
.
.
.
.
.
.

149
151
153
155
159
159
164
167
169
170
178
185


CONTEUDO

Introdu
c
ao
Aritm
etica II - PROFMAT
Nosso texto para a disciplina Aritmetica II visa tratar de temas
elementares e interessantes da Teoria dos N
umeros. Assim, pretendemos
que este livro seja acessvel e nao dependa de pre-requisitos sofisticados
de algebra ou an
alise, mas que apresente ao leitor resultados relevantes
de topicos como equacoes diofantinas de grau 2, funcoes aritmeticas e
aproximacoes diofantinas.
Comecamos com uma revisao de fundamentos de aritmetica (que
constam do programa do curso de Aritmetica I do PROFMAT). A seguir,
no Captulo 1, tratamos de ordens, razes primitivas, resduos quadraticos e da lei de Gauss da reciprocidade quadratica, um dos resultados
mais importantes e surpreendentes da teoria elementar dos n
umeros.
No captulo seguinte tratamos de um tipo particularmente importante
de funcoes aritmeticas: as funcoes multiplicativas, e de um importante
topico relacionado: as formulas de inversao de Mobius. No Captulo 3
apresentamos a teoria elementar das Fracoes Contnuas, que fornecem
as melhores aproximacoes racionais de um n
umero real dado, fazendo,
dessa forma, uma introducao `
a teoria das Aproximacoes Diofantinas.
Finalmente, no Captulo 4, tratamos de Equacoes Diofantinas de grau
2, para as quais ha diversos resultados cl
assicos, bonitos e importantes, comecando com o estudo das triplas pitagoricas, discutindo ideias
geometricas e tratando da equacao de Pell, relacionada com fracoes contnuas e aproximacoes diofantinas.
Esperamos estimular os leitores, especialmente os professores de matematica do ensino medio, a se interessarem pela Teoria dos N
umeros, e
a prosseguirem seus estudos sobre o tema. Observem que inclumos exerccios rotineiros mas tambem varios problemas bastante difceis, muitos
deles de olimpadas de matem
atica. Estimulamos o leitor a tentar fazer
todos os problemas mas com a advertencia que ele nao deve se sentir
frustrado se mesmo com esforco nao conseguir resolver alguns dos problemas: muitos deles s
ao difceis tambem para os autores.
Agradecemos a todos os que nos ajudaram ao longo da elaboracao
com sugestoes variadas. Mas devemos um agradecimento muito especial
a nosso amigo e colega Eduardo Tengan. ET e nosso co-autor no livro de
Teoria dos N
umeros publicado pelo projeto Euclides. O livro do Projeto
Euclides serviu de ponto de partida para a elaboracao deste livro e nos
tres achamos natural que ET tambem fosse contado como co-autor desta
vez: apenas por insistencia dele aceitamos preparar o livro sem o nome
dele. De qualquer forma devemos registrar aqui que sem a participacao
de ET nao existiriam nem o livro do Projeto Euclides nem este.

Captulo 0

Fundamentos
Neste captulo preliminar recordaremos alguns topicos fundamentais:
o Princpio da Induc
ao Finita e o Princpio da Casa dos Pombos, MDC,
Congruencias e os Teoremas de Fermat e de Euler.

0.1

Princpio da Induc
ao Finita

Seja P (n) uma propriedade do n


umero natural n, por exemplo:
n pode ser fatorado em um produto de n
umeros primos;
1 + 2 + + n =

n(n+1)
;
2

a equacao 2x + 3y = n admite solucao com x e y inteiros positivos.


Uma maneira de provar que P (n) e verdadeira para todo natural
n n0 e utilizar o chamado Princpio da Induc
ao Finita (PIF), que e
um dos axiomas que caracterizam o conjunto dos n
umeros naturais. O
PIF consiste em verificar duas coisas:
1. (Base da Inducao) P (n0 ) e verdadeira e
2. (Passo Indutivo) Se P (n) e verdadeira para algum n
umero natural
n n0 , entao P (n + 1) tambem e verdadeira.
Na base da inducao, verificamos que a propriedade e valida para
um valor inicial n = n0 . O passo indutivo consiste em mostrar como
utilizar a validade da propriedade para um dado n (a chamada hip
otese
de induc
ao) para provar a validade da mesma propriedade para o inteiro
seguinte n + 1. Uma vez verificados a base e o passo indutivo, temos
2

FINITA
0.1. PRINCIPIO DA INDUC
AO

uma cadeia de implicacoes


passo
indutivo

P (n0 ) e verdadeira (base) = P (n0 + 1) e verdadeira


passo
indutivo

= P (n0 + 2) e verdadeira

passo
indutivo

= P (n0 + 3) e verdadeira

..
.
de modo que P (n) e verdadeira para todo natural n n0 .
Vejamos alguns exemplos.
Exemplo 0.1. Demonstrar que, para todo inteiro positivo n,
1 + 2 + + n =

n(n + 1)
.
2

o: Observemos que 1 = 12
Solu
ca
2 donde a igualdade vale para n = 1
(base da inducao). Agora suponha que a igualdade valha para n = k
(hip
otese de inducao):
1 + 2 + + k =

k(k + 1)
2

Somando k + 1 a ambos lados da igualdade, obtemos


1 + 2 + + k + (k + 1) =

k(k + 1)
(k + 1)(k + 2)
+ (k + 1) =
2
2

de modo que a igualdade tambem vale para n = k + 1. Pelo PIF, a


igualdade vale para todo n
umero natural n 1.
Exemplo 0.2. Demonstrar que, para todo n
umero natural n,
Mn = n(n2 1)(3n + 2)
e m
ultiplo de 24.
o: Veja que se n = 0 entao M0 = 0, que e um m
Solu
ca
ultiplo de 24
(base da inducao).
Agora, suponhamos que para certo inteiro k o n
umero Mk e divisvel por 24 (hip
otese de inducao) e vamos mostrar que Mk+1 tambem e
divisvel por 24 (passo indutivo). Calculamos primeiramente a diferenca


Mk+1 Mk = (k + 1) (k + 1)2 1 3(k + 1) + 2 k(k 2 1)(3k + 2)
= k(k + 1)[(k + 2)(3k + 5) (k 1)(3k + 2)]

= 12k(k + 1)2 .

CAPITULO 0. FUNDAMENTOS

Um dos n
umeros naturais consecutivos k e k + 1 e par donde k(k + 1)2
e sempre par e 12k(k + 1)2 e divisvel por 24. Por hip
otese de inducao,
Mk e divisvel por 24 e temos portanto que Mk+1 = Mk + 12k(k + 1)2
tambem e divisvel por 24, como se queria demonstrar.
Uma variante do PIF e a seguinte versao (`as vezes apelidada de
princpio de induc
ao forte ou princpio de induc
ao completa), em que
se deve mostrar
1. (Base da Inducao) P (n0 ) e verdadeira e
2. (Passo Indutivo) Se P (k) e verdadeira para todo natural k tal que
n0 k n, entao P (n + 1) tambem e verdadeira.
Exemplo 0.3. A sequencia de Fibonacci Fn e a sequencia definida recursivamente por
F0 = 0,

F1 = 1

Fn = Fn1 + Fn2 para n 2

Assim, seus primeiros termos s


ao
F0 = 0, F1 = 1, F2 = 1, F3 = 2, F4 = 3, F5 = 5, F6 = 8, . . .
Mostre que
Fn =
onde =

1+ 5
2

o:
Solu
ca

e=

1 5
2

n n

s
ao as razes de x2 = x + 1.

Temos que F0 =

0 0

1 1
= 1 (base de
k k
para todo k com

= 0 e F1 =

inducao). Agora seja n 1 e suponha que Fk =


0 k n (hip
otese de inducao). Assim,
Fn+1 = Fn + Fn1

n n n1 n1
+

n
n1
( +
) ( n + n1 )
n+1 n+1
=
=

pois 2 = + 1 = n+1 = n + n1 e analogamente n+1 =


n + n1 .
Observe que, neste exemplo, como o passo indutivo utiliza os valores
de dois termos anteriores da sequencia de Fibonacci, a base requer verificar a formula para os dois termos iniciais F0 e F1 e nao apenas para o
primeiro termo.

FINITA
0.1. PRINCIPIO DA INDUC
AO

Exemplo 0.4. Prove usando o princpio da induc


ao matem
atica que
1 3 5 (2n 1)
1

.
2 4 2n
2n + 1

o: Vejamos que se n = 1 temos que


Solu
ca
para um certo n temos

1
2

<

1 .
3

Suponhamos que

1
1 3 5 (2n 1)

.
2 4 2n
2n + 1

Para o caso seguinte temos que

1
1 3 5 (2n 1) (2n + 1) H.I.
2n + 1
<
,

2 4 2n (2n + 2)
2n + 1 2n + 2

1
2n + 1
1

. Isso sua vez


2n
2n + 3
+ 1 2n + 2
2n + 1
1

e equivalente a mostrar que
, mas, elevando ao
2n + 2
2n + 3
quadrado, tambem vai ser equivalente a mostrar (2n + 1)(2n + 3)
(2n + 2)2 , o que (expandindo os dois lados) e claramente verdadeiro, o
que conclui a prova.
e assim basta mostrar que

Exemplo 0.5. Demonstrar que, para quaisquer naturais n m, o coeficiente binomial


 
n def
n!
=
m
m!(n m)!
e inteiro.
o: Procederemos por inducao sobre a soma m+n. Se m+n = 0
Solu
ca

entao m = n = 0 e 00 = 1 e inteiro (base de inducao). Para o passo
indutivo, observe primeiramente que para 0 < m < n temos a seguinte
identidade de binomiais

 
  
n1
n1
n
+
=
m1
m
m

que segue diretamente das definicoes:



 

n1
n1
(n 1)!
(n 1)!
+
+
=
m
m1
m!(n m 1)! (m 1)!(n m)!

 
(n m) + m (n 1)!
n
=
.
=
m
m!(n m)!

n
Agora suponhamos que m
e inteiro para m + n k (hip
otese de
inducao). Note que podemos
supor
tamb
e
m
que
0
<
m
<
n,
j
a
que se

n
m = n ou m = 0 temos m
= 1 e o resultado vale trivialmente. Assim,



n
n1
se m + n = k + 1, temos que m
= n1
e inteiro tambem pois
m + m1
cada somando da direita e inteiro pela hip
otese de inducao.

CAPITULO 0. FUNDAMENTOS

Exemplo 0.6. Sejam x1 , . . . , xn n


umeros reais positivos. Neste exerccio vamos provar que

x1 + + xn
n x1 xn .
n
Tal desigualdade e conhecida como desigualdade das medias aritmetica
e geometrica.
(a) Utilize o PIF para mostrar a desigualdade das medias para n = 2k .
n
a media arit(b) Sejam x1 , . . . , xn reais positivos fixados e A = x1 ++x
n
metica destes n
umeros. Suponha que a desigualdade valha para n+1
n
umeros reais positivos quaisquer; aplicando-a para x1 , . . . , xn , A,
conclua que a desigualdade vale tambem para quaisquer n n
umeros
reais positivos.

(c) Combinando os itens anteriores, prove a desigualdade para todo n


natural.
o: Primeiro observemos que se a, b > 0 entao
Solu
ca
0 (a b)2 = a2 2ab + b2 = (a + b)2 4ab,

logo ab a+b
2 , assim a desigualdade vale para n = 2. Agora mostremos
que se a desigualdade vale para k entao a desigualdade vale para 2k. De
fato
x

++x

x1 ++xk
+ k+1 k 2k
x1 + + x2k
k
=
2k

2
k x x + k x
1
k
k+1 x2k

2
q

x1 xk k xk+1 x2k = 2k x1 x2k .

Assim a desigualdade e verdadeira para 2, 4, 8, . . . , 2n , . . .. Suponhamos


que a desigualdade e verdadeira para n + 1, e sejam x1 , . . . , xn reais

n
e G = n x1 xn , temos que mostrar
positivos, definamos A = x1 ++x
n
que A G, mas de fato sabemos que
A=

x1 + + xn + A

n+1

p
1
n
x1 xn A = G n+1 A n+1 .

n+1

Daqui facilmente conclumos o que queramos demonstrar.

FINITA
0.1. PRINCIPIO DA INDUC
AO

Um terceiro disfarce do PIF e o chamado princpio da boa ordenac


ao (PBO) dos n
umeros naturais, que afirma que todo subconjunto A
nao vazio de N tem um elemento mnimo. (Voce sabe dizer por que o
princpio da boa ordem nao vale para o conjunto Z de todos os inteiros?)
Vejamos a equivalencia entre os dois princpios. Assuma primeiramente o PBO e seja P (n) uma propriedade para a qual P (0) e verdadeira
e P (n) verdadeira implica P (n + 1) verdadeira. Seja B o conjunto dos
n tais que P (n) e falsa; devemos mostrar que B = . Suponha que nao;
pelo PBO o conjunto B possui um menor elemento b. Como 0
/ B (pois
P (0) e verdadeira por hip
otese) temos que b 1 e assim b 1 N e pela
minimalidade de b temos que b 1
/ B, ou seja, P (b 1) e verdadeira.
Mas por hip
otese temos entao que P (b) tambem e verdadeira, o que e
um absurdo, logo B = .
Assuma agora o PIF e seja A N um subconjunto nao vazio. Defina
agora o conjunto B = {b N | a
/ A para todo a < b}. Trivialmente
0 B. Afirmamos que existe k B tal que k + 1
/ B e nesse caso k
sera o menor elemento de A. De fato, se isto nao acontecer, teremos que
0 B e k B implica que k + 1 B. Logo, pelo PIF, B = N e A = ,
o que e absurdo.
Exemplo 0.7. Demonstrar que toda func
ao f : N N mon
otona n
aocrescente (isto e, n m = f (n) f (m)) e constante a partir de um
certo n
umero natural.
o: Seja A N a imagem de f . Pelo PBO, tal conjunto possui
Solu
ca
elemento mnimo a0 . Seja n0 um natural tal que f (n0 ) = a0 . Como
a funcao e monotona nao-crescente entao para todo n n0 temos que
f (n) f (n0 ), mas pela definicao de a0 temos f (n) a0 . Logo f (n) = a0
para todo n n0 , como queramos demonstrar.
Observa
c
ao 0.8. Dado um conjunto S, uma relac
ao em S e chamada
de ordem parcial em S se ela satisfaz os seguintes axiomas:
1. (Reflexividade) a a para todo a S.
2. (Anti-simetria) se a b e b a ent
ao a = b.
3. (Transitividade) se a b e b c ent
ao a c.
Dizemos que e uma ordem total se, dados quaisquer a, b S, ou
a b ou b a. Uma ordem total em S e uma boa ordem se todo
subconjunto A de S possui um elemento mnimo, isto e, um elemento
possvel demonstrar que para
a A tal que a b para todo b A. E
todo conjunto S podemos definir uma ordem total em S que e uma boa
ordem. Este fato usa o axioma da escolha (e na verdade e equivalente a
ele) e est
a fora do prop
osito deste livro. Veja por exemplo [15].

CAPITULO 0. FUNDAMENTOS

Problemas Propostos
0.1. Demonstrar por induc
ao que para n 1 natural
(a) 12 + 22 + + n2 =

n(n+1)(2n+1)
.
6

(b) 13 + 23 + + n3 = (1 + 2 + + n)2 .
(c) (15 + 25 + + n5 ) + (17 + 27 + + n7 ) = 2(1 + 2 + + n)4 .
(d) sen x + sen 2x + + sen nx =

sen (n+1)x
sen nx
2
2
.
sen x2

0.2. Seja Fn o n-esimo termo da sequencia de Fibonacci. Demonstrar


que para todo natural n 1 temos
(a) F1 + F2 + + Fn = Fn+2 1.
(b) Fn+1 Fn1 Fn2 = (1)n .

n 

Fn+1 Fn
1 1
.
=
(c)
Fn Fn1
1 0

 n2 n3
(d) n0 + n1
1 + 2 + 3 + = Fn+1 , onde na soma interpretamos
m
k = 0 se k > m.
0.3. Demonstrar que

(a) n3 n e um m
ultiplo de 6 para todo natural n.
(b) 5n 1 e m
ultiplo de 24 para todo n
umero natural n par.
(c) 2n + 1 e m
ultiplo de 3 para todo natural mpar n.
0.4. Mostre que para todo natural n 4
(a) 2n < n!.
(b) 2n3 > 3n2 + 3n + 1.
0.5. Dado um inteiro positivo n, definimos T (n, 1) = n e, para todo
k 1, T (n, k + 1) = nT (n,k) . Prove que existe c N tal que, para todo
k 1, T (2010, k) < T (2, k + c). Determine o menor inteiro positivo c
com essa propriedade.
0.6. Mostre que para todo n e k inteiros positivos

 


 
  
n+k+1
n+k
n+2
n+1
n
.
=
+ +
+
+
n+1
n
n
n
n

0.2. PRINCIPIO DA CASA DOS POMBOS

0.7. Demonstre a f
ormula do bin
omio de Newton para n natural:
 
 


 
n n
n n1
n
n n
n
n1
(x + y) =
x +
x
y + +
xy
+
y .
0
1
n1
n
0.8. Encontrar com demonstrac
ao uma express
ao para o multin
omio
(x1 + x2 + + xk )n
em termos dos coeficientes multinomiais


n
n!
def
=
i1 ! ik !
i1 , . . . , ik
onde i1 + + ik = n.
0.9. Considere n retas em posic
ao geral em um plano, isto e, sem que
haja duas retas paralelas ou tres retas concorrentes em um mesmo ponto.
(a) Determine em func
ao de n o n
umero de regi
oes em que as retas
dividem o plano.
(b) Demonstre que e possvel colorir essas regi
oes com duas cores sem
que duas regi
oes vizinhas tenham a mesma cor (duas regi
oes s
ao
vizinhas se elas possuem um segmento de reta em comum).
0.10. Demonstrar que para cada n
umero natural n existe um n
umero
natural M satisfazendo simultaneamente as seguintes duas condic
oes:
(i) M possui n dgitos pertencentes ao conjunto {1, 2}.
(ii) M e divisvel por 2n .
0.11 (IMO1987). Mostre que n
ao existe uma func
ao f : N N tal que
f (f (n)) = n + 1987 para todo n N.

0.2

Princpio da Casa dos Pombos

intuitivamente claro que se colocamos n + 1 objetos em n gavetas


E
entao havera ao menos uma gaveta com mais de um objeto. Isto e
exatamente o que afirma o chamado Princpio da Casa dos Pombos
(PCP) ou Princpio das Gavetas de Dirichlet: se temos kn + 1 pombos
e n casinhas, entao existir
a uma casinha onde havera pelo menos k + 1
pombos. De fato, se em todas as casas houvesse no m
aximo k pombos,
entao o n
umero de pombos nao poderia ultrapassar kn.
O PCP parece bastante inocente, mas tem muitas aplicacoes interessantes, especialmente em argumentos de existencia em que nao se
determina o objeto procurado explicitamente. Como exemplos falam
mais do que 103 palavras, vejamos alguns.

10

CAPITULO 0. FUNDAMENTOS

Exemplo 0.9. Do conjunto A = {1, 2, . . . , 99, 100}, escolhemos ao acaso


51 n
umeros. Demonstrar que entre os n
umeros escolhidos sempre existem dois que s
ao consecutivos.
o: Para provar isto, primeiro escolhamos gavetas adequadas
Solu
ca
ao problema. Distribumos os n
umeros de A em 50 gavetas assim
construdas:
{1, 2} {3, 4} {5, 6}

{99, 100}

Como ha 50 gavetas das quais retiramos 51 n


umeros, sempre existir
a
uma gaveta da qual escolhemos dois n
umeros e estes, gracas `a nossa
construcao, serao consecutivos. Podemos generalizar este resultado considerando os n
umeros {1, 2, . . . , 2n} e escolhendo dentre eles n + 1 n
umeros ao acaso.

Exemplo 0.10. Do conjunto A = {1, 2, . . . , 99, 100}, escolhemos ao


acaso 55 n
umeros. Demonstrar que entre os n
umeros escolhidos sempre
existem dois tais que sua diferenca e 9.
o: Como no exemplo anterior o problema e descobrir como
Solu
ca
formar as gavetas. Consideremos as gavetas numeradas 0, 1, 2, . . . , 8,
onde o n
umero n e colocado na gaveta i se, e s
o se, o resto na divisao
de n por 9 e i. Como escolhemos 55 = 9 6 + 1 n
umeros, pelo PCP
existir
a uma gaveta j na qual ha 7 ou mais n
umeros escolhidos. Mas
em cada gaveta ha no m
aximo 12 n
umeros (por exemplo, o conjunto
{1, 10, 19, 28, 37, 46, 55, 64, 73, 82, 91, 100} possui exatamente 12 elementos). Segue, como no problema anterior, que existir
ao dois n
umeros que
serao consecutivos em tal conjunto, isto e, dois n
umeros cuja diferenca
e 9.
Exemplo 0.11. Demonstrar que qualquer conjunto de n inteiros possui
um subconjunto n
ao vazio cuja soma dos elementos e divisvel por n.
o: Sejam a1 , a2 , . . . , an os elementos do conjunto, e definamos
Solu
ca
as somas parciais sj = a1 + + aj para j = 1, . . . , n. Se algum dos
sj e divisvel por n o problema fica resolvido. Se nenhum e divisvel
por n, entao os possveis restos na divisao por n s
ao 1, 2, . . . , n 1 e
como ha n somas parciais pelo PCP existem duas sj e sk com j < k que
deixam o mesmo. Portanto sk sj = aj+1 + + ak e divisvel por n e
{aj+1 , aj+2 , . . . , ak } e o subconjunto procurado.
Por outro lado, observemos que n e a quantidade mnima de elementos para que se verifique tal condicao, no sentido em que existem

0.2. PRINCIPIO DA CASA DOS POMBOS

11

conjuntos A com n 1 elementos tais que a soma dos elementos de


todo subconjunto nao vazio de A nao e divisvel por n. Por exemplo,
A = {1, n+1, 2n+1, . . . , (n1)n+1} e um destes conjuntos (verifique!).

Exemplo 0.12. Seja um n


umero real. Demonstrar que, para todo
inteiro n 2, existe um inteiro 0 < k < n tal que o m
odulo da diferenca
entre k e seu inteiro mais pr
oximo e menor ou igual a n1 .
o: Vamos denotar por {x} a parte fracion
Solu
ca
aria do n
umero real
x, isto e, o u
nico real que satisfaz 0 {x} < 1 e x = m + {x} para
algum m Z.
Considere {k} para k = 1, 2, . . . , n. Particione o intervalo [0, 1) em
n partes de tamanho n1 :
hn 1 
h 1 h1 2 h2 3

,

,1
[0, 1) = 0,
n
n n
n n
n

Se {k} [0, n1 ) ou {k} [ n1


n , 1) para algum k = 1, . . . , n 1, o
problema acabou. Caso contrario, pelo PCP havera duas partes fracionarias {j} e {k} com 1 j < k n 1 pertencentes a um mesmo
intervalinho dentre os n 2 restantes. Sendo x = (k j), teremos
(
{k} {j}
se {k} {j}
{x} =
1 + {k} {j} se {k} < {j}
coes
e portanto {x} [0, n1 ) ou {x} [ n1
n , 1), assim kj satisfaz as condi
do problema.
Exemplo 0.13. Demonstrar que dados 7 n
umeros reais sempre e possvel escolher 2 deles, digamos a e b, tais que


ab
1


1 + ab < 3 .

o: Vejamos inicialmente que a funcao y = tan x e crescente


Solu
ca
entre ( 2 , 2 ), e alem disso, para cada real r existe um u
nico angulo
neste intervalo com r = tan .
Portanto, dados os 7 n
umeros reais, a cada um deles podemos fazer
corresponder um
angulo no intervalo ( 2 , 2 ), e dividindo tal intervalo
em 6 partes iguais, i.e., em 6 intervalos de comprimento 6 , abertos `a
esquerda, existir
ao 2
angulos e que estejam no mesmo intervalo, e
portanto, | | < 6 .

CAPITULO 0. FUNDAMENTOS

12

y = arctan x

2
Podemos supor sem perda de generalidade que a = tan > tan = b
e como a funcao tangente e crescente,
tan tan

1
ab
=
= tan( ) < tan = ,
1 + ab
1 + tan tan
6
3
como queramos demonstrar.

Problemas Propostos
0.12. Escolhem-se 7 pontos no interior de um ret
angulo de dimens
oes
2 3. Demonstrar que sempre e
possvel encontrar dois pontos tal que
sua dist
ancia e menor ou igual a 2.
0.13. Escolhem-se 9 pontos no interior de um quadrado de lado 1. Demonstrar que e possvel escolher 3 deles de tal forma que a a
rea do
tri
angulo que formam e menor ou igual a 81 .
0.14. Dadas 6 pessoas numa festa, demonstrar que necessariamente
existem 3 pessoas que se conhecem mutuamente ou 3 pessoas que n
ao
se conhecem mutuamente. Suponha que a relac
ao de conhecer e simetrica. Este e um caso particular do teorema de Ramsey, veja por exemplo
[8].
0.15. Do conjunto A = {1, 2, . . . , 99, 100} escolhemos 51 n
umeros. Demonstrar que, entre os 51 n
umeros escolhidos, existem dois tais que um
e m
ultiplo do outro.
0.16. Dado um n
umero irracional u, demonstrar que sempre e possvel
encontrar infinitos n
umeros racionais pq , p, q Z, de tal forma que




u p < 1 .

q q2

13

0.3. DIVISIBILIDADE
Um problema mais difcil e demonstrar existem racionais
que




u p < 1 .

q
5q 2

p
q

de tal forma

Veja o teorema 3.13 e a sec


ao correspondente para este e outros resultados relacionados `
a aproximac
ao de n
umeros reais por n
umeros racionais.

0.17 (IMO1985). Dado um conjunto M com 1985 inteiros positivos distintos, nenhum dos quais tem divisores primos maiores do que 23, mostre
que h
a 4 elementos em M cujo produto e uma quarta potencia.
0.18 (OIbM1998). Determinar o mnimo valor de n para o qual, de todo
subconjunto de {1, 2, . . . , 999} com n elementos, e possvel selecionar
quatro inteiros diferentes a, b, c, d tais que a + 2b + 3c = d.
0.19. Demonstrar que de qualquer conjunto de 2n+1 1 n
umeros inteiros
n
positivos e possvel escolher 2 elementos de tal forma que sua soma e
divisvel por 2n .
0.20 (IMO2001). Sejam n1 , n2 , . . . , nm inteiros com m mpar. Denotemos por x = (x1 , . . . , xm ) uma permutac
ao dos inteiros 1, 2, . . . , m,
e definamos f (x) = x1 n1 + + xm nm . Demonstre que existem duas
permutac
oes a e b tais que f (a) f (b) e divisvel por m!.
0.21 (IMO1991). Seja S = {1, 2, . . . , 280}. Encontrar o menor inteiro
n para o qual todo subconjunto de S com n elementos contem cinco
n
umeros que s
ao dois a dois primos entre si.
0.22 (Erdos). Mostrar que toda a sequencia com n2 + 1 n
umeros reais contem ou uma subsequencia crescente com n + 1 termos ou uma
subsequencia decrescente com n + 1 termos.
0.23. Pintamos todos os pontos do plano de azul, verde ou preto. Mostrar que existe no plano um ret
angulo cujos vertices tem todos a mesma
cor.

0.3

Divisibilidade

Dados dois inteiros d e a, dizemos que d divide a ou que d e um


divisor de a ou ainda que a e um m
ultiplo de d e escrevemos
d|a
se existir q Z com a = qd. Caso contrario, escrevemos d a. Por
exemplo, temos que 5 | 10 mas 10 5.
Eis algumas propriedades importantes da divisibilidade:

CAPITULO 0. FUNDAMENTOS

14

Lema 0.14. Sejam a, b, c, d Z. Temos


(i) (d divide) Se d | a e d | b, ent
ao d | ax + by para qualquer
combinac
ao linear ax + by de a e b com coeficientes x, y Z.
(ii) (Limitac
ao) Se d | a, ent
ao a = 0 ou |d| |a|.
(iii) (Transitividade) Se a | b e b | c, ent
ao a | c.
o: Se d | a e d | b, entao podemos escrever a = dq1 e
Demonstra
ca
b = dq2 com q1 , q2 Z, logo ax+by = d(q1 x+q2 y). Como q1 x+q2 y Z,
temos d | ax + by. Para mostrar (ii), suponha que d | a e a 6= 0. Neste
caso, a = dq com q 6= 0, assim |q| 1 e |a| = |d||q| |d|. Finalmente,
se a | b e b | c, entao existem q1 , q2 Z tais que b = aq1 e c = bq2 , logo
c = aq1 q2 e portanto a | c.
Vejamos como utilizar estas propriedades para resolver alguns problemas de divisibilidade.
Exemplo 0.15. Encontre todos os inteiros positivos n tais que 2n2 + 1 |
n3 + 9n 17.
o: Utilizando o 2n2 + 1 divide para reduzir o grau de n3 +
Solu
ca
9n 17, temos que
(
2n2 + 1 | n3 + 9n 17
2n2 + 1 | 2n2 + 1
= 2n2 + 1 | (n3 + 9n 17) 2 + (2n2 + 1) (n)

2n2 + 1 | 17n 34

Como o grau de 17n 34 e menor do que o de 2n2 + 1, podemos utilizar


a limitacao para obter uma lista finita de candidatos a n. Temos
17n 34 = 0 n = 2 ou |2n2 + 1| |17n 34| n = 1, 4 ou 5.
Destes candidatos, apenas n = 2 e n = 5 s
ao solucoes.
Exemplo 0.16 (IMO1994). Determine todos os pares (m, n) de inteiros
n3 +1
e inteiro.
positivos para os quais mn1
o:
Solu
ca
Temos

Vamos tentar reduzir o grau em n utilizando o d divide.

mn 1 | n3 + 1 = mn 1 | (n3 + 1) m (mn 1) n2
mn 1 | n2 + m.

15

0.3. DIVISIBILIDADE
Da mesma forma,
mn 1 | n2 + m = mn 1 | (n2 + m) m (mn 1) n
mn 1 | m2 + n

e, finalmente,
mn 1 | m2 + n = mn 1 | (m2 + n) m (mn 1)
mn 1 | m3 + 1

que e a mesma expressao com que comecamos, trocando n por m. Assim,


temos que a condicao e simetrica em m e n e as divisibilidades acima
s
ao todas equivalentes entre si. Portanto podemos supor sem perda de
generalidade que m n. Utilizando a limitacao temos
mn 1 | n2 + m = mn 1 n2 + m m(n 1) n2 + 1.
2

2
+1
= n + 1 + n1
. Como estamos assumindo
Se n 6= 1 temos m nn1
m n, se n 4 temos apenas duas possibilidades: m = n ou m = n+1.
Agora temos alguns casos a analisar.

Se m n = 1 devemos ter m 1 | 12 + m = m 1 | m + 1
(m 1) m 1 | 2 e portanto m = 2 ou m = 3, ambos os
casos fornecendo solucoes.
Se m n = 2 devemos ter 2m 1 | 22 + m = 2m 1 |
2(m + 4) (2m 1) 2m 1 | 9 m = 2 ou m = 5,
ambos os casos fornecendo solucoes.
Se m n = 3 devemos ter 3m 1 | 32 + m = 3m 1 |
3(m + 9) (3m 1) 3m 1 | 28 m = 5, que fornece
uma solucao.
Se m = n 4 devemos ter

n2 1 | n2 + n n 1 | n

= n 1 | n (n 1) n 1 | 1

o que nao e possvel pois n 4.


Se m = n + 1 5 devemos ter

(n + 1)n 1 | n2 + (n + 1)

n2 + n 1 | (n2 + n + 1) (n2 + n 1)
n2 + n 1 | 2

o que novamente nao e possvel pois n 4.

Logo as solucoes (m, n) s


ao (1, 2), (2, 1), (1, 3), (3, 1), (2, 2), (2, 5), (5, 2),
(3, 5), (5, 3).

CAPITULO 0. FUNDAMENTOS

16

0.4

mdc, mmc e Algoritmo de Euclides

Dados dois n
umeros inteiros a e b com a 6= 0 ou b 6= 0, a cada
um deles pode-se associar seu conjunto de divisores positivos, Da e Db
respectivamente, e a interseccao de tais conjuntos Da Db e finita (pela
limitacao) e nao vazia (ja que 1 pertence `
a interseccao). Por ser finito,
Da Db possui elemento m
aximo, que e chamado de m
aximo divisor
comum (mdc) dos n
umeros a e b. Denotamos este n
umero por mdc(a, b)
(alguns autores usam a notacao (a, b)). Para a = b = 0 convencionamos
mdc(0, 0) = 0. Quando mdc(a, b) = 1 dizemos que a e b s
ao primos
entre si.
Por outro lado, se denotamos por Mn o conjunto dos m
ultiplos positivos de n, dados dois n
umeros inteiros a e b com a 6= 0 e b 6= 0,
entao a interseccao Ma Mb e nao vazia (ja que |ab| esta na interseccao). Como os naturais s
ao bem ordenados, Ma Mb possui elemento
mnimo. Tal n
umero e chamado mnimo m
ultiplo comum (mmc) de a e
b e o denotaremos por mmc(a, b) (alguns autores escrevem [a, b]).
Para calcularmos o mdc e o mmc de maneira eficiente, vamos descrever o chamado algoritmo de Euclides ou algoritmo das divis
oes sucessivas. Primeiramente, vamos relembrar o conceito de divis
ao euclidiana,
ou divis
ao com resto, que e uma das quatro operacoes que toda crianca
aprende na escola. Sua formulacao precisa e: dados a, b Z com b 6= 0,
existem q, r Z com
a = bq + r

0 r < |b|.

Tais q e r estao unicamente determinados pelas duas condicoes acima


(veja o argumento a seguir) e s
ao chamados o quociente e resto da divisao
de a por b. O resto r e tambem denotado por a mod b.
Para x R, definimos o piso ou parte inteira x de x como sendo
ou
nico k Z tal que k x < k + 1; definimos o teto x dex como
ou
nico k Z tal que k 1 < x k. Por exemplo, temos 2 = 1,
2 = 2, 10 = 10 = 10, = 4 e = 3. Podemos agora
mostrar a existencia de q e r satisfazendo as duas condicoes acima: basta
tomar
(
a/b se b > 0
q=
e
r = a bq em ambos os casos
a/b se b < 0
e e facil verificar que 0 r < |b| a partir das definicoes das funcoes piso
e teto. Por outro lado, se a = bq1 + r1 = bq2 + r2 com 0 r1 , r2 < |b|,
entao temos que r2 r1 = b(q1 q2 ) e um m
ultiplo de b com |r2 r1 | < |b|,
portanto r2 r1 = 0 e assim q1 = q2 tambem, o que prova a unicidade.
Podemos agora descrever o algoritmo de Euclides para calcular o
mdc, que se baseia na seguinte simples observacao:

17

0.4. MDC, MMC E ALGORITMO DE EUCLIDES


Lema 0.17 (Euclides). Se a = bq + r, ent
ao mdc(a, b) = mdc(b, r).

o: Basta mostrar que Da Db = Db Dr , ja que se


Demonstra
ca
estes conjuntos forem iguais em particular os seus m
aximos tambem
serao iguais. Se d Da Db temos d | a e d | b, logo d | a bq d | r
e portanto d Db Dr . Da mesma forma, se d Db Dr temos d | b e
d | r, logo d | bq + r d | a e assim d Da Db .
O algoritmo de Euclides consiste na aplicacao reiterada do lema
acima onde q e r s
ao o quociente e o resto na divisao de a por b (note que
o lema vale mesmo sem a condicao 0 r < |b|). Como os restos formam
uma sequencia estritamente decrescente, o algoritmo eventualmente para
quando atingimos o resto 0.
Exemplo 0.18. Calcule mdc(1001, 109).
o: Realizando as divisoes sucessivas, temos
Solu
ca
1001 = 109 9 + 20
109 = 20 5 + 9
20 = 9 2 + 2

9=24+1

2=12+0

Assim, temos mdc(1001, 109) = mdc(109, 20) = mdc(20, 9) = mdc(9, 2) =


mdc(2, 1) = mdc(1, 0) = 1.
Exemplo 0.19. Sejam m 6= n dois n
umeros naturais. Demonstrar que
(
1 se a e par,
2m
2n
mdc(a + 1, a + 1) =
2 se a e mpar.
o: Suponha sem perda de generalidade que m > n e observe a
Solu
ca
fatoracao
m

m1

a2 1 = (a2
m

m2

+ 1)(a2

m3

+ 1)(a2

+ 1) . . . (a2 + 1)(a2 1)

Logo a2 + 1 = (a2 + 1) q + 2 com q Z e assim


m

mdc(a2 + 1, a2 + 1) = mdc(a2 + 1, 2)
n

que e igual a 2 se a2 + 1 for par, isto e, se a for mpar, e e igual a 1


caso contrario.

18

CAPITULO 0. FUNDAMENTOS

Alem de servir de ferramenta computacional para o calculo do mdc,


a divisao euclidiana tem consequencias te
oricas importantes. O proximo
teorema mostra que e sempre possvel escrever o mdc de dois n
umeros
como combinacao linear destes (com coeficientes inteiros).
Teorema 0.20 (Bachet-Bezout). Sejam a, b Z. Ent
ao existem x, y
Z com
ax + by = mdc(a, b).
Portanto se c Z e tal que c | a e c | b ent
ao c | mdc(a, b).
o: O caso a = b = 0 e trivial (temos x = y = 0). Nos
Demonstra
ca
outros casos, considere o conjunto de todas as combinacoes Z-lineares
de a e b:
def
I(a, b) = {ax + by : x, y Z}
Seja d = ax0 + by0 o menor elemento positivo de I(a, b) (ha pelo menos
um elemento positivo, verifique!). Afirmamos que d divide todos os
elementos de I(a, b). De fato, dado m = ax + by I(a, b), sejam q, r Z
o quociente e o resto na divisao euclidiana de m por d, de modo que
m = dq + r e 0 r < d. Temos
r = m dq = a(x qx0 ) + b(y qy0 ) I(a, b).
Mas como r < d e d e o menor elemento positivo de I(a, b), segue que
r = 0 e portanto d | m.
Em particular, como a, b I(a, b) temos que d | a e d | b, logo d
mdc(a, b). Note ainda que se c | a e c | b, entao c | ax0 + by0 c | d.
Tomando c = mdc(a, b) temos que mdc(a, b) | d o que, juntamente com
a desigualdade d mdc(a, b), mostra que d = mdc(a, b).
Corol
ario 0.21. Sejam a, b, c Z. A equac
ao
ax + by = c
admite soluc
ao inteira em x e y se, e somente se, mdc(a, b) | c.
o: Se a equacao admite solucao inteira, entao mdc(a, b)
Demonstra
ca
divide o lado esquerdo, logo deve dividir o direito tambem. Reciprocamente, se mdc(a, b) | c, digamos c = k mdc(a, b) com k Z, pelo
teorema acima existem inteiros x0 e y0 tais que ax0 + by0 = mdc(a, b) e
multiplicando tudo por k obtemos que x = kx0 e y = ky0 s
ao solucoes
da equacao dada.

0.4. MDC, MMC E ALGORITMO DE EUCLIDES

19

Temos uma outra importante consequencia do teorema anterior:


Proposi
c
ao 0.22. Se mdc(a, b) = 1 e a | bc, ent
ao a | c.
o: Como mdc(a, b) = 1, existem x, y Z tais que
Demonstra
ca
ax + by = 1 = a cx + (bc) y = c. Do fato de a dividir cada termo
do lado esquerdo, temos que a | c.
Lembramos que um natural p > 1 e chamado primo se os u
nicos
divisores positivos de p s
ao 1 e p e um natural n > 1 e chamado composto
se admite outros divisores alem de 1 e n. Observemos que 1 nao e nem
primo nem composto.
Claramente, se p e primo e p a temos mdc(p, a) = 1. Usando a
proposicao anterior e inducao temos o seguinte resultado:
Corol
ario 0.23. Seja p um n
umero primo e sejam a1 , . . . am Z. Se
p | a1 am , ent
ao p | ai para algum i, 1 i m.
O proximo lema resume algumas propriedades u
teis do mdc:
Lema 0.24. Temos
1. Se p e primo, ent
ao mdc(a, p) e 1 ou p.
2. Se k e um inteiro, ent
ao mdc(a, b) = mdc(a kb, b).
3. Se a | c, ent
ao mdc(a, b) | mdc(c, b).
4. Se mdc(a, b) = 1, ent
ao mdc(ac, b) = mdc(c, b).
o: O primeiro item e claro e o segundo e apenas uma
Demonstra
ca
reformulacao do lema 0.17. Para provar o terceiro item, observe que
mdc(a, b) | a e a | c implicam que mdc(a, b) | c. Como tambem temos mdc(a, b) | b, conclumos que mdc(a, b) | mdc(b, c) por BachetBezout. Finalmente, para mostrar o u
ltimo item, note primeiro que
mdc(c, b) | mdc(ac, b) pois mdc(c, b) divide simultaneamente ac e b. Reciprocamente, para mostrar que mdc(ac, b) | mdc(c, b), podemos escrever
ax + by = 1 com x, y Z por Bachet-Bezout. Assim, mdc(ac, b) divide
ac x + b cy = c e tambem divide b, logo divide mdc(c, b).
Vejamos como podemos usar as propriedades acima para solucionar
o seguinte
Exemplo 0.25. Sejam an = 100 + n2 e dn = mdc(an , an+1 ). Calcular
dn para todo n.

CAPITULO 0. FUNDAMENTOS

20

o: Aplicando a propriedade 2 temos que


Solu
ca
dn = mdc(100 + n2 , 100 + (n + 1)2 ) = mdc(100 + n2 , 2n + 1).
Como 2n + 1 e mpar, mdc(4, 2n + 1) = 1 e pelas propriedades 4 e 2
temos que
dn = mdc(400 + 4n2 , 2n + 1)
= mdc(400 + 4n2 (2n + 1)(2n 1), 2n + 1)
= mdc(401, 2n + 1).

Como 401 e primo, entao mdc(401, 2n + 1) = 401 se 2n + 1 = 401k (com


k = 2r + 1 inteiro mpar) e mdc(401, 2n + 1) = 1 caso contrario, ou seja,
(
401 se n = 401r + 200 com r Z
dn =
1
caso contrario.

A proxima proposicao conecta o mdc e o mmc de dois inteiros e pode


ser utilizada, juntamente com o algoritmo de Euclides, para o calculo
eficiente do mmc.
Proposi
c
ao 0.26. Sejam a e b dois n
umeros naturais, ent
ao
mdc(a, b) mmc(a, b) = a b.
o: Escreva d = mdc(a, b) e a = a1 d e b = b1 d onde
Demonstra
ca
a1 , b1 Z s
ao tais que mdc(a1 , b1 ) = 1. Temos mmc(a, b) = al para
algum l Z; alem disso, b | mmc(a, b) b1 d | a1 dl b1 | a1 l.
Como mdc(a1 , b1 ) = 1, isto implica que b1 | l pela proposicao 0.22.
Pela definicao de mnimo m
ultiplo comum, temos que l deve ser o mnimo n
umero divisvel por b1 , assim conclumos que l = b1 e portanto
mmc(a, b) = b1 a. Logo mdc(a, b) mmc(a, b) = d b1 a = a b.
A demonstracao que demos do teorema de Bachet-Bezout nao mostra
como efetivamente encontrar uma solucao de ax + by = mdc(a, b). Porem, isto pode ser feito utilizando-se o algoritmo de Euclides, como mostra o exemplo a seguir. De fato, este exemplo pode servir como ponto de
partida para uma segunda demonstracao do teorema de Bachet-Bezout
(veja os exerccios).
Exemplo 0.27. Encontre todos os x, y Z tais que
1001x + 109y = mdc(1001, 109).

0.4. MDC, MMC E ALGORITMO DE EUCLIDES

21

o: Fazemos as divisoes sucessivas para o calculo de


Solu
ca
mdc(1001, 109) = 1
utilizando o algoritmo de Euclides (veja o exemplo 0.18). Em seguida,
isolamos os restos:
20 = 1001 109 9
9 = 109 20 5
2 = 20 9 2
1 = 9 2 4

Note que a u
ltima divisao permite expressar o mdc 1 como combinacao
linear de 9 e 2:
9 1 2 4 = 1.
Mas da pen
ultima divisao, temos que 2 = 20 9 2, logo substituindo
esta expressao na combinacao linear acima, temos
9 ( 20 9 2) 4 = 1 9 9 20 4 = 1
e agora expressamos 1 como combinacao linear de 20 e 9. Repetindo este
procedimento, eventualmente expressaremos 1 como combinacao linear
de 1001 e 109. Tomamos o cuidado de lembrar quais s
ao os coeficientes a e b nas equacoes ax + by = mdc(a, b) durante as simplificacoes.
Continuando, obtemos
1 = ( 109 20 5) 9 20 4 = 109 9 20 49

1 = 109 9 ( 1001 109 9) 49 = 1001 (49) + 109 450


Logo uma solucao da equacao 1001x + 109y = 1 e (x0 , y0 ) = (49, 450).
Para encontrar as demais, escrevemos o lado direito desta equacao utilizando a solucao particular que acabamos de encontrar:
1001x + 109y = 1001x0 + 109y0 1001(x x0 ) = 109(y y0 ).
Como mdc(1001, 109) = 1 temos pela proposicao 0.22 que 1001 divide
y y0 , ou seja, y y0 = 1001t para algum t Z e, portanto, x x0 =
109t. Assim, as solucoes da equacao dada s
ao todos os pontos da reta
1001x + 109y = 1 da forma
(x, y) = (x0 109t, y0 + 1001t) = (49, 450) + (109, 1001) t
com t Z.

CAPITULO 0. FUNDAMENTOS

22

Em geral, o raciocnio do exemplo acima mostra que se mdc(a, b) = 1


e (x0 , y0 ) e uma solucao da equacao ax + by = c, entao todas as solucoes
inteiras s
ao dadas por x = x0 bk e y = y0 + ak com k Z.
Exemplo 0.28. Sejam a, b inteiros positivos com mdc(a, b) = 1. Mostre
que para todo c Z com c > ab a b, a equac
ao ax + by = c admite
soluc
oes inteiras com x, y 0.
o: Seja (x0 , y0 ) uma solucao inteira (que existe pelo teorema
Solu
ca
de Bachet-Bezout). Devemos mostrar a existencia de um inteiro k tal
que
x = x0 bk > 1
e
y = y0 + ak > 1,
ou seja,

y0 + 1
x0 + 1
<k<
.
a
b

Mas isto segue do fato de o intervalo ( y0a+1 , x0b+1 ) ter tamanho maior
do que 1:
x0 + 1  y0 + 1  ax0 + by0 + a + b
c+a+b
=

=
> 1.
b
a
ab
ab

Exemplo 0.29. Seja n um n


umero inteiro com 1312345 n 1312463.
Determine quantas soluc
oes inteiras positivas no m
aximo tem a equac
ao
983x + 1237y = n. Determine todas elas em termos de n e de outro
par
ametro inteiro.
o: Primeiro encontremos todas as solucoes inteiras de
Solu
ca
983x + 1237y = mdc(983, 1237).
Para isto, usaremos uma tabela gerada usado o seguinte algoritmo que
calcula mdc(a, b):
1. As primeiras duas linhas da tabela s
ao preenchidas como
Resto
a
b

Quociente

x
0
1

y
1
0

2. Da terceira linha para frente, a linha i + 1 da tabela e calculada a


partir das duas linhas anteriores da seguinte forma

0.4. MDC, MMC E ALGORITMO DE EUCLIDES


Resto
ri1
ri
ri+1

Quociente
qi1
qi
qi+1

x
xi1
xi
xi1 qi+1 xi

23

y
yi1
yi
yi1 qi+1 yi

onde qi+1 e ri+1 s


ao respectivamente o quocente e o resto da divisao
de ri1 por ri .
Usando esta construcao podemos provar por inducao que, em cada linha
da tabela, temos ri = axi + byi . Assim, quando rl+1 = 0, temos que
rl = mdc(a, b).
Em nosso caso especifico, geramos a seguinte tabela
Resto
1237
983
254
221
33
23
10
3
1

Quociente

1
3
1
6
1
2
3

x
0
1
1
4
5
34
39
112
375

y
1
0
1
3
4
27
31
89
298

Uma solucao de 983x + 1237y = n (onde n e um inteiro fixo entre


1312345 e 1312463) e x0 = (
375n e y0 = 298n. Portanto, todas as
x = 375n + 1237k
solucoes desta equacao s
ao
e k e um inteiro
y = 298n 983k
arbitr
ario. Como estamos interessados nas solucoes positivas, devemos
ter
375n + 1237k > 0 e 298n 983k > 0,
isto e

375n
e
1237
Assim, esta equacao tem no m
aximo
k>

k<

298n
.
983

n
1312463
298n 375n

=2
983
1237
1215971
1215971

solucoes positivas. Como, para n = 1312447, 375n


1237 < 397872 < 397873 <
298n
,
temos
nesse
caso
2
solu
c

o
es
(correspondentes
a k = 397872 e k =
983
397873). Assim, o n
umero m
aximo de solucoes inteiras positivas de uma
tal equacao e 2.

CAPITULO 0. FUNDAMENTOS

24

0.5

O Teorema Fundamental da Aritm


etica

Estamos agora prontos para enunciar o teorema que caracteriza todo


n
umero natural em termos de seus constituintes primos.
Teorema 0.30 (Teorema Fundamental da Aritmetica). Seja n 2 um
n
umero natural. Podemos escrever n de uma u
nica forma como um
produto
n = p1 pm
onde m 1 e um natural e p1 . . . pm s
ao primos.

o: Mostramos a existencia da fatoracao de n em primos


Demonstra
ca
por inducao. Se n e primo nao ha o que provar (escrevemos m = 1,
p1 = n). Se n e composto podemos escrever n = ab, a, b N, 1 < a < n,
1 < b < n. Por hip
otese de inducao, a e b se decompoem como produto
de primos. Juntando as fatoracoes de a e b (e reordenando os fatores)
obtemos uma fatoracao de n.
Vamos agora mostrar a unicidade. Suponha por absurdo que n possui
duas fatoracoes diferentes
n = p1 pm = q 1 q m ,
com p1 . . . pm , q1 . . . qm e que n e mnimo com tal propriedade. Como p1 | q1 qm temos p1 | qi para algum valor de i pelo
corolario 0.23. Logo, como qi e primo, p1 = qi e p1 q1 . Analogamente
temos q1 p1 , donde p1 = q1 . Mas
n/p1 = p2 pm = q2 qm
admite uma u
nica fatoracao, pela minimalidade de n, donde m = m e
pi = qi para todo i, o que contradiz o fato de n ter duas fatoracoes.
Outra forma de escrever a fatoracao acima e
n = pe11 . . . pemm ,
com p1 < < pm e ei > 0. Ainda outra formulacao e escrever
n = 2e2 3e3 5e5 . . . pep . . .

onde o produto e tomado sobre todos os primos mas apenas um n


umero
finito de expoentes e maior do que zero. Vamos nos referir a qualquer
destas expressoes como a fatorac
ao can
onica de n em primos.
A fatoracao u
nica em primos se aplica em contextos mais gerais,
como veremos mais tarde. Aqui, como aplicacao imediata do Teorema
Fundamental da Aritmetica, vamos mostrar a prova atribuda a Euclides
para a existencia de infinitos primos (uma prova com mais de 2000 anos
e que ainda funciona!).


0.5. O TEOREMA FUNDAMENTAL DA ARITMETICA

25

Teorema 0.31 (Euclides). Existem infinitos primos.


o: Suponha por absurdo que p1 , p2 , . . . , pm fossem toDemonstra
ca
dos os primos. O n
umero N = p1 p2 . . . pm + 1 > 1 nao seria divisvel
por nenhum primo pi , o que contradiz o Teorema Fundamental da Aritmetica.
Observe que n
ao provamos que p1 p2 . . . pm + 1 e primo para algum
conjunto finito de primos (por exemplo, os m primeiros primos). Ali
as,
2 3 5 7 11 13 + 1 = 30031 = 59 509 nao e primo. N
ao se conhece
nenhuma formula simples que gere sempre n
umeros primos.
Embora a quantidade de primos seja infinita, uma quest
ao natural e
saber o quao raros ou frequentes eles s
ao. Na segunda parte do livro,
discutiremos mais a fundo esta quest
ao sobre a distribuicao dos primos.
Por outro lado, e interessante notar que existem cadeias arbitrariamente
longas de n
umeros compostos consecutivos: na sequencia
(k + 1)! + 2, (k + 1)! + 3, (k + 1)! + 4, . . . , (k + 1)! + (k + 1),
nenhum termo e primo, pois eles admitem fatores proprios 2, 3, 4, . . . , k+
1, respectivamente.
Uma interessante prova alternativa, devida a Erdos, de que existem
infinitos primos e a seguinte:
Suponha, por contradicao, que existe um n
umero finito de primos,
digamos p1 , p2 , . . . , pk . Seja n um n
umero natural. Entao podemos escrever qualquer n
umero m n na forma m = m21 m2 , onde m21 n
e
m2 = pa11 pa22 pakk
onde ak = 0 ou 1 para cada k.
Assim, considerando todas as possveis maneiras de escrever os naturais

m n, temos: 2k escolhas para m2 e no m


aximo n escolhas para
m1 . Ou seja, para todo n natural, vale que

n 2k n
absurdo, pois esta desigualdade nao vale para n suficientemente grande.
Exemplo 0.32 (OIbM1987). A sequencia pn e definida da seguinte
forma:
(i) p1 = 2.
(ii) Para todo n 2, pn e o maior divisor primo da express
ao
p1 p2 p3 pn1 + 1.

CAPITULO 0. FUNDAMENTOS

26
Demonstrar que pn e diferente de 5.

o: Dado que p1 = 2, p2 = 3, p3 = 7, segue-se que para qualquer


Solu
ca
n 3, p1 p2 pn1 e m
ultiplo de 2 e de 3, portanto p1 p2 pn1 + 1
nao e m
ultiplo nem de 2 nem de 3. Alem disso, como p1 = 2, entao pn
e mpar para todo n 2, assim p1 p2 pn1 nao e m
ultiplo de 4.
Suponhamos que exista n tal que pn = 5, isto e, o maior divisor
primo de p1 p2 pn1 + 1 e 5. Como 2 e 3 nao dividem p1 p2 pn1 + 1,
temos que
p1 p2 pn1 + 1 = 5k .
Portanto
p1 p2 pn1 = 5k 1 = (5 1)(5k1 + 5k2 + + 5 + 1),
donde 4 | p1 p2 pn1 , uma contradicao.
Exemplo 0.33. Determine todas as ternas (a, b, c) de inteiros positivos
tais que a2 = 2b + c4 .
o: Como a2 = 2b + c4 (a c2 )(a + c2 ) = 2b , pelo Teorema
Solu
ca
Fundamental da Aritmetica existem dois naturais m > n tais que m +
n = b, a c2 = 2n e a + c2 = 2m . Subtraindo as duas u
ltimas equacoes,
2
m
n
2
n1
mn
obtemos que 2c = 2 2 , assim c = 2
(2
1). Como 2n1 e
mn
2
1 s
ao primos entre si e o seu produto e um quadrado perfeito (i.e.
os expoentes das potencias de primos distintos s
ao pares), novamente
pelo Teorema Fundamental da Aritmetica 2n1 e 2mn 1 devem ser
ambos quadrados perfeitos, logo n 1 e par e 2mn 1 = (2k 1)2 para
algum inteiro positivo k. Como 2mn = (2k 1)2 + 1 = 4k(k 1) + 2 e
divisvel por 2 mas nao por 4, temos m n = 1. Assim, fazendo n 1 =
2t, temos que todas as solucoes s
ao da forma (a, b, c) = (3 22t , 4t + 3, 2t )
com t N e e facil verificar que todos os n
umeros desta forma s
ao
solucoes.
Segue do Teorema Fundamental da Aritmetica que todo divisor de
n = pe11 . . . pemm e da forma
pd11 . . . pdmm
com 0 di ei . Assim, obtemos o outro algoritmo usual para calcular o mdc de dois n
umeros: fatoramos os dois n
umeros em primos e
tomamos os fatores comuns com os menores expoentes. Este algoritmo e
bem menos eficiente do que o de Euclides para inteiros grandes (que em
geral nao sabemos fatorar de forma eficiente computacionalmente) mas
e instrutivo saber que os dois algoritmos dao o mesmo resultado. Alem
disso, este algoritmo tem consequencias te
oricas importantes, como por
exemplo o


0.5. O TEOREMA FUNDAMENTAL DA ARITMETICA

27

Corol
ario 0.34. Se mdc(a, n) = mdc(b, n) = 1, ent
ao mdc(ab, n) = 1.
o: Evidente a partir do algoritmo descrito acima.
Demonstra
ca
Para encerrar esta secao, vejamos ainda algumas outras aplicacoes
do Teorema Fundamental da Aritmetica.
ao de n em potencias
Proposi
c
ao 0.35. Seja n = pe11 . . . pemm a fatorac
def P
k
de primos distintos pi e seja k (n) = d|n, d>0 d a soma das k-esimas
potencias dos divisores positivos de n. Ent
ao
(e +1)k

(e +1)k

p 1
1
pmm
1
k (n) = 1 k
...
.
k
pm 1
p1 1
Para k = 0, a f
ormula acima deve ser interpretada tomando-se o limite
k 0, de modo que a quantidade de divisores positivos de n e 0 (n) =
(e1 + 1) (em + 1).
o: Como a soma na definicao de k (n) percorre todos os
Demonstra
ca
dm k com 0 d e , temos a seguinte
n
umeros da forma dk = pd11 k . . . pm
i
i
fatoracao:
e1 k
k
2k
em k
k (n) = (1 + pk1 + p2k
1 + + p1 ) . . . (1 + pm + pm + + pm ).
ei k
Somando as progress
oes geometricas 1 + pki + p2k
=
i + + pi
o resultado segue.

(ei +1)k
1

pi

pki 1

Proposi
c
ao 0.36 (Fatores do Fatorial). Seja p um primo. Ent
ao a
maior potencia de p que divide n! e p onde
     
n
n
n
=
+ 2 + 3 +
p
p
p
Observe que a soma acima e finita pois os termos
zero.

n
pi

s
ao eventualmente

o: No produto n! = 1 2 . . . n, apenas os m
Demonstra
ca
ultiplos
n
ultiplos entre 1 e n.
de p contribuem com um fator p. H
a p tais m
2
Destes,
ao m
ultiplos de p contribuem com um fator p extra
 os
 que s
e ha pn2 tais fatores. Dentre estes u
ltimos, os que s
ao m
ultiplos de
3
p contribuem com mais um fator p e assim por diante, resultando na
formula acima.

CAPITULO 0. FUNDAMENTOS

28

Exemplo 0.37. Determine com quantos zeros termina 1000!.


o: O problema e equivalente a determinar qual a maior potenSolu
ca
cia de 10 que divide 1000! e como ha muito mais fatores 2 do que 5 em
1000!, o expoente desta potencia coincide com o da maior potencia de 5
que divide 1000!, ou seja,

 
 
 

1000
1000
1000
1000
+
+
+
= 249.
5
52
53
54
Assim, 1000! termina com 249 zeros.

Problemas Propostos
0.24 (IMO1959). Mostre que a frac
ao
natural.

21n+4
14n+3

e irredutvel para todo n

0.25. Encontre todos os inteiros positivos tais que


(a) n + 1 | n3 1
(b) 2n 1 | n3 + 1
(c)

1
n

1
m

1
143

(d) 2n3 + 5 | n4 + n + 1
0.26. Demonstre:
(a) se m | a b, ent
ao m | ak bk para todo natural k.
(b) se f (x) e um polin
omio com coeficientes inteiros e a e b s
ao inteiros
quaisquer, ent
ao a b | f (a) f (b).
(c) se k e um natural mpar, ent
a o a + b | a k + bk .
0.27. Mostre que
(a) 215 1 e 210 + 1 s
ao primos entre si.
(b) 232 + 1 e 24 + 1 s
ao primos entre si.
0.28. Demonstrar que (n 1)2 | nk 1 se, e s
o se, n 1 | k.
0.29 (IMO1992). Encontrar todos os inteiros a, b, c com 1 < a < b < c
tais que (a 1)(b 1)(c 1) e divisor de abc 1.

Dica: Mostrar primeiro que a 4 e considerar os possveis casos.


0.5. O TEOREMA FUNDAMENTAL DA ARITMETICA

29

0.30 (IMO1998). Determine todos os pares de inteiros positivos (a, b)


tais que ab2 + b + 7 divide a2 b + a + b.
Dica: Mostre que ab2 + b + 7 | 7a b2 e considerar tres casos: 7a b2
maior, menor ou igual a zero.
0.31. Mostre que, se n > 1, ent
ao
n
X
1
1
1
= 1 + + +
k
2
n
k=1

n
ao e um n
umero inteiro.
0.32 (OBM1997). Sejam c Q, f (x) = x2 + c. Definimos
f 0 (x) = x,

f n+1 (x) = f (f n (x)), n N.

Dizemos que x R e pre-peri


odico se {f n (x), n N} e finito. Mostre
que {x Q| x e pre-peri
odico} e finito.
0.33. Demonstrar que se mdc(a, 2n+1 ) = 2n e mdc(b, 2n+1 ) = 2n , ent
ao
n+1
n+1
mdc(a + b, 2
)=2
.
0.34. Demonstrar que se a, b, c, d, m e n s
ao inteiros tais que adbc = 1
e mn 6= 0, ent
ao
mdc(am + bn, cm + dn) = mdc(m, n).
0.35. Seja Fn o n-esimo termo da sequencia de Fibonacci.
(a) Encontrar dois n
umeros inteiros a e b tais que 233a + 144b = 1
(observe que 233 e 144 s
ao termos consecutivos da sequencia de Fibonacci).
(b) Mostre que mdc(Fn , Fn+1 ) = 1 para todo n 0.
(c) Determine xn e yn tais que Fn xn + Fn+1 yn = 1.
0.36. Sejam a e b dois inteiros positivos e d seu m
aximo divisor comum.
Demonstrar que existem dois inteiros positivos x e y tais que axby = d.
0.37. Definimos a sequencia de fracoes de Farey de ordem n como o
conjunto de frac
oes reduzidas ab tais que 0 ab 1, 1 b n. Por
exemplo a sequencia de Farey de ordem 3 e 10 , 13 , 21 , 23 , 11 .
(a) Demonstrar que se ab e dc s
ao dois termos consecutivos de uma
sequencia de Farey, ent
ao cb ad = 1.
(b) Demonstrar que se ab11 , ab22 , ab33 s
ao tres termos consecutivos de uma
+a3
sequencia de Farey, ent
ao ab22 = ab11 +b
.
3

CAPITULO 0. FUNDAMENTOS

30

0.38. Utilize induc


ao em min{a, b} e o algoritmo de Euclides para mostrar que ax + by = mdc(a, b) admite soluc
ao com x, y Z, obtendo uma
nova demonstrac
ao do teorema de Bachet-Bezout.
0.39. Sejam a e b n
umeros inteiros positivos. Considere o conjunto
C = {ax + by | x, y N}
Lembre-se de que j
a mostramos no exemplo 0.28 que todo n
umero maior
que ab a b pertence a C.
(a) Demonstre que o n
umero ab a b n
ao pertence a C.
(b) Achar a quantidade de n
umeros inteiros positivos que n
ao pertencem
a C.
0.40 (IMO1984). Dados os inteiros positivos a, b e c, dois a dois primos
entre si, demonstrar que 2abc ab bc ca e o maior n
umero inteiro
que n
ao pode expressar-se na forma xbc + yca + zab com x, y e z inteiros
n
ao negativos.
0.41 (IMO1977). Sejam a, b inteiros positivos. Quando dividimos a2 +b2
por a + b, o quociente e q e o resto e r. Encontrar todos os a, b tais que
q 2 + r = 1977.
0.42. Demonstrar que mdc(2a 1, 2b 1) = 2mdc(a,b) 1 para todo
a, b N.
Pelo algoritmo de Euclides aplicado aos expoentes, basta mostrar que
mdc(2bq+r 1, 2b 1) = mdc(2b 1, 2r 1). Mas isto segue novamente
do lema de Euclides, pois 2bq+r 1 = 2r (2bq 1) + 2r 1 e 2bq 1 =
(2b 1)(2b(q1) + 2b(q2) + + 2b + 1) e um m
ultiplo de 2b 1.
0.43. Encontrar todas as func
oes f : Z Z Z satisfazendo simultaneamente as seguintes propriedades
(i) f (a, a) = a.
(ii) f (a, b) = f (b, a).
(iii) Se a > b, ent
ao f (a, b) =

a
ab f (a

b, b).

0.44. Mostre que se n e um n


umero natural composto, ent
ao n e divisvel

por um primo p com p n.


0.45 (IMO1989). Prove que, para todo inteiro positivo n, existem n
inteiros positivos consecutivos, nenhum dos quais e potencia de primo.



0.46 (Chi1998). Encontrar todos os n para os quais 1 + n1 + n2 + n3
divide 22000 .


0.6. CONGRUENCIAS

31

0.47 (IMO2002). Sejam d1 < d2 < < dk os divisores positivos de


um inteiro n > 1. Seja d = d1 d2 + d2 d3 + + dk1 dk . Mostre que
d < n2 e encontrar todos os n para os quais d | n2 .
Temos d =
n2
n2
n2
2 ( 1 + 1 + 1 + ) = n2 ( 1
+
+

+
<
n
dk dk1
dk1 dk2
d2 d1
12
23
34
1
1
2

+ 12 13 + 31 41 + ) = n2 . Por outro lado, se p e o menor primo


2
2
que divide n2 , temos que d dk1 dk = np . Como np e o maior divisor
pr
oprio de n2 e d > dk1 dk se k > 2, temos que d | n2 se, e s
o se, n = p
e primo.
0.48 (IMO1997). Encontrar todos os pares (x, y) de inteiros positivos
2
tais que xy = y x .
oes can
onicas de
Dica: Sejam x = p1 1 . . . pnn e y = p1 1 . . . pnn as fatorac
x e y. Mostre que j = tj e x = y t para algum t Q e tente limitar
os valores de t.
n

ao
0.49. Generalizar o resultado anterior para xy = y x , onde x e y s
inteiros positivos.
0.50 (IMO1984). Sejam a, b, c, d inteiros mpares tais que 0 < a < b <
c < d e ad = bc. Demonstre que se a + d = 2k e b + c = 2m para inteiros
k e m, ent
ao a = 1.

0.6

Congru
encias

Sejam a, b, n Z. Dizemos que a e congruente a b m


odulo n, e
escrevemos
a b (mod n)

se n | a b, ou seja, se a e b deixam o mesmo resto na divisao por n.


Por exemplo, temos que 17 3 (mod 7) e 10 5 (mod 3).
Proposi
c
ao 0.38. Para quaisquer a, b, c, d, n Z temos:
1. (Reflexividade) a a (mod n);

2. (Simetria) se a b (mod n), ent


ao b a (mod n);
3. (Transitividade) se a b (mod n) e b c (mod n), ent
ao a c
(mod n);
4. (Compatibilidade com a soma e diferenca) Podemos somar e subtrair membro a membro:
(
(
a b (mod n)
a + c b + d (mod n)
=
c d (mod n)
a c b d (mod n)
Em particular, se a b (mod n), ent
ao ka kb (mod n) para
todo k Z.

CAPITULO 0. FUNDAMENTOS

32

5. (Compatibilidade com o produto) Podemos multiplicar membro a


membro:
(
a b (mod n)
= ac bd (mod n)
c d (mod n)
Em particular, se a b (mod n), ent
ao ak bk (mod n) para
todo k N.
6. (Cancelamento) Se mdc(c, n) = 1, ent
ao
ac bc

(mod n) a b (mod n).

o: Para o item (1) basta observar que n | a a = 0.


Demonstra
ca
Em (2), se n | a b, entao n | (a b) n | b a. Em (3), se
n | a b e n | b c, entao n | (a b) + (b c) n | a c. Em
(4) e (5), se n | a b e n | c d, entao n | (a b) + (c d) n |
(a + c) (b + d), n | (a b) (c d) n | (a c) (b d) e
n | (a b)c + (c d)b n | ac bd. Finalmente, como mdc(c, n) = 1
temos que n | acbc n | (ab)c n | ab pela proposicao 0.22.
As propriedades acima mostram que a relacao (mod n) (ser congruente m
odulo n) tem um comportamento muito similar `a relacao de
igualdade usual. Sao estas propriedades que tornam as congruencias tao
u
teis em problemas de divisibilidade. Vejamos alguns exemplos.
Exemplo 0.39. Demonstrar que 31 | 2015 1.
o: Isto e equivalente a demonstrar que 2015 1 (mod 31).
Solu
ca
Para isso observemos que
20 11 (mod 31)
e assim 202 (11)2 (mod 31)
121 3 (mod 31) temos

()

202 121 (mod 31). Como

202 3 (mod 31).

()

Multiplicando () e () membro a membro, obtemos 203 33 (mod 31)


e, como 33 2 (mod 31),
203 2 (mod 31).
Elevando a 5, temos que 2015 32 (mod 31) e como 32 1 (mod 31),
obtemos 2015 1 (mod 31), como desejado.


0.6. CONGRUENCIAS

33

Exemplo 0.40. Encontre os restos das divis


oes de
1. 31000 por 101
20

2. 53

por 13

o: Como 34 20 (mod 101), elevando ao quadrado obtemos


Solu
ca
8
3 400 (mod 101) 38 4 (mod 101). Multiplicando por 32 ,
obtemos 310 36 (mod 101). Portanto
320 1296

(mod 101) 320 17 (mod 101)

380 196

(mod 101) 380 6 (mod 101)

340 289

380 320 (6) (17)

(mod 101) 340 14 (mod 101)


(mod 101) 3100 1 (mod 101).

Assim, elevando a u
ltima congruencia a 10, obtemos 31000 1 (mod 101),
ou seja, 31000 deixa resto 1 na divisao por 101.
20
Para encontrar o resto da divisao de 53 por 13, note que como
54 1 (mod 13), os restos de 5n por 13 se repetem com perodo 4:
50
51
52
53

1
5
1
5

(mod
(mod
(mod
(mod

13)
13)
13)
13)

54
55
56
57

1
5
1
5

(mod
(mod
(mod
(mod

13)
13)
13)
13)

Por outro lado, temos 3 1 (mod 4) = 320 1 (mod 4), isto e,


20
320 deixa resto 1 na divisao por 4. Assim, 53 51 (mod 13), ou seja,
20
53 deixa resto 5 na divisao por 13.
O problema a seguir tem uma hist
oria interessante. Em um artigo
publicado em 1969, D. J. Lewis afirmava que a equacao x3 117y 3 = 5
tem no m
aximo 18 solucoes inteiras. Na verdade, ela nao possui nenhuma, como foi provado dois anos mais tarde por R. Finkelstein e
H. London, utilizando metodos de Teoria Algebrica dos N
umeros. Em
1973, F. Halter-Koch e V. S t. Udresco observaram independentemente
que existe uma prova muito mais simples deste fato, como mostra o
exemplo a seguir.
Exemplo 0.41. Mostre que a equac
ao x3 117y 3 = 5 n
ao possui soluc
oes inteiras.
o: Observe que como 117 e m
Solu
ca
ultiplo de 9, qualquer solucao
inteira deve satisfazer
x3 117y 3 5 (mod 9) x3 5 (mod 9).
Porem, x s
o pode deixar resto 0, 1, . . . , 8 na divisao por 9. Analisando
estes 9 casos, temos

CAPITULO 0. FUNDAMENTOS

34
x mod 9
x3 mod 9

0
0

1
1

2
8

3
0

4
1

5
8

6
0

7
1

8
8

Ou seja, x3 s
o pode deixar resto 0, 1 ou 8 na divisao por 9. Logo x3 5
(mod 9) e impossvel e a equacao nao possui solucoes inteiras.
Exemplo 0.42 (AusPol2002). Encontrar todas as ternas (a, b, c) de inteiros n
ao negativos tais que 2a + 2b + 1 e m
ultiplo de 2c 1.

o: O problema pede para determinar quando 2a + 2b + 1 0


Solu
ca
(mod 2c 1). Note que como 2c 1 (mod 2c 1), escrevendo a = cq1 +a
e b = cq2 + b com 0 a , b < c temos que
2a + 2b + 1 0 (mod 2c 1)

(2c )q1 2a + (2c )q2 2b + 1 0 (mod 2c 1)

2a + 2b + 1 0 (mod 2c 1)

que e o mesmo problema com a e b no lugar de a e b. Assim, basta


resolver o problema supondo 0 a, b < c. Temos alguns casos a analisar.
N
ao ha solucoes com c = 0 e para c = 1 temos que (a, b, 1) e solucao
para todos os a, b 0. Se c = 2, temos que apenas (0, 0, 2) e solucao com
0 a, b < c = 2, o que da origem `
as solucoes (2m, 2n, 2) para todos os m
e n naturais. Se c = 3, temos que apenas (1, 2, 3) e (2, 1, 3) s
ao solucoes
com 0 a, b < c = 3, o que nos fornece solucoes (1 + 3m, 2 + 3n, 3) e
(2 + 3m, 1 + 3n, 3) para todos os m e n naturais. Finalmente, para c 4,
temos que se a < c 1 ou b < c 1, entao
3 2a + 2b + 1 2c1 + 2c2 + 1 = 3 2c2 + 1 < 2c 1
e assim 2a + 2b + 1 nao pode ser m
ultiplo de 2c 1. Neste caso devemos
ter a = b = c 1 e 2c1 + 2c1 + 1 0 (mod 2c 1) 2c + 1 0
(mod 2c 1) 2 0 (mod 2c 1), o que nao ocorre pois 2c 1 15
nao pode dividir 2. Logo nao ha solucoes neste u
ltimo caso.
Resumindo, as ternas pedidas s
ao (m, n, 1), (2m, 2n, 2), (1 + 3m, 2 +
3n, 3) e (2 + 3m, 1 + 3n, 3) onde m e n s
ao naturais arbitr
arios.

0.7

Bases

A notacao usual para naturais e a chamada base 10, com algarismos


0, . . . , 9. Isto significa, por exemplo, que
196883 = 1 105 + 9 104 + 6 103 + 8 102 + 8 101 + 3 100 .
O teorema abaixo mostra como escrever qualquer natural em qualquer
base d.

35

0.7. BASES

Teorema 0.43. Seja n 0 e d > 1. Ent


ao existe uma u
nica sequencia
(os dgitos de n na base d) a0 , . . . , ak , . . . com as seguintes propriedades:
1. para todo k, 0 ak < d,
2. existe m tal que se k m, ent
ao ak = 0,
P
3. n = k0 ak dk .

o: Escrevemos n = n0 = n1 d + a0 , 0 a0 < d, n1 =
Demonstra
ca
n2 d + a1 , 0 a1 < d e em geral nk = nk+1 d + ak , 0 ak < d. Nossa
primeira afirmacao e que nk = 0 para algum valor de k. De fato, se
n0 < dm , entao n1 = nd0 < dm1 e mais geralmente, por inducao,
nk < dmk ; fazendo k m temos nk < 1 donde nk = 0. Segue da que
ak = 0 para k m. A identidade do item 3 e facilmente demonstrada
por inducao.
P
P
Para a unicidade, suponha k0 ak dk = k0 bk dk . Se as sequencias ak e bk s
ao distintas existe um
P j, para o qual
P menor ndice, digamos
aj 6= bj . Podemos escrever aj + k>j ak dkj = bj + k>j bk dkj donde
aj bj (mod d), o que e uma contradicao, pois 0 < |aj bj | < d e
portanto aj bj nao pode ser um m
ultiplo de d.
Ignorando os dgitos 0s iniciais, denotamos por (an an1 a1 a0 )d o
natural cuja representacao na base d tem dgitos ak como no teorema
acima:
X
def
a k dk .
(an an1 a1 a0 )d =
0kn

Muitos dos famosos criterios de divisibilidade que aprendemos na


escola decorrem diretamente da representacao acima. Por exemplo, se
N = (an an1 a1 a0 )10 , como 10 1 (mod 9), temos que 10k 1
(mod 9), donde
X
X
ak (mod 9).
ak 10k
N=
0kn

0kn

Segue que N e a soma de seus dgitos na base 10 possuem o mesmo resto


na divisao por 9; em particular N e divisvel por 9 se, e s
o se, a soma de
seus dgitos a0 + + an e divisvel por 9.
De forma similar, para o criterio de divisibilidade por 11, observemos
que 10 1 (mod 11), logo
X
X
(1)k ak (mod 11)
ak 10k
N=
0kn

0kn

e assim um n
umero e divisvel por 11 se, e s
o se, a soma dos dgitos
em posicao par menos a soma dos dgitos em posicao mpar e divisvel

CAPITULO 0. FUNDAMENTOS

36

por 11. De igual forma, podemos encontrar criterios de divisibilidade


por 7, 13 e 37, que deixamos como exerccio para o leitor enuncia-los e
demonstra-los (utilize o fato que 103 1 (mod 7), 103 1 (mod 13)
e 103 1 (mod 37)).
Exemplo 0.44. Encontrar os u
ltimos dois algarismos em representac
ao
decimal de 3200 .
o: Como
Solu
ca
(an an1 a1 a0 )10 = 102 (an 10n2 + + a2 ) + (10 a1 + a0 )
= 100 (an . . . a2 )10 + (a1 a0 )10

temos que o n
umero formado pelos dois u
ltimos algarismos de (an a1 a0 )10
e o resto da divisao deste n
umero por 100, logo o problema se resume a
calcular 3200 m
odulo 100. Podemos utilizar o bin
omio de Newton para
simplificar as contas:
X 100
200
100
100
10100k (1)k ,
3 = 9 = (10 1) =
k
0k100


100


100

logo 3200 99 10 + 100 (mod 100) 3200 1 (mod 100) e


assim os dois u
ltimos dgitos de 3200 s
ao 01.
Exemplo 0.45. Demonstrar que, para todo n natural mpar,
sn = 22n (22n+1 1)
termina em 28 quando escrito em notac
ao decimal.
o: Vamos mostrar por inducao em n que sn termina em 28.
Solu
ca
Para n = 1 temos que s1 = 28. Suponhamos que para algum n 1
mpar sn termina em 28 e vamos mostrar que sn+2 termina em 28 ou,
equivalentemente, que 100 | sn+2 sn . Temos
sn+2 sn = 22(n+2) (22(n+2)+1 1) 22n (22n+1 1)
= 22n (16 22n+5 16 22n+1 + 1)
= 5 22n (51 22n+1 3).

Como, para n mpar,


22 1 (mod 5) = 22n 1 (mod 5)

= 22n+1 2 (mod 5),

temos que 51 22n+1 3 1 (2) 3 (mod 5) 51 22n+1 3 0


(mod 5). Assim, sn+2 sn e divisvel por 5 4 5 = 100.


0.8. O ANEL DE INTEIROS MODULO
N

0.8

37

O Anel de Inteiros M
odulo n

As semelhancas entre as relacoes de congruencia m


odulo n e igualdade nao s
ao mero fruto do acaso, ambas s
ao inst
ancias de relac
oes de
equivalencia em Z. Em geral, uma relacao sobre um conjunto X e
dita de equivalencia se ela e reflexiva (x x para todo x X), simetrica
(x y y x) e transitiva (x y e y z = x z).
Dar uma rela
F cao de equivalencia em X e o mesmo que dar uma
partic
ao X = X de X, i.e., uma colecao de subconjuntos X 6= ,
dois a dois disjuntos, cuja uni
ao e X. De fato, dada a particao acima,
podemos definir uma relacao de equivalencia declarando que x y se,
e somente se, x e y pertencem a um mesmo X . Reciprocamente, se e
uma relacao de equivalencia, dado um elemento x X podemos definir
a classe de equivalencia x de x como o conjunto de todos os elementos
equivalentes a x:
x = {y X | y x}.
Observe que ou x y = (se x 6 y) ou x = y (se x y). Assim,
as distintas classes de equivalencia x formam uma particao de X. O
conjunto {x | x X} das classes de equivalencia de e chamado de
quociente de X por e e denotado por X/. Intuitivamente, X/ e o
conjunto obtido igualando-se elementos equivalentes entre si.
Agora aplicamos esta construcao geral ao nosso caso. O quociente de
Z pela relacao (mod n) e chamado de anel de inteiros m
odulo n e e
denotado por uma das notacoes Z/(n), Z/nZ, Z/n ou `as vezes Zn . Por
exemplo, para n = 2, temos que Z/2Z possui apenas dois elementos,
0 e 1 (popularmente conhecidos como conjunto dos pares e mpares,
respectivamente).
A definicao de a como um subconjunto de Z raramente sera importante, sendo apenas uma maneira de formalizar o fato de que estamos
identificando todos os inteiros que deixam o mesmo resto na divisao por
n (como no exemplo dos pares e mpares acima). Assim, o importante
e sabermos que
a = a a a

(mod n)

a e a deixam o mesmo resto na divisao por n.

Se n > 0, a divisao euclidiana diz que todo inteiro a e congruo a um


u
nico inteiro a com 0 a < n; podemos reescrever este fato na nossa
nova linguagem como
Z/(n) = {0, 1, . . . , n 1}.
Os itens (4) e (5) da proposicao 0.38 dizem que as operacoes de soma,
diferenca e produto s
ao compatveis com a relacao de congruencia. Uma

CAPITULO 0. FUNDAMENTOS

38

formulacao mais abstrata da mesma ideia e dizer que as operacoes +,


e passam ao quociente, i.e., que podemos definir a soma, subtracao e o
produto de classes de congruencia por
a+b=a+b
ab=ab
ab=ab

respectivamente. A d
uvida `
a primeira vista seria se a escolha de a e b
nao afeta a resposta: afinal existem infinitos inteiros a e b com a = a e
b = b . Os itens (4) e (5) da proposicao s
ao exatamente o que precisamos:
eles nos dizem que nestas condicoes a b = a b e a b = a b , de
modo que as operacoes acima estao bem definidas.
Por exemplo, em Z/6Z temos as seguintes tabelas de soma e produto:
+

0
1
2
3
4
5

0
1
2
3
4
5

1
2
3
4
5
0

2
3
4
5
0
1

3
4
5
0
1
2

4
5
0
1
2
3

5
0
1
2
3
4

0
1
2
3
4
5

0
0
0
0
0
0

0
1
2
3
4
5

0
2
4
0
2
4

0
3
0
3
0
3

0
4
2
0
4
2

0
5
4
3
2
1

A proxima proposicao diz quando podemos dividir por a m


odulo
n, isto e, quando o inverso multiplicativo de a m
odulo n esta definido:
Proposi
c
ao 0.46. Sejam a, n Z, n > 0. Ent
ao existe b Z com
ab 1 (mod n) se, e somente se, mdc(a, n) = 1.
o: Temos que ab 1 (mod n) admite solucao na vaDemonstra
ca
ri
avel b se, e somente se, existem b, k Z tais que ab 1 = nk
ab nk = 1. Pelo corolario 0.21 do teorema de Bachet-Bezout, isto
ocorre se, e s
o se, mdc(a, n) = 1.
Dizemos portanto que a e invertvel m
odulo n quando mdc(a, n) = 1
e chamamos b com ab 1 (mod n) de inverso multiplicativo de a m
odulo
n. O inverso e sempre u
nico m
odulo n: se ab ab 1 (mod n) temos
b b 1 b (ab ) (ba) b 1 b b

(mod n).

Assim, b esta bem definido e, em termos de classes de congruencia, temos


que a b = 1; denotamos b por (a)1 . Note que pela demonstracao
da proposicao acima calcular (a)1 e equivalente a resolver a equacao
diofantina linear ax + ny = 1 e para isto podemos utilizar o metodo do
exemplo 0.27.


0.8. O ANEL DE INTEIROS MODULO
N

39

Definimos o grupo de unidades (Z/nZ) Z/nZ do anel de inteiros


m
odulo n como o subconjunto formado pelos elementos invertveis de
Z/nZ:
(Z/nZ) = {a Z/nZ | mdc(a, n) = 1}.
Observe que o produto de elementos de (Z/nZ) e sempre um elemento
de (Z/nZ) . Por exemplo, temos a seguinte tabela de multiplicacao em
(Z/15Z) :

1 2 4 7 8 11 13 14
1 1 2 4 7 8 11 13 14
2 2 4 8 14 1 7 11 13
4 4 8 1 13 2 14 7 11
7 7 14 13 4 11 2 1 8
8 8 1 2 11 4 13 14 7
11 11 7 14 2 13 1 8 4
13 13 11 7 1 14 8 4 2
14 14 13 11 8 7 4 2 1

Uma aplicacao do inverso multiplicativo e o famoso teorema de Wilson. Primeiramente precisamos de um lema.
Lema 0.47. Se p e primo, ent
ao as u
nicas soluc
oes de x2 = 1 em Z/(p)

s
ao 1 e 1. Em particular, se x (Z/(p)) {1, 1}, ent
ao x1 6= x
em Z/(p).
o: Temos
Demonstra
ca
x2 1 (mod p) p | (x2 1) p | (x 1)(x + 1)
p | x 1 ou p | x + 1

x 1 (mod p) ou x 1 (mod p)

donde o resultado segue.


Teorema 0.48 (Wilson). Seja n > 1. Ent
ao n | (n 1)! + 1 se, e s
o se,
n e primo. Mais precisamente,
(
1 (mod n) se n e primo
(n 1)!
0 (mod n) se n e composto e n 6= 4.
o: Se n e composto mas nao e o quadrado de um primo
Demonstra
ca
podemos escrever n = ab com 1 < a < b < n. Neste caso tanto a quanto
b s
ao fatores de (n 1)! e portanto (n 1)! 0 (mod n). Se n = p2 ,
p > 2, entao p e 2p s
ao fatores de (n 1)! e novamente (n 1)! 0
(mod n); isto demonstra que para todo n 6= 4 composto temos (n1)!
0 (mod n).

CAPITULO 0. FUNDAMENTOS

40

Se n e primo podemos escrever (n 1)! 2 3 . . . (n 2) (mod n);


mas pelo lema anterior podemos juntar os inversos aos pares no produto
do lado direito, donde (n 1)! 1 (mod n).
Vejamos uma aplicacao do teorema de Wilson.
Teorema 0.49 (Teorema de Wolstenholme). Seja p > 3 um n
umero
primo. Ent
ao o numerador do n
umero
1 1
1
+ + +
2 3
p1

1+
e divisvel por p2 .

o: Note que somando os extremos temos


Demonstra
ca
X

1ip1

1
=
i

X 1
1 
=p
+
i
pi
p1

1i

1i p1
2

1
.
i(p i)

Como o mmc dos n


umeros de 1 a p1 nao e divisvel por p, basta mostrar
que o numerador da u
ltima soma e m
ultiplo de p. Equivalentemente,
como p (p 1)!, devemos mostrar que o inteiro
def

S =

1i p1
2

(p 1)!
i(p i)

e um m
ultiplo de p. Para 1 i p 1, denote por ri o inverso de
i mod p, ou seja, iri 1 (mod p). Note que rpi ri (mod p), assim
S

1i p1
2

1i p1
2

(p 1)!
iri (p i)rpi
i(p i)
(p 1)!ri rpi

ri2

(mod p)

1i p1
2

pelo teorema de Wilson. Note que como cada ri e congruente a um


2 ao congruentes a um
dos n
umeros 1, 2, . . . , p1
2 , temos que os ri s
2
dos n
umeros 12 , 22 , . . . , ( p1
odulo p. Vamos mostrar que todos eles
2 ) m
2
2
aparecem. De fato, se ri rj (mod p), entao p | (ri rj )(ri + rj ), isto
e, ri rj (mod p). Multiplicando por ij, temos que j i (mod p),
o implica i = j pois 1 i, j p1
2 .
P
P
2
2
Assim, S 1i p1 i (mod p) e como 1i p1 i2 = p(p241) e
2
2
um m
ultiplo de p (pois mdc(p, 24) = 1), o resultado segue.


0.8. O ANEL DE INTEIROS MODULO
N

41

O teorema de Wilson produz ainda resultados interessantes sobre os


coeficientes binomiais. Suponhamos que k e h s
ao inteiros positivos tais
que k + h = p 1 onde p e primo. Entao
h!k! (1)h (p 1)(p 2) (p h)k! = (1)k (p 1)!
(1)k+1

(mod p).

Portanto


p1
(p 1)! (mod p)
h!k!
k


k+1 p 1
1 (mod p)
(1)
k


p1
(1)k (mod p).

k
Exemplo 0.50. Demonstrar que se p > 3 e primo, ent
a o p3 |

2p
p

2.

o: Primeiramente, vamos relembrar algumas identidades com


Solu
ca
coeficientes
bem conhecidas. Para todo 1 i p 1, temos

 p binomiais
p1
p
que i = i i1 (basta utilizar a definicao) enquanto que
 2
   2  2
p
p
p
2p
+ +
+
=
p
1
0
p

pois podemos escolher p objetos dentre 2p escolhendo i objetos dentre


os p primeiros e p i dos p u
ltimos para todo i entre 0 e p, logo
 
X p p 
X p2
2p
=
=
.
p
i
pi
i
0ip

0ip

Utilizando estas identidades, temos que


 
2p
2=
p
p
i

1ip1

p!
i!(pi)!



p2 p 1 2
= p2
i2 i 1

1ip1



1 p1 2
.
i2 i 1

e um m
ultiplo de p para 1 i p 1 pois
2
=
o denominador desta fracao nao e divisvel por p. Assim, i12 p1
i1


P
1 p1 2
1 p 2
e inteiro e portanto a soma 1ip1 i2 i1 e inteira e devemos
p2 i
mostrar que ela e um m
ultiplo de p. Para isto observemos que cada
1 i p1 e invertvel m
odulo p; seja ri tal que 1 ri p1 e iri 1
(mod p). Pela unicidade de ri m
odulo p, temos que os ri s formam uma
Note que

CAPITULO 0. FUNDAMENTOS

42


i1 (mod p),
permutacao de 1, 2, . . . , p 1. Assim, como p1
i1 (1)
temos
X 1 p 12
X (iri )2 p 12

(mod p)
i1
i2 i 1
i2
1ip1
1ip1
X 1 p 12
X
X
2

r
=
i2 (mod p).

i
i2 i 1
1ip1

P
Como 1ip1 i2 =
1), a prova acaba.

1ip1

p(p1)(2p1)
6

1ip1

e um m
ultiplo de p (pois mdc(p, 6) =

Os termos grupo e anel empregados nesta secao estao em conformi


dade com o jarg
ao usualmente utilizado em Algebra.
Grupo e o nome
emprestado a um conjunto G juntamente com uma operacao bin
aria
(produto) que satisfaz os seguintes tres axiomas:
1. (Associatividade) Para quaisquer a, b, c G, (a b) c = a (b c).
2. (Existencia de elemento neutro) Existe um elemento e G tal que,
para todo a G, a e = e a = a.
3. (Existencia de inverso) Para qualquer elemento a G existe um
elemento a1 G tal que a a1 = a1 a = e.
Se, alem dos tres axiomas acima, o grupo G satisfaz
4. (Comutatividade) Para quaisquer a, b G, a b = b a.
entao G e chamado de grupo abeliano.
Um anel e um conjunto A com duas operacoes bin
arias + (soma) e
(produto) satisfazendo axiomas que abstraem as propriedades usuais
dos inteiros (por exemplo). Estes axiomas s
ao
1. (A, +) e um grupo abeliano com elemento neutro 0.
2. (Associatividade do produto) (ab)c = a(bc) para todo a, b, c A.
3. (Elemento neutro do produto) Existe um elemento 1 A tal que
1 a = a 1 = a para todo a A.
4. (Distributividade) a (b + c) = a b + a c e (b + c) a = b a + c a
para todo a, b, c A.
Se a b = b a para todo a, b A, dizemos que o anel A e comutativo.
Um anel comutativo A 6= 0 (isto e, 0 6= 1 em A) e chamado de domnio
se, para a, b A, a b = 0 = a = 0 ou b = 0. Por outro lado, se um
anel comutativo A 6= 0 e tal que todo elemento nao nulo possui inverso
multiplicativo (ou seja, (A\{0}, ) e um grupo) entao dizemos que o anel
A e um corpo. Um importante resultado e a seguinte

DE EULER E O TEOREMA DE EULER-FERMAT43


0.9. A FUNC
AO
Proposi
c
ao 0.51. O anel Z/nZ e um corpo se, e s
o se, n e primo.
o: Temos que Z/nZ e um corpo se, e somente se, todo
Demonstra
ca
elemento a 6= 0 e invertvel, ou seja, se e somente se, mdc(a, n) = 1 para
todo a com 0 < a < n. Mas isto e equivalente a n ser primo, pois se n e
composto e a | n com 1 < a < n, entao mdc(a, n) = a 6= 1.
Um fato curioso e muito u
til quando trabalhamos no corpo Z/pZ (p
primo) e a seguinte
Proposi
c
ao 0.52 (Sonho de todo estudante). Seja p um primo. Ent
ao
em Z/pZ temos
p
(a + b)p = ap + b
para quaisquer a, b Z/pZ.
o: Devemos mostrar que (a + b)p ap + bp (mod p)
Demonstra
ca
para todo a, b Z. Temos que se 0 < k < p
 
p
p!
0 (mod p)
=
k!(p k)!
k
pois ha um fator p no numerador que nao pode ser cancelado com nada
que apareca no denominador. Assim, utilizando o bin
omio de Newton,
temos
X p
p
apk bk ap + bp (mod p)
(a + b) =
k
0kp

como queramos mostrar.

0.9

A Func
ao de Euler e o Teorema de EulerFermat

Dizemos que um conjunto de n n


umeros inteiros a1 , . . . , an forma um
sistema completo de restos m
odulo n (scr) se
{a1 , a2 , . . . , an } = Z/(n),
isto e, se os ai representam todas as classes de congruencia m
odulo n. Por
exemplo, 0, 1, 2, . . . , n 1 formam um scr m
odulo n. Equivalentemente,
podemos dizer que a1 , a2 , . . . , an formam um scr m
odulo n se, e somente
se, ai aj (mod n) implicar i = j.
De igual forma, dizemos que os n
umeros inteiros b1 , b2 , . . . , b(n) formam um sistema completo de invertveis m
odulo n (sci) se
{b1 , b2 , . . . , b(n) } = (Z/(n)) ,

CAPITULO 0. FUNDAMENTOS

44

onde (n) representa o n


umero de elementos de (Z/(n)) . Em outras
palavras, b1 , b2 , . . . , b(n) formam um sci m
odulo n se, e somente se,
representam todas as classes de congruencia invertveis m
odulo n ou,
equivalentemente, mdc(bi , n) = 1 para todo i e bi bj (mod n) implica
i = j. O conjunto {k Z | 1 k n e mdc(n, k) = 1} e um exemplo
de sci m
odulo n.
Defini
c
ao 0.53. A func
ao
def

(n) = |(Z/nZ) |
e chamada de funcao phi de Euler.
Temos (1) = (2) = 1 e, para n > 2, 1 < (n) < n. Se p e primo,
(p) = p 1; mais geralmente (pk ) = pk pk1 pois mdc(a, pk ) = 1 se,
e somente se, a nao e m
ultiplo de p e ha pk1 m
ultiplos de p no intervalo
k
1 a p . Para calcular a funcao no caso geral, vamos mostrar que
se mdc(n, m) = 1, entao (nm) = (n)(m). Consideremos os n
umeros
1, 2, . . . , nm, onde mdc(n, m) = 1 e os arrumamos em forma matricial
assim:
1
n+1
..
.

2
n+2
..
.

3
n+3
..
.

...
...
..
.

n
2n
..
.

n(m 1) + 1 n(m 1) + 2 n(m 1) + 3 . . . n(m 1) + n


Note que, como mdc(ni + j, n) = mdc(j, n), se um n
umero nesta tabela
e primo relativo com n, entao todos os n
umeros nessa coluna s
ao primos
relativos com n. Logo existem (n) colunas nas quais todos os n
umeros
s
ao primos relativos com n. Por outro lado, toda coluna possui um
conjunto completo de restos m
odulo m: se duas entradas s
ao tais que
ni1 + j ni2 + j (mod m), entao i1 i2 (mod m) pois n e invertvel
m
odulo m ja que mdc(m, n) = 1, logo como 0 i1 , i2 < m devemos
ter i1 = i2 . Desta forma, em cada coluna existem exatamente (m)
n
umeros que s
ao primos relativos com m e portanto o total de n
umeros
nesta tabela que s
ao simultaneamente primos relativos com m e n (i.e.
primos com nm) e (nm) = (n)(m).
Assim, se n = p1 1 pk k e a fatoracao de n em potencias de primos
distintos pi , temos que

Y
Y
Y 
1
1
(n) =
(pi i ) =
(pi i pi i 1 ) = n
.
pi
1ik

1ik

1ik

Agora estamos prontos para enunciar e provar o importante

DE EULER E O TEOREMA DE EULER-FERMAT45


0.9. A FUNC
AO
Teorema 0.54 (Euler-Fermat). Sejam a e m > 0 s
ao dois inteiros com
mdc(a, m) = 1, ent
ao
a(m) 1 (mod m).
o: Observemos que se r1 , r2 , . . . , r(m) e um sistema
Demonstra
ca
completo de invertveis m
odulo m e a e um n
umero natural tal que
mdc(a, m) = 1, entao ar1 , ar2 , . . . , ar(m) tambem e um sistema completo de invertveis m
odulo m. De fato, temos que mdc(ari , m) = 1 para
todo i e se ari arj (mod m), entao ri rj (mod m) pois a e invertvel
m
odulo m, logo ri = rj e portanto i = j. Consequentemente cada ari
deve ser congruente com algum rj e, portanto,
Y

1i(m)

(m)

(ari )

1i(m)

ri

ri

(mod m)

ri

(mod m).

1i(m)

1i(m)

Mas como cada ri e invertvel m


odulo m, simplificando o fator
obtemos o resultado desejado.

1i(m) ri ,

Como caso particular do teorema anterior obtemos o


Teorema 0.55 (Pequeno Teorema de Fermat). Seja a um inteiro positivo e p um primo, ent
ao
ap a

(mod p)

o: De fato, observemos que se p | a o resultado e eviDemonstra


ca
dente. Entao, podemos supor que mdc(a, p) = 1. Como (p) = p 1,
pelo teorema de Euler temos ap1 1 (mod p), logo multiplicando por
a obtemos o resultado desejado.

Observa
c
ao 0.56. O teorema de Euler-Fermat tambem pode ser provado utilizando-se o seguinte corol
ario do teorema de Lagrange em Teoria
dos Grupos: se G e um grupo finito e g G, ent
ao g |G| = e (identidade).

Aplicando este resultado para G = (Z/mZ) , temos que a(m) = 1 para


todo a (Z/mZ) , que e uma formulac
ao equivalente para o teorema
de Euler-Fermat.
Observemos que o teorema de Euler-Fermat pode ser otimizado da
seguinte forma:

CAPITULO 0. FUNDAMENTOS

46

Proposi
c
ao 0.57. Sejam a e n n
umeros inteiros tais que mdc(a, n) = 1
e n se fatora como n = p1 1 p2 2 . . . pk k em potencias de primos distintos.
Ent
ao
aM 1 (mod n)

onde

M = mmc((p1 1 ), (p2 2 ), . . . , (pk k )).


j

o: Pelo teorema de Euler-Fermat sabemos que a(pj )


Demonstra
ca
j

1 (mod pj ) para todo j = 1, . . . k. Elevando a M/(pj j ), obtemos

aM 1 (mod pj j ). Assim, aM 1 e m
ultiplo de pj j para todo j
e como estes n
umeros s
ao dois a dois primos entre si conclumos que
n | aM 1 aM 1 (mod n), como desejado.
Vejamos agora algumas aplicacoes do teorema de Euler-Fermat.
Exemplo 0.58. Mostre que existem infinitos n
umeros da forma
20000 . . . 009
que s
ao m
ultiplos de 2009.
o:
Demonstra
ca
turais k tais que

O problema e equivalente a encontrar infinitos na-

2 10k + 9 0 (mod 2009) 2 10k + 9 2009 (mod 2009)


10k3 1 (mod 2009)

pois 2000 e invertvel m


odulo 2009. Como mdc(10, 2009) = 1, pelo
teorema de Euler-Fermat temos que 10(2009) 1 (mod 2009) =
10(2009)t 1 (mod 2009) para todo t N, logo basta tomar k =
(2009)t + 3.
Exemplo 0.59. Encontre um n
umero n N tal que 2n > 102000 e
n
2 tenha entre suas 2000 u
ltimas casas decimais pelo menos 1000 zeros
consecutivos.
2000

o: Sabemos que 2(5 ) 1 (mod 52000 ) pelo teorema de


Solu
ca
Euler-Fermat. Portanto existe b N com
2000 )

2(5

2000 )

= 52000 b + 1 = 22000+(5

= 102000 b + 22000 .

2000

Portanto os 2000 u
ltimos dgitos de 22000+(5 ) coincidem com a representacao decimal de 22000 , que tem no m
aximo 667 dgitos pois 22000 <
3
667
667
(2 )
< 10 . Desta forma, ha pelo menos 2000 667 = 1333 zeros
2000
consecutivos dentre as 2000 u
ltimas casas decimais de 22000+(5 ) e
assim n = (52000 ) + 2000 = 4 51999 + 2000 satisfaz as condicoes do
enunciado.

DE EULER E O TEOREMA DE EULER-FERMAT47


0.9. A FUNC
AO
Exemplo 0.60. Mostre que n
ao existe inteiro x tal que 103 | x3 2.
o: Note primeiramente que 103 e primo. Agora suponha que
Solu
ca
3
x 2 (mod 103), de modo que 103 x. Elevando ambos os lados
desta congruencia a (103 1)/3 = 34, obtemos x102 234 (mod 103) e
sabemos pelo teorema de Euler-Fermat que x102 1 (mod 103). Porem,
fazendo as contas, obtemos que 234 46 (mod 103), uma contradicao.
Logo nao ha inteiro x tal que 103 | x3 2.
Utilizando o mesmo raciocnio do exemplo anterior, temos que se p e
um primo tal que p 1 (mod 3) e p a, entao uma condicao necessaria
para que x3 a (mod p) tenha solucao em x e que a(p1)/3 1 (mod p).
Esta condicao tambem e suficiente, pela existencia de razes primitivas
m
odulo p, como mostraremos no final deste captulo.
Exemplo 0.61. Demonstrar que se p > 2 e primo, ent
ao
1p1 + 2p1 + 3p1 + + (p 1)p1 p + (p 1)!

(mod p2 ).

o: Pelo pequeno teorema de Fermat, sabemos que ip1 1


Solu
ca
(mod p) para todo 1 i p 1, isto e, que ip1 = ki p + 1 onde ki e um
inteiro. Assim, 1p1 +2p1 + +(p1)p1 = (k1 +k2 + +kp1 )p+p1
e portanto devemos mostrar que (k1 + k2 + + kp1 )p (p 1)! + 1
(mod p2 ).
Multiplicando as equacoes ip1 = ki p + 1, temos
(k1 p+1)(k2 p+1) (kp1 p+1) = 1p1 2p1 (p1)p1 = ((p1)!)p1 .
Por um lado, (k1 p+1)(k2 p+1) (kp1 p+1) (k1 +k2 + +kp1 )p+1
(mod p2 ). Por outro, pelo teorema de Wilson sabemos que (p 1)! 1
(mod p), ou seja, (p 1)! = Kp 1 para algum K inteiro. Segue que
(k1 + k2 + + kp1 )p + 1 (Kp 1)p1 (mod p2 )


p1
Kp (mod p2 )
= (k1 + k2 + + kp1 )p + 1 1
1

= (k1 + k2 + + kp1 )p Kp

(mod p2 )

= (k1 + k2 + + kp1 )p (p 1)! + 1

(mod p2 )

o que encerra a prova.


Conclumos esta secao apresentando brevemente uma aplicacao do
Teorema de Euler que tem particular interesse pratico: a Criptografia
RSA. Trata-se de um metodo de criptografia com chave p
ublica, isto
e, um metodo que permite a qualquer pessoa transmitir mensagens por

48

CAPITULO 0. FUNDAMENTOS

uma via insegura (ou seja, que pode ser monitorada por espi
oes) de
modo que, na pratica, apenas o legtimo destinatario, que conhece uma
chave, pode recuperar a mensagem original. A sigla vem dos nomes de
Ron Rivest, Adi Shamir, e Leonard Adleman, que desenvolveram esse
metodo.
Para isso, o receptor publica um inteiro N que e o produto de dois
primos razoavelmente grandes p e q (aproximadamente da mesma ordem
de grandeza); N e p
ublico mas a sua fatoracao pq s
o e conhecida pelo
receptor. O receptor tambem publica um expoente s (em geral nao muito
grande) com mdc(s, (p 1)(q 1)) = 1. O receptor calcula (usando o
algoritmo de Euclides) o inverso de s mod (p 1)(q 1) = (N ), isto e,
um natural r < (p 1)(q 1) com rs 1 (mod (p 1)(q 1)) (donde
rs = 1+k(N ), para algum natural k); esse r e chamado a chave privada
da criptografia. Note que apesar de N e s serem p
ublicos, nao parece ser
facil calcular (N ) ou r (neste contexto, calcular (N ) = (p 1)(q 1)
dado N = pq e equivalente a fatorar N , i.e., a encontrar os fatores primos
p e q).
Uma mensagem e um n
umero natural m < N . O emissor envia (ou
s
publica) m
:= m (mod N ), com 0 < m
< N . O receptor recupera m
via
mm
r (mod N ).
Para verificar essa equivalencia, podemos observar que
m
r (ms )r = mrs = m1+k(p1)(q1) = m (mp1 )k(q1) m (mod p);
note que, se p | m, os dois lados s
ao 0 mod p, e, caso contrario, mp1 1
r
(mod p); analogamente m
m (mod q), donde m
r m (mod N ). Essas tarefas s
ao relativamente r
apidas computacionalmente. Mais precisamente, veremos a seguir que existem algoritmos polinomiais para testar
primalidade, assim como para as demais operacoes necess
arias (veja o
captulo 7, especialmente a secao sobre o teste de Agrawal, Kayal e Saxena que garante que testar primalidade de um n
umero da ordem de N
leva tempo no m
aximo polinomial em log N ).
Se existem algoritmos polinomiais para testar primalidade, nao e
verdade que sejam conhecidos algoritmos polinomiais (e determinsticos) para obter primos novos de uma determinada ordem de grandeza.
Pelo teorema dos n
umeros primos (captulo 5 e apendice A), para todo
N grande, a probabilidade de um n
umero escolhido ao acaso entre N
e 2N ser primo e (1 + o(1))/ log N , o que implica que, se testarmos
C log N n
umeros ao acaso entre N e 2N , a probabilidade de algum deles
ser primo e da ordem de 1 exp(C(1 + o(1))), que esta muito perto de
1 para C grande. Se ao inves de sortear n
umeros procurarmos o menor
primo maior ou igual a N (testando um por um) entao, novamente pelo

DE EULER E O TEOREMA DE EULER-FERMAT49


0.9. A FUNC
AO
teorema dos n
umeros primos, em media o n
umero de tentativas sera da
ordem de log(n). Entretanto, ha gaps bem maiores do que log N e sabese muito pouco sobre o tamanho dos gaps (para um primo p, o gap g(p)
e igual a q p onde q e o menor primo maior do que p). Por exemplo,
Harald Cramer conjectura que g(p) < C(log(p))2 (para algum C > 0;
[5]): se isto for verdade entao o algoritmo proposto acima e realmente
polinomial. Pode ser que outra estrategia permita encontrar primos sem
demonstrar esta conjectura, mas nada de tempo polinomial e conhecido.
H
a um projeto Polymath sobre este assunto: veja o preprint [10] e as
paginas indicadas juntamente nas referencias. Ainda assim, podemos
considerar que o problema de obter primos e razoavelmente facil e rapido para aplicacoes praticas pois a devemos permitir algoritmos que
dependem de sorteios e que obtem o que e pedido em tempo polinomial
com probabilidade quase igual a 1. No interessante artigo de divulgacao
[12] e discutido o problema de gerar primos grandes, e em particular e
apresentado um algoritmo que funciona em muitos casos e gera primos
grandes cuja primalidade pode ser verificada por criterios bem mais simples que o teste de Agrawal, Kayal e Saxena, como o teste de Pocklington
(veja o captulo 7).
N
ao se conhecem algoritmos polinomiais para fatorar inteiros (grandes). A maioria dos especialistas duvida que exista tal algoritmo mas
e preciso enfatizar que a nao-existencia de um tal algoritmo nao e um
teorema. Mais do que isso, a nao-existencia de tal algoritmo implica diretamente em P 6= N P (um dos mais importantes problemas em aberto
da matem
atica) mas P 6= N P nao parece implicar a nao existencia do
algoritmo.
Existe ainda a possibilidade de que nao exista um algoritmo rapido,
mas que ainda assim exista uma m
aquina (no sentido literal) capaz de
fatorar inteiros rapidamente. De fato, a mecanica quantica parece permitir a construcao de um computador qu
antico e Peter Shor encontrou
um algoritmo que permite a um computador quantico fatorar inteiros
em tempo polinomial [16]. Ate 2010 foram construdos computadores
quanticos mnimos, suficientes para fatorar o n
umero 15 pelo algoritmo
de Shor mas insuficientes para n
umeros maiores [11]. N
ao e claro se sera
possvel construir computadores quanticos maiores.
Resumindo, a criptografia RSA e eficiente e segura pois e muito mais
rapido achar primos grandes do que fatorar n
umeros grandes e ele e
bastante utilizado para encriptar mensagens transmitidas pela internet.
Para mais informacoes sobre a criptografia RSA, veja [4].
Exemplo 0.62. Sabendo que a chave p
ublica de criptografia RSA s
ao
os n
umeros N = 24797 e s = 143, determine a chave privada.
o: Vamos fatorar N = 24797 usando o metodo de Fermat que
Solu
ca

CAPITULO 0. FUNDAMENTOS

50

consiste em
umeros a e b tais que N = a2 b2 . De fato,
encontrar dois n
se x0 = 24797 + 1 = 158 temos
q
x2i 24797
xi

,
158
167
159
22
logo 24797 = (159 + 22) (159 22) = 181 137. Observemos que o
metodo de Fermat e computacionalmente efetivo quanto os dois fatores
do n
umero estao proximos. Portanto (24797) = (181) (137) =
180 136 = 24480. Segue que a chave privada e a solucao da congruencia
143x 1 (mod 24480). Esta congruencia pode ser resolvida usando o
algoritmo estendido da divisao
resto
24480
143
27
8
3
2
1

quocente

171
5
3
2
1

x
0
1
171
856
2739
6634
9073

Portanto a chave privada e 24480 9073 = 15407.

Problemas Propostos
0.51. Demonstrar que
(a) 61 | 2015 1.
(b) 13 | 270 + 370 .
0.52. Encontrar os u
ltimos 3 dgitos de 32009 em notac
ao decimal.
0.53. Verificar se 987654321 e divisvel por 9, 11, 13, 17 ou 19.
0.54. Demonstrar que todo n
umero palndromo com um n
umero par de
dgitos e divisvel por 11. O que acontece com os n
umeros palndromos
com um n
umero mpar de dgitos?
0.55. Encontrar todos os n
umeros N de tres dgitos em representac
ao
decimal, tais que N e divisvel por 11 e alem disso N/11 e igual `
a soma
dos quadrados dos dgitos de N .

DE EULER E O TEOREMA DE EULER-FERMAT51


0.9. A FUNC
AO
0.56. Mostre que o dgito das dezenas de qualquer potencia de 3 e um
n
umero par (por exemplo, o dgito das dezenas de 36 = 729 e 2).
0.57. Mostre que, para todo n 0, vale que 13 | 72n+1 + 62n+1 .
0.58. Encontre todas as soluc
oes da congruencia x2 1 (mod 30).
Conclua que existem valores de x tais que 30 n
ao divide x + 1 nem
x 1 mas divide x2 1. Generalize esse resultado.
0.59. Encontre um n
umero positivo k < 50 tal que ak 1 (mod 99)
para todo inteiro a primo relativo com 99.
0.60. Mostre que para todo inteiro a temos que a561 a (mod 561) e
a1105 a (mod 1105), mas 561 e 1105 n
ao s
ao primos, o que mostra
que o recproco do pequeno teorema de Fermat e falso.
0.61. Mostre que
a12 b12

(mod 91) mdc(a, 91) = mdc(b, 91).

0.62. (P. Sabini) Mostre que entre os n


umeros da forma
14,

144,

1444,

14444,

144 44, . . .

os u
nicos quadrados perfeitos s
ao 144 = 122 e 1444 = 382 .
0.63. Seja f : N>0 N uma func
ao definida do conjunto dos inteiros
positivos no conjunto dos n
umeros naturais tal que
(a) f (1) = 0;
(b) f (2n) = 2f (n) + 1;
(c) f (2n + 1) = 2f (n).
Utilize a representac
ao em base 2 de n para encontrar uma f
ormula n
ao
recursiva para f (n).
0.64. Mostre que todo n
umero racional positivo pode ser escrito de maneira u
nica na forma
ak
a1 a2
+
+ +
1!
2!
k!
onde:
0 a1 ,

0 a2 < 2,

0 a3 < 3,

...,

0 < ak < k.

0.65 (OBM1991). Demonstrar que existem infinitos m


ultiplos de 1991
que s
ao da forma 19999 . . . 99991.

CAPITULO 0. FUNDAMENTOS

52

possvel escolher 1983 inteiros positivos distintos,


0.66 (IMO1983). E
todos menores que 105 , tal que n
ao existam tres que sejam termos consecutivos de uma progress
ao aritmetica?
Dica: Usar base 3.
5

0.67. Seja S(n) a soma dos dgitos de n. Encontrar S(S(S(22 + 1))).


0.68 (Chi2003). Encontrar todas as ternas (d, m, n) de inteiros positivos
tais que dm + 1 divide dn + 203.
0.69. Seja p > 2 um n
umero primo. Demonstrar que
 2

p1
! (1)(p+1)/2 (mod p).
2
0.70 (AusPol1996). Mostrar que n
ao existem inteiros n
ao negativos m, n
tais que m! + 48 = 48(m + 1)n
0.71. Seja p um n
umero primo. Demonstrar que (p 1)! + 1 e uma
potencia de p se, e s
o se, p = 2, 3 ou 5.

0.72. Demonstrar que para todo n
umero primo p > 3, o n
umero np
p n
e divisvel por p3+r onde pr e a maior potencia de p que divide n.
0.73. Demonstrar que
X

1kn
mdc(n,k)=1

k=

n(n)
.
2

0.74. Demonstrar que se mdc(a, b) = 1, ent


ao todos os divisores primos
de a2 + b2 s
ao da forma 4k + 1.
Dica: Utilize o teorema de Euler-Fermat.
0.75. Demonstrar que existem infinitos primos da forma 4k + 1.
0.76. Sejam m, n inteiros positivos. Demonstrar que 4mnmn nunca
pode ser o quadrado de um n
umero inteiro.
0.77 (IMO1985). Seja d um n
umero positivo distinto de 2, 5 e 13. Demonstrar que e possvel encontrar dois n
umeros diferentes a e b que
pertencam ao conjunto {2, 5, 13, d} tais que ab 1 n
ao e um quadrado
perfeito.
0.78. Demonstrar que se p | (ap bp ), ent
ao p2 | (ap bp ).
0.79 (IMO1984). Encontre todos os pares de inteiros positivos a, b tais
que ab(a + b) n
ao e divisvel por 7, mas (a + b)7 a7 b7 e divisvel por
7
7 .
(a + b)7 a7 b7 = 7ab(a + b)(a2 + ab + b2 )2 .

0.10. EQUAC
OES
LINEARES MODULO
M

53

0.80 (OIbM2001). Demonstrar que para cada inteiro positivo n existe


ltimos
um inteiro m tal que 2m tem no mnimo 23 n 1 zeros entre seus u
n algarismos em notac
ao base 10.
0.81 (IMO2003). Seja p um n
umero primo mpar. Demonstre que existe
um primo q tal que para todo n, o n
umero np p n
ao e divisvel por q.
0.82 (IMO1979). Sejam m e n inteiros positivos tais que
m
1 1 1
1
1
= 1 + +
+
.
n
2 3 4
1318 1319
Mostrar que m e divisvel por 1979.
0.83. Seja p um n
umero primo mpar e sejam a e b inteiros n
ao divisk
n
n
k
veis por p tais que p | a b. Mostrar que p | a b p | n(a b).
0.84. Sem usar computador (mas podendo usar calculadora) e sabendo
que os fatores de n est
ao perto um do outro, use o metodo de Fermat
para determine os fatores de
(a) n = 62236177.
(b) n = 6218583803.
0.85. Fatorar (sem usar computador) 801621073 sabendo que tem tres
fatores primos, um muito pequeno e os outros dois muito pr
oximos.
0.86. Encontre os fatores de 521827 sabendo que e produto de dois primos e (521827) = 520056.
0.87. Sabendo que a chave p
ublica de criptografia RSA s
ao os n
umeros
N = 26549 e s = 101, determine a chave privada.

0.10

Equac
oes Lineares M
odulo m

Se mdc(a, m) = 1, como a e invertvel m


odulo m, a equacao
ax b (mod m),
tem solucao u
nica m
odulo m, dada por x a(m)1 b (mod m) (utilizando o teorema de Euler-Fermat para encontrar o inverso de a
Z/(m)). Assim, todas as solucoes da equacao acima s
ao da forma
(m)1
x=a
b + km onde k Z. No caso geral, se mdc(a, m) = d > 1
temos que
ax b

(mod m) = ax b

(mod d) b 0 (mod d).

CAPITULO 0. FUNDAMENTOS

54

Logo uma condicao necessaria para que a congruencia linear ax b


(mod m) tenha solucao e que d | b. Esta condicao e tambem suficiente,
ja que escrevendo a = da , b = db e m = dm , temos que
ax b (mod m) a x b

(mod m ).

Como mdc(a , m ) = 1, ha uma u


nica solucao (a )(m )1 b m
odulo m ,

isto e, ha d solucoes distintas m


odulo m, a saber x (a )(m )1 b +
km (mod m) com 0 k < d. Note ainda que como resolver ax b
(mod m) e equivalente a resolver a equacao diofantina linear ax + my =
b, poderamos tambem ter utilizado o teorema de Bachet-Bezout e o
algoritmo de Euclides para encontrar as solucoes desta congruencia linear
como no exemplo 0.27. Resumimos esta discuss
ao na seguinte
Proposi
c
ao 0.63. A congruencia linear
ax b (mod m)
admite soluc
ao se, e somente se, mdc(a, m) | b. Neste caso, h
a exatamente mdc(a, m) soluc
oes distintas m
odulo m.
Agora queremos encontrar condicoes para que um sistema de congruencias lineares tenha solucao. O seguinte teorema nos garante a existencia de tais solucoes.
Teorema 0.64 (Teorema Chines dos Restos). Se b1 , b2 , . . . , bk s
ao inteiros quaisquer e a1 , a2 , . . . , ak s
ao primos relativos dois a dois, o sistema
de equac
oes
x b1

(mod a1 )

x b2
..
.

(mod a2 )

x bk

(mod ak )

admite soluc
ao, que e u
nica m
odulo A = a1 a2 . . . ak .
o: Daremos duas provas do teorema chines dos resDemonstra
ca
tos. Para a primeira, consideremos os n
umeros Mi = aAi . Temos que
mdc(ai , Mi ) = 1, logo existe Xi tal que Mi Xi 1 (mod ai ). Note que
se j 6= i entao Mj e m
ultiplo de ai e portanto Mj Xj 0 (mod ai ).
Assim, temos que
x 0 = M 1 X 1 b1 + M 2 X 2 b2 + + M k X k bk

0.10. EQUAC
OES
LINEARES MODULO
M

55

e solucao do sistema de equacoes, pois x0 Mi Xi bi bi (mod ai ).


Alem disso, se x1 e outra solucao, entao x0 x1 (mod ai ) ai |
x0 x1 para todo ai , e como os ai s s
ao dois a dois primos, temos que
A | x0 x1 x0 x1 (mod A), mostrando a unicidade m
odulo A.
Para a segunda prova, considere o mapa natural
f : Z/(A) Z/(a1 ) Z/(a2 ) Z/(ak )

b mod A 7 (b mod a1 , b mod a2 , . . . , b mod ak ).

Note que este mapa esta bem definido, isto e, o valor de f (b mod A)
independe da escolha do representante da classe de b mod A, pois quaisquer dois representantes diferem de um m
ultiplo de A, que tem imagem (0 mod a1 , . . . , 0 mod ak ) no produto Z/(a1 ) Z/(ak ). Observemos agora que o teorema chines dos restos e equivalente a mostrar que f e uma bijecao: o fato de f ser sobrejetor corresponde `a
existencia da solucao do sistema, enquanto que o fato de f ser injetor
corresponde `
a unicidade m
odulo A. Como o domnio e o contradomnio de f tem mesmo tamanho (ambos tem A elementos), para mostrar
que f e uma bijecao basta mostrarmos que f e injetora. Suponha que
f (b1 mod A) = f (b2 mod A), entao b1 b2 (mod ai ) para todo i, e como
na primeira demonstracao temos que isto implica b1 b2 (mod A), o
que encerra a prova.
Observa
c
ao 0.65. Como mdc(b, a1 a2 ...ak ) = 1 mdc(b, aj ) =
1, j k, a bijec
ao f definida na segunda prova do teorema anterior
satisfaz f ((Z/(A)) ) = (Z/(a1 )) (Z/(a2 )) (Z/(ak )) .
Em particular, isso nos d
a uma nova prova de que
(a1 a2 ...ak ) = (a1 )(a2 )...(ak ) sempre que mdc(ai , aj ) = 1, i 6= j.
Por exemplo, para k = 2, a1 = 3 e a2 = 5, temos a seguinte tabela,
que mostra, para cada i e j com 0 i < 3 e 0 j < 5, a u
nica solucao
x com 0 x < 3 5 = 15 tal que x i (mod 3) e x j (mod 5):
0 mod 3
1 mod 3
2 mod 3

0 mod 5
0
10
5

1 mod 5
6
1
11

2 mod 5
12
7
2

3 mod 5
3
13
8

4 mod 5
9
4
14

Vejamos algumas aplicacoes.


Exemplo 0.66. Determine as soluc
oes de x130 1 (mod 85).
o: Como x(85) = x64 1 (mod 85), temos que
Solu
ca
1 x130 = (x64 )2 x2 x2

(mod 85).

CAPITULO 0. FUNDAMENTOS

56

Assim temos que encontrar as solucoes do sistema


O que e equivalente a encontrar x tal que
x 2 (mod 5)

x2 1 4 (mod 5)
x2 1 16 (mod 17)

e x 4 (mod 17).

Assim, obtemos 4 sistemas, dependendo da escolha dos sinais:


(

x2
(1)
x4

(mod 5)
(mod 17)

x 2 (mod 5)
(3)
x 4 (mod 17)

(2)

(4)

x 2 (mod 5)
x 4 (mod 17)

x 2 (mod 5)
x 4 (mod 17)

Observe que se a e a solucao de (1) entao a e solucao de (3), e de igual


forma se a e a solucao de (2) entao a e solucao de (4). Assim, basta
solucionar (3) e (4).
Como x = 13 e solucao de (3), entao 85 13 = 72 e solucao de (1).
Como x = 47 e solucao de (4), entao 85 47 = 38 e solucao de (2).
Portanto, as solucoes da equacao original s
ao 13, 38, 47 e 72 (mod 85).

Exemplo 0.67. Um inteiro e livre de quadrados se ele n


ao e divisvel
pelo quadrado de nenhum n
umero inteiro maior do que 1. Demonstrar
que existem intervalos arbitrariamente grandes de inteiros consecutivos,
nenhum dos quais e livre de quadrados.
o: Seja n um n
Solu
ca
umero natural qualquer. Sejam p1 , . . . , pn primos distintos. O teorema chines dos restos nos garante que o sistema
x 1 (mod p21 )

x 2 (mod p22 )
..
.
x n

(mod p2n )

tem solucao. Se x0 e uma solucao positiva do sistema, entao cada um


dos n
umeros x0 + 1, x0 + 2, . . . , x0 + n e divisvel pelo quadrado de um
inteiro maior do que 1, logo nenhum deles e livre de quadrados.

0.10. EQUAC
OES
LINEARES MODULO
M

57

Exemplo 0.68. Seja P (x) um polin


omio n
ao constante com coeficientes
inteiros. Demonstrar que para todo inteiro n, existe um inteiro i tal que
P (i), P (i + 1), P (i + 2), . . . , P (i + n)
s
ao n
umeros compostos.
o: Demonstraremos primeiro o seguinte
Solu
ca
Lema 0.69. Seja P (x) um polin
omio n
ao constante com coeficientes
inteiros. Para todo par de inteiros k, i, tem-se que P (i) | P (k P (i) + i).
o: Dado que (kP (i) + i)n in (mod P (i)) para todo n
Demonstra
ca
inteiro nao negativo, e facil ver que P (kP (i)+i) P (i) 0 (mod P (i)).
Suponhamos por contradicao que a sequencia P (i), P (i+1), . . . , P (i+
n) contem um n
umero primo para cada i. Entao a sequencia {P (i)}i1
assume infinitos valores primos. Consideremos os n + 1 primos distintos
P (i0 ), P (i1 ), . . . , P (in ). Pelo teorema chines dos restos segue que existem
infinitas solucoes x do sistema de equacoes
x i0

(mod P (i0 ))

x i1 1 (mod P (i1 ))

x i2 2 (mod P (i2 ))
..
.
x in n

(mod P (in ))

onde, se x0 e uma solucao, entao x = x0 + k(P (i0 ) P (in )) tambem e


solucao para todo k 0. Assim, pelo lema anterior, podemos dizer que
P (x), P (x + 1), . . . , P (x + n) s
ao n
umeros compostos quando k e suficientemente grande, m
ultiplos respectivamente de P (i0 ), P (i1 ), . . . , P (in ).

Exemplo 0.70. Uma potencia nao trivial e um n


umero da forma mk ,
onde m, k s
ao inteiros maiores do que ou iguais a 2. Dado n N, prove
que existe um conjunto A P
N com n elementos tal que para todo subconjunto B A n
ao vazio,
x e uma potencia n
ao trivial. Em outras
xB

palavras, se A = {x1 , x2 , . . . , xn } ent


ao todas as somas x1 , x2 , . . . , xn ,
x1 + x2 , x1 + x3 , . . . , xn1 + xn , . . . , x1 + x2 + + xn s
ao potencias n
ao
triviais.

CAPITULO 0. FUNDAMENTOS

58

o: Vamos provar a existencia de um tal conjunto por inducao


Solu
ca
em n. Para n = 1, A = {4} e solucao e, para n = 2, A = {9, 16} e
solucao. Suponha agora que A = {x1 , . P
. . , xn } e um conjunto com n
elementos e para todo B A, B 6= ,
x = mkBB . Vamos mostrar
xB

que existe c N tal que o conjunto A = {cx1 , cx2 , . . . , cxn , c} satisfaz


o enunciado. Seja = mmc{kB | B A, B 6= }, o mnimo m
ultiplo
comum de todos os expoentes kB . Para cada B A, B 6= , associamos
um n
umero primo pB > , de forma que B1 6= B2 implica pB1 6= pB2 .
Pelo teorema chines dos restos existe um natural rB com
rB

0 (mod pX ) para todo subconjunto X A, X 6= B

rB 1 (mod pB ).

( e invertvel m
odulo pB ). Tomemos
c=

(1 + mkXX )rX

XA
X6=

e vamos mostrar que A = {cx1 , cx2 , . . . , cxn , c} continua a satisfazer as


condicoes do enunciado.
Dado B {cx1 , cx2 , . . . , cxn }, temos que B = {cx | x B} para
algum B A. Como c e uma
P potencia -esima, c tambem e uma
potencia kB -esima, portanto, xB x = cmkBB sera uma potencia kB esima para todo B 6= . Alem disso, para subconjuntos de A da forma
B {c}, temos
X

xB {c}

x = c (1 + mkBB ) =

 Y

XA
X6=,B


(1 + mkXX )rX (1 + mkBB )rB +1 ,

que e uma potencia pB -esima, pois rB + 1 e rX (X 6= B) s


ao m
ultiplos
de pB .

Problemas Propostos
0.88. Determine a menor soluc
ao inteira positiva do sistema

x 3 (mod 7)
x 5 (mod 9)

x 2 (mod 8).

0.10. EQUAC
OES
LINEARES MODULO
M

59

0.89. Determine todas as soluc


oes do sistema

x 1 (mod 6)
x 7 (mod 10)

x 4 (mod 33).

Observe que o sistema n


ao satisfaz as condic
oes do teorema Chines dos
restos.
0.90. Determine todas as soluc
oes de x2 + x + 18 0 (mod 42).
0.91. Quantos elementos tem (Z/(210)) ? Quantos deles tem ordem
24?
0.92. Resolver as equac
oes lineares
(a) 7x 12 (mod 127)
(b) 12x 5 (mod 122)
(c) 40x 64 (mod 256)
0.93. Resolver o sistema de congruencias lineares
x

0 (mod 7)
1 (mod 12)

x 5 (mod 17)
0.94. Um inteiro positivo n e chamado de auto-replicante se os u
ltimos
dgitos de n2 formam o n
umero n. Por exemplo, 25 e auto-replicante
pois 252 = 625. Determine todos os n
umeros auto-replicantes com exatamente 4 dgitos.
0.95. Sejam a, n N>0 e considere a sequencia (xk ) definida por x1 = a,
xk+1 = axk para todo k N. Demonstrar que existe N N tal que
xk+1 xk (mod n) para todo k N .
0.96. Demonstrar que o sistema de equac
oes
x b1

(mod a1 )

x b2
..
.

(mod a2 )

x bk

(mod ak )

tem soluc
ao se, e s
o se, para todo i e j, mdc(ai , aj ) | (bi bj ). (No
caso particular em que mdc(ai , aj ) = 1, o problema se reduz ao teorema
chines dos restos).

60

CAPITULO 0. FUNDAMENTOS

0.97. Demonstrar que, para k e n n


umeros naturais, e possvel encontrar
k n
umeros consecutivos, cada um dos quais tem ao menos n divisores
primos diferentes.
0.98. Demonstrar que se a, b e c s
ao tres inteiros diferentes, ent
ao
existem infinitos valores de n para os quais a + n, b + n e c + n s
ao
primos relativos dois a dois.
0.99. Demonstrar que para todo inteiro positivo m e todo n
umero par 2k,
este u
ltimo pode ser escrito como a diferenca de dois inteiros positivos,
cada um dos quais e primo relativo com m.
0.100. Demonstrar que existem progress
oes aritmeticas de comprimento
arbitr
ario formadas por inteiros positivos tais que cada termo e a potencia de um inteiro positivo com expoente maior do que 1.

Captulo 1

Pot
encias e Congru
encias
Neste captulo continuamos a discutir congruencias, com especial
interesse por potencias de um inteiro a m
odulo p, p primo.
Inicialmente faremos uma revisao de alguns fatos basicos sobre o
anel dos polin
omios com coeficientes sobre um corpo qualquer K; o leitor
reconhecera que varios resultados sobre polin
omios com coeficientes reais
ainda valem quando os coeficientes estao, digamos, em Z/(p).
Depois disso discutiremos a existencia de razes primitivas m
odulo
n.
Finalmente, discutiremos se um inteiro a e ou nao um quadrado
m
odulo p. Para isso apresentaremos o smbolo de Legendre e suas principais propriedades. O captulo conclui com o Teorema da Reciprocidade
Quadratica, para o qual daremos tres demonstracoes explorando ideias
diferentes.

1.1

Polin
omios

Dado um anel comutativo K, definimos o anel comutativo K[x] como


sendo o conjunto das expressoes da forma f (x) = a0 + a1 x + a2 x2 + +
an xn com ai K, chamados de polin
omios com coeficientes em K.
A somaPe o produto emP
K[x] s
ao definidos da maneira usual: dados
f (x) = i ai xi e g(x) = i bi xi elementos de K[x] temos
def

f (x) + g(x) =

(ai + bi )xi ;

def

f (x) g(x) =

ck xk onde ck =

ai bj .

i+j=k

Definimos o grau deg f (x) de um polin


omio f (x) = a0 + a1 x + a2 x2 +
n
+ an x como sendo o maior i tal que ai 6= 0; o grau do polin
omio
61

62

CAPITULO 1. POTENCIAS
E CONGRUENCIAS

nulo 0 e definido como sendo . Tal convencao visa a tornar validas


as seguintes identidades para todos os polin
omios f (x), g(x) K[x]:

deg f (x) g(x) = deg f (x) + deg g(x)
e

deg f (x) + g(x) max{deg f (x), deg g(x)}.

O coeficiente do termo de maior grau de um polin


omio e chamado de coeficiente lder. Um polin
omio cujo coeficiente lder e igual a 1 e chamado
de m
onico.
Observe que nas definicoes acima x e um smbolo formal e nao um
elemento de K. Apesar disso, cada polin
omio f (x) = a0 + a1 x + a2 x2 +
+ an xn define uma func
ao polinomial
f: K K

c 7 f (c) = a0 + a1 c + a2 c2 + + an cn

tambem chamada de f . A distincao entre um polin


omio e uma funcao
polinomial e bem ilustrada pelo polin
omio f (x) = xp x (Z/(p))[x]:
este polin
omio e nao nulo pois seus coeficientes s
ao nao nulos, mas para
todo c Z/(p) temos f (c) = 0 pelo pequeno teorema de Fermat. Dado
um polin
omio f (x) K[x], qualquer c K tal que f (c) = 0 e chamado
de raiz ou zero de f (x).
Como veremos nesta secao, polin
omios guardam muitas semelhancas
com n
umeros inteiros. Por exemplo, podemos definir divisibilidade de
polin
omios de maneira completamente an
aloga: d(x) | f (x) em K[x] se,
e s
o se, existe g(x) K[x] tal que f (x) = d(x) g(x). Temos tambem
uma generalizacao da divisao euclidiana:
Proposi
c
ao 1.1 (Algoritmo da divisao). Seja K um corpo. Dados polin
omios f (x), g(x) K[x], com g(x) 6= 0, existem q(x), r(x) K[x]
(chamados respectivamente de quociente e resto da divis
ao de f (x) por
g(x)), unicamente determinados, tais que
f (x) = q(x) g(x) + r(x)

com

deg r(x) < deg g(x).

o: Sejam n = deg f (x) e m = deg g(x). Para demonsDemonstra


ca
trar a existencia de q(x) e r(x), procederemos por inducao sobre n. Note
que se m > n, entao basta tomar q(x) = 0 e r(x) = f (x), logo podemos
supor que m n. Se n = m = 0, entao f (x) = a e g(x) = b s
ao ambos
constantes nao nulas, logo basta tomar q(x) = a/b e r(x) = 0 neste caso.
Agora suponha que n 1. Escreva f (x) = an xn + f1 (x) e g(x) =
bm xm + g1 (x) com an 6= 0, bm 6= 0 e deg f1 (x) < n, deg g1 (x) < m.
Observemos que o polin
omio f (x) bamn xnm g(x) = f1 (x) bamn xnm g1 (x)


1.1. POLINOMIOS

63

e de grau menor que n. Por hip


otese de inducao existem dois polin
omios
q(x) e r(x) tais que
f (x)

an nm
x
g(x) = q(x)g(x) + r(x)
bm

com

deg r(x) < deg g(x).

Logo podemos escrever f (x) = ( bamn xnm + q(x)) g(x) + r(x), que era o
que se queria demonstrar.
Para demonstrar que os polin
omios q(x) e r(x) s
ao u
nicos, suponha
que
f (x) = q1 (x)g(x) + r1 (x) = q2 (x)g(x) + r2 (x)
com q1 (x) 6= q2 (x) e deg r1 (x), deg r2 (x) < deg g(x). Entao r2 (x)
r1 (x) = (q1 (x) q2 (x))g(x) 6= 0 e um m
ultiplo de g(x) de grau estritamente menor do que deg g(x), o que e um absurdo.

Corol
ario 1.2. Seja K um corpo, f (x) K[x] e a K. Ent
ao
x a | f (x) f (a) = 0.
o: Como deg(xa) = 1, dividindo f (x) por xa temos
Demonstra
ca
que f (x) = (x a)q(x) + r com r K. Assim, substituindo x por a,
temos que f (a) = r donde o resultado segue.

Exemplo 1.3. Seja f (x) Q[x] um polin


omio com coeficientes racionais tal que f (1) = 3 e f (3) = 5. Determine o resto da divis
ao do
polin
omio f (x) pelo polin
omio g(x) = x2 4x + 3.
o: Seja q(x) e r(x) respectivamente o quociente o resto da
Solu
ca
divisao de f (x) por g(x), isto e, f (x) = q(x)g(x) + r(x), onde r(x) e um
polin
omio de grau menor ou igual a 1, assim r(x) = ax + b. Observe que
g(x) = (x 1)(x 3), assim g(1) = g(3) = 0, e portanto
3 = f (1) = q(1)g(1) + r(1) = a + b
5 = f (3) = q(3)g(3) + r(3) = 3a + b.
Segue que a = 4 e b = 7 e portanto r(x) = 4x + 7.
Proposi
c
ao 1.4. Seja K um corpo. Um polin
omio f (x) K[x] n
ao
nulo de grau n tem no m
aximo n razes em K.

64

CAPITULO 1. POTENCIAS
E CONGRUENCIAS

o: A demonstracao e feita por inducao em n = deg f (x);


Demonstra
ca
os casos n = 0 e n = 1 s
ao triviais. Se f (x) tivesse n + 1 razes distintas a1 , . . . , an+1 , entao f (x) = (x an+1 )g(x) para algum g(x) K[x]
pelo corolario anterior. Assim, para i 6= n + 1, teramos 0 = f (ai ) =
(ai an+1 )g(ai ) = g(ai ) = 0 pois (ai an+1 ) 6= 0 e invertvel em K.
Logo g(x), de grau n 1, teria n razes distintas a1 , . . . , an , contradizendo a hip
otese de inducao.

Note que a proposicao anterior e falsa se K nao e um corpo. Por


exemplo, o polin
omio f (x) = x2 1 Z/8Z[x] tem 4 razes em Z/8Z, a
saber 1, 3, 5, 7.
A seguinte proposicao ajuda a achar as razes de um polin
omio em
Q[x].
Proposi
c
ao 1.5. Seja f (x) = an xn + + a0 Z[x] um polin
omio de
grau n. Mostre que se p/q e uma raiz racional de f (x), com p, q Z e
mdc(p, q) = 1, ent
ao p | a 0 e q | a n .
o: Multiplicando f (p/q) = 0 por q n , segue que
Demonstra
ca
an pn + an1 pn1 q + + a0 q n = 0,
donde p | a0 q n e q | an pn . Como mdc(p, q) = 1, conclumos que p | a0 e
q | an .
Exemplo 1.6. Seja f (x) Q[x] um polin
omio de grau 5 tal que f (k) =
2k para k = 0, 1, . . . , 5. Determine o valor de f (6).
o: Suponhamos que f (x) = a5 x5 + a4 x4 + + a0 , onde os
Solu
ca
ai s
ao os coeficientes do polin
omio. Assim se substitumos por 0, 1, 5
obteramos um sistema de 6 equacoes lineares com 6 incognitas, que pode
ser solucionado com algum processo de resolucao de sistemas lineares.
Dado que nosso objetivo nao e calcular o polin
omio e sim o valor de
f (6), empregaremos um metodo distinto. De fato vamos resolver um
problema um pouco mas geral. Suponhamos que f e um polin
omio de
grau n tal que f (j) = 2j para j = 0, 1, . . . n e queremos calcular f (n+1).
Definamos os polin
omios auxiliares (
Pj (x) de grau n tal que para cada
1 se i = j
i = 0, 1, . . . n se tem que Pj (i) =
. E facil verificar por
0 se i 6= j
n
P
2j Pj (x). Agora cada um dos polin
omios e
diretamente que f (x) =
j=0


1.1. POLINOMIOS

65

facilmente calcul
avel, ja que s
ao polin
omios de grau n e conhecemos n
razes; de fato, se Sj = {0, 1, . . . , n} \ {j}, entao
Q
(x i)
Y xi
iSj
=
.
Pj (x) =
ji
(1)nj j!(n j)!
iSj

Em particular,
Q

(n + 1 i)



(1)nj (n + 1)!
nj n + 1
Pj (n + 1) =
=
= (1)
.
j
(1)nj j!(n j)!
j!(n + 1 j)!
iSj

Conclumos que
f (n + 1) =

n
X

(1)

nj

j=0

n+1

= 2


n+1 j
2
j

n+1
X

(1)

n+1j

j=0



n+1 j
2
j

= 2n+1 (2 1)n+1 = 2n+1 1.


Vejamos uma aplicacao dos resultados anteriores quando K = Z/(p),
p primo. A primeira e uma nova demonstracao do teorema de Wilson:
Teorema 1.7. Seja p um primo. Considere a func
ao sim
etrica elementar i em 1, 2, . . . , p 1 dada pela soma de todos os p1
produtos de i
i
termos distintos dentre 1, 2, . . . , p 1:
1 = 1 + 2 + + (p 1)

2 = 1 2 + 1 3 + + (p 2)(p 1)
..
.

p1 = 1 2 . . . (p 1).
Ent
ao 1 , . . . , p2 s
ao todos m
ultiplos de p e p1 = (p 1)! 1
(mod p) (teorema de Wilson).
o: Pelo teorema de Fermat e pela proposicao anterior,
Demonstra
ca
ao todas as razes de xp1 1 em Z/(p). Logo
temos que 1, 2, . . . , p 1 s
aplicando o corolario e comparando coeficientes lderes obtemos a fatoracao
xp1 1 = (x 1)(x 2) . . . (x p 1).
Mas o polin
omio do lado direito e igual a xp1 1 xp2 + 2 xp3 +
p1
(1) p1 . Comparando coeficientes, obtemos o resultado.

CAPITULO 1. POTENCIAS
E CONGRUENCIAS

66

Seja K um corpo. Podemos considerar tambem congruencias de


polin
omios em K[x]: se a(x), b(x), m(x) K[x], escrevemos
a(x) b(x)

(mod m(x)) m(x) | a(x) b(x).

As mesmas demonstracoes do caso inteiro mostram que as congruencias


m
odulo m(x) definem uma relacao de equivalencia em K[x] compatvel
com as operacoes de soma, subtracao e produto. Assim, podemos formar
o anel quociente
K[x]

m(x)

cujos elementos s
ao os conjuntos da forma
def

a(x) = {b(x) K[x] | b(x) a(x)

(mod m(x))}

e as operacoes no anel quociente s


ao dadas por
def

f (x) + g(x) = f (x) + g(x)

def

f (x) g(x) = f (x) g(x)

sendo independentes das escolhas dos representantes de classe f (x) e


g(x). Se deg m(x) = n, um sistema completo de resduos m
odulo m(x)
e dado pelos polin
omios de grau menor do que n (os possveis restos na
divisao euclidiana por m(x)):
{a0 + a1 x + + an xn1 | ai K}
Em particular,

K[x]
(m(x))

e infinito se K tambem o e.

Exemplo 1.8. Determine o resto da divis


ao de (x+1)2010 por x2 +x+1
em Q[x].
o: Multiplicando por x 1 a congruencia x2 + x + 1 0
Solu
ca
(mod x2 + x + 1), obtemos x3 1 (mod x2 + x + 1). Assim, temos
(x + 1)2 x

(mod x2 + x + 1)

= (x + 1)2010 x1005 = (x3 )335

(mod x2 + x + 1)

= (x + 1)2010 1 (mod x2 + x + 1)

Assim, o resto da divisao e 1.


Podemos tentar definir o mdc d(x) de dois polin
omios f (x) e g(x)
(com f (x) 6= 0 ou g(x) 6= 0) de maneira an
aloga ao mdc de inteiros,
tomando o polin
omio d(x) de maior grau que divide f (x) e g(x) simultaneamente. Entretanto, d(x) nao esta bem determinado, pois qualquer
m
ultiplo c d(x) com c 6= 0 constante ainda satisfaz as condicoes acima.


1.1. POLINOMIOS

67

Para evitar esta ambiguidade, definimos o mdc de f (x) e g(x) como


sendo o polin
omio m
onico de maior grau que divide f (x) e g(x) simultaneamente. Analogamente, define-se o mmc de f (x) e g(x) (com f (x) 6= 0
e g(x) 6= 0) como o polin
omio m
onico de menor grau que e divisvel tanto
por f (x) como por g(x).
A divisao euclidiana permite estender resultados de Z para K[x] de
maneira quase trivial. Por exemplo, temos
Teorema 1.9 (Bachet-Bezout). Seja d(x) o m
aximo divisor comum de
dois polin
omios f (x) e g(x). Ent
ao existem dois polin
omios m(x) e n(x)
tais que f (x)m(x) + g(x)n(x) = d(x).
o: An
Demonstra
ca
aloga ao teorema 0.20; como naquele teorema
d(x) sera o polin
omio m
onico de menor grau no conjunto
def

I(f, g) = {f (x)m(x) + g(x)n(x) | m(x), n(x) K[x]}.

Exemplo 1.10. Sejam f (x) = x6 + x + 1 e g(x) = x4 + 1 polin


omios
em (Z/(2))[x]. Determine dois polin
omios m(x), n(x) (Z/(2))[x] tal
que f (x)m(x) + g(x)n(x) = mdc(f (x), g(x)).
o: Observe que em Z/(2) temos que g(x) = (x 1)4 , isto e,
Solu
ca
g(x) tem uma raiz quadrupla em x = 1. Por outro lado f (1) = 1,
logo mdc(f, g) = 1. Para calcular m(x) e n(x) basta usar o algoritmo
estendido da divisao:
f (x) = x2 g(x) + x2 + x + 1
g(x) = (x2 + x)(x2 + x + 1) + x + 1
x2 + x + 1 = x(x + 1) + 1.
Segue que
1 =
=

x2 + x + 1 + x x + 1


x2 + x + 1 + x g(x) + (x2 + x) x2 + x + 1

= (x3 + x2 + 1) x2 + x + 1 + xg(x)

= (x3 + x2 + 1)(f (x) + x2 g(x)) + xg(x)


= (x3 + x2 + 1)f (x) + (x5 + x4 + x2 + x)g(x).

68

CAPITULO 1. POTENCIAS
E CONGRUENCIAS

Defini
c
ao 1.11. Seja K um corpo. Dizemos que um polin
omio n
ao
constante f (x) K[x] e irredutvel em K[x] se f (x) n
ao e o produto de
dois polin
omios em K[x] de graus estritamente menores do que deg f (x).
Polin
omios irredutveis fazem o papel de n
umeros primos para polinomios. Por exemplo, x2 + 1 R[x] e irredutvel em R[x], pois caso
contrario ele poderia ser escrito como produto de polin
omios de grau 1
em R[x], contradizendo o fato de x2 + 1 = 0 nao possuir razes reais.
Por outro lado, x2 + 1 e redutvel em C[x] ja que x2 + 1 = (x i)(x + i).
Isto mostra que irredutibilidade e um conceito que depende do anel de
polin
omios sobre o qual estamos trabalhando.
Os exemplos mais evidentes de polin
omios irredutveis em K[x] s
ao
os lineares m
onicos, i.e., os da forma x a, a K. Quando estes
s
ao os u
nicos polin
omios irredutveis em K[x] dizemos que o corpo K e
algebricamente fechado. Observe que em geral polin
omios de graus 2 ou
3 s
ao irredutveis em K[x] se, e somente se, nao tem razes em K.
A partir do teorema de Bachet-Bezout, como no caso dos inteiros,
obtemos (c.f. proposicao 0.23 e teorema 0.30):
Proposi
c
ao 1.12. Seja K um corpo e sejam p(x), a1 (x), . . . am (x)
K[x] com p(x) irredutvel em K[x]. Se p(x) | a1 (x) . . . am (x), ent
ao
p(x) | ai (x) para algum i.

Teorema 1.13 (Fatoracao Unica).


Seja K um corpo. Todo polin
omio
n
ao nulo em K[x] pode ser fatorado como um produto de polin
omios irredutveis em K[x]; esta fatorac
ao e u
nica a menos da ordem dos fatores
e multiplicac
ao por constantes n
ao nulas.
Outra importante consequencia do teorema de Bachet-Bezout e o
seguinte (c.f. teorema 0.51)
Teorema 1.14. Seja K um corpo e f (x) um polin
omio irredutvel em
K[x]. Ent
ao K[x]/(f (x)) e um corpo.
o: Assim como na demonstracao de que Z/pZ e um
Demonstra
ca
corpo para p primo, a dificuldade aqui e mostrar que todo elemento
a(x) 6= 0 e invertvel em K[x]/(f (x)). Temos que mdc(a(x), f (x)) = 1
pois f (x) e irredutvel e f (x) nao divide a(x), caso contrario teramos
a(x) = 0. Logo, pelo teorema de Bachet-Bezout, existem r(x), s(x)
K[x] tais que
a(x)r(x) + f (x)s(x) = 1 = a(x)r(x) 1 (mod f (x))
Portanto r(x) e o inverso multiplicativo de a(x).


1.1. POLINOMIOS

69

Por exemplo, seja K = Z/(2) e f (x) = x2 + x + 1 K[x]. Temos que


f (x) e irredutvel pois ele tem grau 2 e nao possui razes em K. Assim,
K[x]/(f (x)) e um corpo, que possui 4 elementos. As tabelas de adicao
e multiplicacao deste corpo s
ao as seguintes:
+

x+1

0
1
x
x+1

0
1
x
x+1

1
0
x+1
x

x
x+1
0
1

x+1
x
1
0

x+1

0
0
0
0

0
1
x
x+1

0
x
x+1
1

0
x+1
1
x

0
1
x
x+1

Exemplo 1.15. Calcule ordG (x + 1) no grupo G = (Z/(7))[x]/(p(x))


onde p(x) = x2 3x 1, isto e, o menor inteiro positivo k tal que
(x + 1)k 1 em G.
o: Observemos que o grupo G tem 72 1 = 48 elementos (todo
Solu
ca
elemento pode ser representado como ax + b, com a, b Z/(7), e o u
nico
elemento nao invertvel e o 0, pois p(x) e irredutvel em (Z/(7))[x]).
Assim, ordG (x + 1) e um divisor de 48. Os divisores de 48 s
ao 1, 2, 3, 4,
6, 8, 12, 16, 24 e 48. Assim temos que testar todas estas potencias
(x + 1)2 x2 + 2x + 1 5x + 2 2x + 2

(x + 1)3 (2x + 2)(x + 1) 2x2 + 2 6x x


(x + 1)4 x(x + 1) x2 + x 4x + 1

(x + 1)6 (x + 1)3 (x + 1)3 x2 3x + 1

(x + 1)8 (x + 1)4 (x + 1)4 2x2 + x + 1 3

(x + 1)12 (x + 1)4 (x + 1)8 (4x + 1)(3) 2x + 3

(x + 1)16 (x + 1)82 2

(x + 1)24 (x + 1)83 1
Portanto ordG (x + 1) = 48.
Encerramos esta secao com um importante criterio de irredutibilidade para polin
omios com coeficientes inteiros. Primeiro, precisamos de
uma

70

CAPITULO 1. POTENCIAS
E CONGRUENCIAS

Defini
c
ao 1.16. Um polin
omio n
ao nulo f (x) Z[x] e dito primitivo
se o mdc de seus coeficientes e 1.
Lema 1.17. O produto de dois polin
omios primitivos e primitivo.
o: Sejam g(x) e h(x) dois polin
Demonstra
ca
omios primitivos. Seja
p um primo e suponha por absurdo que p divida todos os coeficientes de
g(x)h(x). Assim, em Z/pZ[x] teramos que g(x)h(x) = g(x)h(x) = 0,
onde a barra denota o polin
omio obtido reduzindo-se seus coeficientes
m
odulo p. Por outro lado, g(x) 6= 0 e h(x) 6= 0, ja que por hip
otese p
nao divide todos os coeficientes de g(x) e o mesmo para h(x). Assim,
temos uma contradicao pois Z/pZ[x] e um domnio, isto e, o produto de
dois polin
omios nao nulos em Z/pZ[x] e diferente de zero (de fato, olhe
por exemplo para os coeficientes lderes e use o fato de que Z/pZ e um
corpo).
O lema anterior e o passo essencial na prova do famoso lema de Gau,
que permite reduzir a quest
ao da irredutibilidade de um polin
omio em
Q[x] para a mesma quest
ao em Z[x].
Teorema 1.18 (Lema de Gau). Seja f (x) Z[x] um polin
omio primitivo n
ao constante. Ent
ao f (x) e irredutvel em Q[x] se, e somente se,
f (x) e irredutvel em Z[x] (isto e, n
ao podemos escrever f (x) = g(x)h(x)
com g(x), h(x) Z[x] n
ao constantes).
claro que se f (x) e irredutvel sobre Q[x], entao
o: E
Demonstra
ca
ele e irredutvel sobre Z[x]. Reciprocamente, suponha por contradicao que f (x) seja irredutvel sobre Z[x] mas que f (x) = g(x)h(x) com
g(x), h(x) Q[x], ambos nao constantes. Multiplicando esta u
ltima
igualdade por um inteiro conveniente d > 0, podemos escrever
d f (x) = e g0 (x)h0 (x)
com g0 (x), h0 (x) Z[x] primitivos e e N. Como f (x) e g0 (x)h0 (x)
(pelo lema anterior) s
ao primitivos, temos que d e o mdc dos coeficientes
de df (x), enquanto que e e o mdc dos coeficientes de eg0 (x)h0 (x). Logo
d = e e assim f (x) = g0 (x)h0 (x) e redutvel sobre Z[x], uma contradicao.

Finalmente, para polin


omios em Z[x], podemos aplicar o
Proposi
c
ao 1.19 (Criterio de Eisenstein). Seja f (x) = an xn + +
a1 x + a0 Z[x] um polin
omio primitivo n
ao constante. Suponha que
exista um n
umero primo p tal que p an , p | aj para todo 0 j < n e
p2 a0 . Ent
ao f (x) e irredutvel em Z[x].


1.1. POLINOMIOS

71

o: Suponha por absurdo que f (x) e redutvel, i.e., exisDemonstra


ca
tem g(x), h(x) Z[x] tais que f (x) = g(x)h(x) e 0 < deg g(x), deg h(x) <
n. Em Z/pZ[x], temos entao f (x) = g(x)h(x), onde a barra denota o
polin
omio obtido reduzindo-se os seus coeficientes m
odulo p. Porem,
como p | aj para todo 0 j < n, temos que f (x) = an xn e portanto,
pela fatoracao u
nica em Z/pZ[x] (teorema 1.13), devemos ter g(x) = bxi
j
e h(x) = cx com 0 < i, j < n, i + j = n e b c = an . Mas isto significa
que os coeficientes de x0 em g(x) e h(x) s
ao m
ultiplos de p, e como
f (x) = g(x)h(x), que a0 e m
ultiplo de p2 , absurdo.
Exemplo 1.20. Seja p um primo. Demonstrar que o polin
omio f (x) =
p1
p2
x
+x
+ + x + 1 e irredutvel em Q[x].
o: Pelo lema de Gau, basta provar a irredutibilidade sobre
Solu
ca
Z[x] e para isto utilizaremos o criterio de Eisenstein. Observemos que
p 1
, logo
f (x) = xx1


 
(x + 1)p 1
p
p p2
p1
f (x + 1) =
x
+ +
=x
+
p1
1
x
e, com excecao do coeficiente lder, todos os coeficientes deste
omio
 polin
p
s
ao m
ultiplos de p, sendo que o termo independente p1 = p nao e
m
ultiplo de p2 . Pelo criterio de Eisenstein, f (x + 1) e irredutvel em
Z[x] e, portanto, f (x) tambem o e.
Observa
c
ao 1.21. Existem polin
omios primitivos irredutveis f (x)
Z[x] mas que s
ao redutveis m
odulo p para todo primo p, por exemplo
f (x) = x4 10x2 + 1 (veja o exemplo 1.45). Por outro lado, se f (x)
Z[x] admite raiz m
odulo p para todo primo p suficientemente grande,
ent
ao f (x) possui raiz em Z! Veja o excelente artigo de Serre [14] para
uma demonstrac
ao deste fato.

Problemas Propostos
1.1. Sejam f (x) = x7 4x5 5x3 + x2 + 1 e g(x) = x5 + 2x3 + 6x2 + 5
polin
omios em Q[x]. Determine mdc(f (x), g(x)) e ache dois polin
omios
m
onicos com coeficientes inteiros m(x) e n(x) tais que
f (x)m(x) + g(x)n(x) = mdc(f (x), g(x)).
1.2. Sejam f (x) = x4 + x2 + 1 e g(x) = x3 + 1 polin
omios em (Z/(2))[x].
Determine mdc(f (x), g(x)) e ache dois polin
omios m
onicos (com coeficientes em Z/(2)) m(x) e n(x) tais que
f (x)m(x) + g(x)n(x) = mdc(f (x), g(x)).

72

CAPITULO 1. POTENCIAS
E CONGRUENCIAS

1.3. Seja f (x) C[x] um polin


omio que deixa restos 10 e 1 quando
dividido por x 1 e x 10 respectivamente. Encontrar o resto de f (x)
na divis
ao por (x 1)(x 10).
1.4. Determine o resto da divis
ao de f (x) = x100 por g(x) = x3 + 2x2
x 2.
Dica: Fatore g(x).
1.5. Determine o resto da divis
ao de f (x) = x100 por g(x) = x3 + 2x2 +
x + 2.
1.6. Mostre que o polin
omio f (x) = x4 4x3 + 6x2 x + 1 e irredutvel
em Q[x].
Dica: substituir x por x + a com a adequado e usar o criterio de
Einsentein.
1.7. Mostre que o polin
omio f (x) = x4 20x2 +16 e irredutvel em Q[x].
1.8. Fatore o polin
omio x8 x (Z/(2))[x] em fatores irredutveis.
1.9. Fatore o polin
omio x25 x (Z/(5))[x] em fatores irredutveis.
1.10. Encontre um valor de a N tal que o polin
omio f (x) = x100 +
98
ax + 11 n
ao tenha razes racionais mas n
ao seja irredutvel em Q[x]
Dica: Calcule um a tal que f (x) seja divisvel por um polin
omio
irredutvel de grau 2.
1.11. Seja uma raiz de X 3 3X + 1 = 0. Mostre que 2 2 tambem
e uma raiz deste polin
omio.
1.12. Seja K = (Z/(3))[x]/(f (x)) onde f (x) = x2 + x + 2. Mostre que
f (x) e irredutvel em (Z/(3))[x] e portanto K e um corpo. Construa a
tabela de multiplicac
ao do grupo K e usando esta tabela determine o
menor inteiro positivo n tal que xn = 1 em K.
1.13. Seja R e n um inteiro positivo. Calcule o resto da divis
ao do
n
2
polin
omio (cos + x sin ) R[x] por x + 1.
1.14 (IMO1993). Seja f (x) = xn + 5xn1 + 3 onde n > 1. Demonstrar
que f (x) n
ao pode se expressar como produto de dois polin
omios n
ao
constantes com coeficientes inteiros.
1.15. Encontrar todos os pares (c, P (x)) onde c e um real e P (x) e um
polin
omio n
ao nulo tal que
P (x4 + x2 + x) = (x6 + x5 + x4 + x3 + x2 + x + 1)P (cx).
1.16 (AusPol1998). Encontrar todos os inteiros positivos n e m tais que
todas as soluc
oes de x3 17x2 + mx n2 = 0 s
ao inteiras.

1.2. ORDEM E RAIZES PRIMITIVAS

73

1.17. Dados x, y N, defina a := x(y +1)(y!+1). Mostre que imagem


da func
ao f : N N N dada por
f (x, y) =


y1
|a2 1| (a2 1) + 2
2

e exatamente o conjunto dos n


umeros primos.

1.18. Prove a seguinte modificac


ao do Criterio de Eisenstein: seja f (x) =
n
an x + + a1 x + a0 Z[x] um polin
omio primitivo n
ao constante e
sem razes racionais. Suponha que exista um n
umero primo p tal que
p an , p | aj para todo 0 j < n e p2 a1 . Ent
ao f (x) e irredutvel em
Z[x].
1.19. (Zagier) Dado um n
umero primo, associe a ele um polin
omio cujos
coeficientes s
ao os dgitos decimais desse primo (por exemplo, 9x3 +4x2 +
3 para o primo 9403). Mostre que este polin
omio e sempre irredutvel
em Z[x].
1.20. Encontrar todos os valores de k para os quais o polin
omio x2k+1 +
x + 1 e divisvel por xk + x + 1.
1.21 (IMO2002). Encontrar todos os pares de inteiros m, n > 2 tais que
existam infinitos valores de k para os quais
km + k 1
kn + k2 1
e inteiro.

1.2

Ordem e Razes Primitivas

Dado a (Z/nZ) , definimos a ordem de a, denotado por ord a,


como o menor inteiro t > 0 tal que at = 1 em Z/nZ. Se a, n Z
com mdc(a, n) = 1, definimos a ordem de a m
odulo n, denotado por
ordn a, como a ordem de a (Z/nZ) . Note que pelo teorema de EulerFermat, temos que ordn a (n). Se ordn a = (n), dizemos que a
e raiz primitiva m
odulo n. Por exemplo, 2 e raiz primitiva m
odulo 5,
1
2
3
4
pois 2 = 2, 2 = 4, 2 = 8, 2 = 16, que e a primeira potencia de 2
congruente a 1 m
odulo 5 e 4 = (5).
O resultado basico mais importante sobre ordem e a seguinte
Proposi
c
ao 1.22. Temos que at 1 (mod n) se, e s
o se, ordn a | t.
o: Como aordn a 1 (mod n), para todo k N tem-se
Demonstra
ca
k
ord
a
n
a
1 (mod n). Por outro lado, se at 1 (mod n), pelo algoritmo

CAPITULO 1. POTENCIAS
E CONGRUENCIAS

74

da divisao existem inteiros q e r tais que 0 r < ordn a e t = q ordn a+r.


Portanto
1 at = aq ordn a+r = (aordn a )q ar ar

(mod n)

Ou seja, ar 1 (mod n). Pela minimalidade de ordn a, temos que r = 0,


i.e., ordn a | t.
Corol
ario 1.23. ordn a | (n).
Exemplo 1.24. Determine a ordem de 2 m
odulo 7337 (observac
ao:
7337 = 11 23 29)
o: Como
Solu
ca
25 1 (mod 11) temos que ord11 2 = 10,
211 = 2048 1 (mod 23) temos ord23 2 = 11 e
27 = 128 12 (mod 29), 214 122 1 (mod 29) temos ord29 2 =
28.
assim ord7337 2 = mmc(10, 11, 28) = 1540.
Exemplo 1.25. Demonstrar que n | (an 1) para todo inteiro positivo
a > 1.
o: Ja que mdc(a, an 1) = 1, pelo teorema de Euler-Fermat
Solu
ca
n
temos que a(a 1) 1 (mod an 1); por outro lado, n e a ordem de
a m
odulo an 1 ja que an 1 (mod an 1) e se 0 < t < n temos
0 < at 1 < an 1 e assim an 1 at 1. Pela proposicao, temos
portanto n | (an 1).
Exemplo 1.26. Demonstrar que n
ao existe um inteiro n > 1 tal que
n | 2n 1.
o: Suponhamos o contrario; seja p o menor divisor primo de n
Solu
ca
e r = ordp 2. Sabemos que 2n 1 (mod p) e alem disso, pelo teorema
de Fermat, 2p1 1 (mod p).
Portanto r | n e r | p 1, o que implica que r | mdc(n, p 1). Mas
mdc(n, p1) = 1 pois p e o menor divisor primo de n e assim os divisores
primos de p 1 s
ao menores que os divisores primos de n. Isto mostra
que r = 1, isto e 21 1 (mod p), donde p | 1, uma contradicao.

1.2. ORDEM E RAIZES PRIMITIVAS

75

Exemplo 1.27. Sejam a, m e n inteiros positivos; defina m e n por


m = mdc(m, n) m e n = mdc(m, n) n , de modo que mdc(m , n ) = 1.
Mostre que

mdc(m,n) + 1

a
mdc(am + 1, an + 1) = 2

se m e n s
ao mpares.

se m + n e a s
ao mpares.

se m + n e mpar e a e par.

o: Como
Solu
ca

mdc(am + 1, an + 1) = mdc (amdc(m,n) )m + 1, (amdc(m,n) )n + 1 ,

o resultado no caso geral seguir


a do caso em que mdc(m, n) = 1. Assim,
vamos supor m e n s
ao primos entre si e seja d = mdc(an + 1, am + 1).
Temos
(
(
a2n 1 (mod d)
an 1 (mod d)
=
am 1 (mod d)
a2m 1 (mod d)
= ordd a | mdc(2n, 2m) = 2.

Assim, a2 1 (mod d). Digamos que m seja mpar (como estamos


supondo mdc(m, n) = 1, nao podemos ter m e n ambos pares), de modo
que
a (a2 )(m1)/2 = am 1 (mod d) = a 1 (mod d)
d | a + 1.

Se n e mpar tambem, entao d = a+1 ja que a+1 | am +1 e a+1 | an +1


neste caso (utilize a fatoracao am + 1 = (a + 1)(am1 am2 + am3
+1) ou a implicacao a 1 (mod a+1) = am 1 (mod a+1)).
Por outro lado, se n e par, temos
(a2 )n/2 = an 1 (mod d) = 1 1 (mod d)
= d = 1 ou d = 2

O caso d = 2 ocorre se, e s


o se, am + 1 e an + 1 s
ao ambos pares, ou
seja, quando a e mpar. Isto encerra a an
alise de casos e com isso o
problema.
Uma outra caracterizacao de raiz primitiva e dada pela
Proposi
c
ao 1.28. O n
umero a e raiz primitiva m
odulo n se, e somente
se, {at , t N} = (Z/nZ) .

CAPITULO 1. POTENCIAS
E CONGRUENCIAS

76

o: Para todo a Z com mdc(a, n) = 1 temos {at , t


Demonstra
ca

N} (Z/nZ) . Note que {at , t N} = {1, a, a2 , . . . , aordn a1 } e um


conjunto com ordn a elementos. De fato, para qualquer t N temos
at = ar onde r e o resto na divisao de t por ordn a; por outro lado,
ao distintos pois caso ai = aj com
os elementos 1, a, a2 , . . . , aordn a1 s
0 i < j < ordn a, entao aji = 1 com 0 < j i < ordn a, o que e
absurdo.
Assim, {at , t N} = (Z/nZ) se, e s
o se, mdc(a, n) = 1 e ordn a =
(n), isto e, se, e s
o se, a e uma raiz primitiva m
odulo n.
Corol
ario 1.29. Se m divide n e a e raiz primitiva m
odulo n, ent
ao a
e raiz primitiva m
odulo m.
o: Como o mapa natural (Z/nZ) (Z/mZ) que
Demonstra
ca
leva x mod n em x mod m e sobrejetor, temos que se as potencias de
a mod n cobrem todo o (Z/nZ) , entao as potencias de a mod m tambem cobrem todo o (Z/mZ) . Pela proposicao, isto implica o corolario.
Razes primitivas s
ao muito u
teis em diversas quest
oes de Teoria dos
N
umeros. Entretanto elas nem sempre existem para qualquer m
odulo
n. O resto desta secao e dedicado a provar o seguinte importante
Teorema 1.30. Existe alguma raiz primitiva m
odulo n se, e s
o se,
n = 2, n = 4, n = pk ou n = 2pk onde p e primo mpar.
A demonstracao deste teorema e longa e e composta de varios passos.
Comecamos com a seguinte
Proposi
c
ao 1.31. Se k 3, ent
ao n
ao existe nenhuma raiz primitiva
k
m
odulo 2 .
o: Pelo corolario anterior, basta provar que nao existe
Demonstra
ca
raiz primitiva m
odulo 8, e isso segue do fato de que se mdc(a, 8) = 1,
isto e, a = 2r + 1, r N, entao a2 = 4r(r + 1) + 1 1 (mod 8) (sendo
r(r+1) par, visto que e o produto de dois n
umeros consecutivos). Assim,
nao ha elemento de ordem (8) = 4 m
odulo 8.
Proposi
c
ao 1.32. Se n = ab, com a 3 e b 3 inteiros tais que
mdc(a, b) = 1, ent
ao n
ao existe raiz primitiva m
odulo n.
o: Como (n) = (a)(b) e a 3 e b 3, segue que
Demonstra
ca
(a) e (b) s
ao pares (verifique!). Se mdc(k, n) = 1, entao temos
k (n)/2 = (k (b)/2 )(a) 1 (mod a)
k

(n)/2

= (k

(a)/2 (b)

1 (mod b).

1.2. ORDEM E RAIZES PRIMITIVAS

77

Assim, k (n)/2 1 (mod n) e portanto ordn k (n)/2 < (n) para


todo k primo com n.
Proposi
c
ao 1.33. Se p e um n
umero primo e a Z e uma raiz primitiva m
odulo p, ent
ao a ou a + p e raiz primitiva m
odulo p2 .
o: Por hip
Demonstra
ca
otese, ordp a = ordp (a + p) = (p) = p 1.
Portanto p 1 | ordp2 a, pois at 1 (mod p2 ) implica at 1 (mod p).
Alem disso, como ordp2 a | (p2 ) = p(p 1), devemos ter ordp2 a = p 1
ou ordp2 a = p(p 1) = (p2 ). Do mesmo modo, ordp2 (a + p) = p 1 ou
ordp2 (a + p) = p(p 1) = (p2 ). Basta provar, portanto, que ordp2 a 6=
p 1 ou ordp2 (a + p) 6= p 1. Suponha que ordp2 a = p 1. Portanto
ap1 1 (mod p2 ) e assim




p 1 p2
p 1 p3 2
p1
p1
(a + p)
=a
+
a p+
a p +
1
2
1 pap2

(mod p2 ).

Portanto (a + p)p1 nao e congruente a 1 m


odulo p2 , pois p2 nao divide
pap2 (lembre-se de que mdc(a, p) = 1), donde ordp2 (a+p) 6= p1.
Proposi
c
ao 1.34. Se p e um n
umero primo mpar e a e raiz primitiva
2
m
odulo p , ent
ao a e raiz primitiva m
odulo pk para todo k N.
o: Como ap1 1 (mod p), mas ap1 nao e congruente
Demonstra
ca
2
a 1 m
odulo p (ja que a e raiz primitiva m
odulo p2 ), temos ap1 = 1+b1 p,
k1
onde p nao divide b1 . Vamos mostrar por inducao que ap (p1) =
1 + bk pk , onde p nao divide bk , para todo k 1. De fato, para k 1 e
p > 2 primo,
 
 
p 2 2k
p
k
pk (p1)
k p
b p +
bk p +
a
= (1 + bk p ) = 1 +
2 k
1
= 1 + pk+1 (bk + pt)

para algum t Z e assim bk+1 = bk + pt tambem nao e divisvel por p


pois p bk .
Vamos agora mostrar por inducao que a e raiz primitiva m
odulo pk
k
para todo k 2. Suponha que a seja raiz primitiva m
odulo p . Como
ordpk+1 a
ordpk+1 a
k+1
k
a
1 (mod p ) = a
1 (mod p ) temos
pk1 (p 1) = (pk ) = ordpk a | ordpk+1 a | (pk+1 ) = pk (p 1).
Portanto ordpk+1 a = pk1 (p 1) ou ordpk+1 a = pk (p 1) = (pk+1 ),
k1
mas o primeiro caso e impossvel pois ap (p1) = 1 + bk pk com p bk .
odulo pk+1 .
Logo ordpk+1 a = (pk+1 ) e a e raiz primitiva m

CAPITULO 1. POTENCIAS
E CONGRUENCIAS

78

Por exemplo 2 e raiz primitiva m


odulo 5k para todo k 1. De fato,
2 e raiz primitiva m
odulo 5 e, como 24 = 16 6 1 (mod 25), 2 e raiz
primitiva m
odulo 25 = 52 tambem. Portanto, pela proposicao anterior,
2 e raiz primitiva m
odulo 5k para todo k 1.
Proposi
c
ao 1.35. Se p e primo mpar e a e um inteiro mpar tal que
a e raiz primitiva m
odulo pk , ent
ao a e raiz primitiva m
odulo 2pk . Em
k
particular, se a e raiz primitiva qualquer m
odulo p , ent
ao a ou a + pk
e raiz primitiva m
odulo 2pk (pois um deles e mpar).
o: Temos, como nas provas acima, (pk ) = ordpk a |
Demonstra
ca
ord2pk a e ord2pk a | (2pk ) = (pk ), logo ord2pk a = (2pk ).
Para completar a prova do teorema 1.30, falta provar que se p e primo
mpar, entao existe raiz primitiva m
odulo p. Para isto, precisamos de
dois lemas.
Lema 1.36.

d|n (d)

= n para todo n N.

o: Seja d um divisor de n. A quantidade de as tais


Demonstra
ca
que 1 a n e d = mdc(n, a) e igual a ( nd ) pois d = mdc(n, a)
d | a e 1 = mdc( nd , ad ). Como ( nd ) conta justamente a quantidade
de
entre
1 e nd (inclusive) que s
ao primos com nd , temos que
P
P inteiros
n
umeros a entre 1 e n
d|n (d) conta a quantidade de n
d|n ( d ) =
(inclusive), particionados segundo os valores de mdc(a, n).
Lema 1.37. Seja p um primo e d um divisor de p 1. Defina N (d)
como a quantidade de elementos a (Z/pZ) com ord a = d. Ent
ao
N (d) (d).
o: Podemos supor que N (d) > 0, logo existe a tal que
Demonstra
ca
ordp a = d. Logo ad = 1 e, para 0 k < d, as classes de ak s
ao todas
k
d
d
distintas m
odulo p. Como (a ) = 1 e a equacao x 1 = 0 tem no
m
aximo d razes distintas em Z/pZ (pois Z/pZ e um corpo), suas razes
s
ao exatamente ak , 0 k < d. Por outro lado, se ordp ak = d, entao
mdc(k, d) = 1, pois caso r = mdc(k, d) > 1, entao (ak )d/r = (ad )k/r 1
(mod p), logo ordp (ak ) d/r < d. Desta forma,
{b (Z/pZ) | ordp b = d} {ak | 0 k < d e mdc(k, d) = 1},
portanto N (d) (d) (na verdade, os dois conjuntos acima s
ao iguais,
como ficara claro a partir da demonstracao da proposicao abaixo).

1.2. ORDEM E RAIZES PRIMITIVAS

79

Proposi
c
ao 1.38. Se p e um primo, ent
ao existe uma raiz primitiva
m
odulo p.
o: Para cada a (Z/pZ) , tem-se ordp a | p 1 e
Demonstra
ca
P
portanto p 1 = d|p1 N (d). Por outro lado, temos pelos dois lemas
acima que
X
X
p1=
N (d)
(d) = p 1.
d|p1

d|p1

Logo devemos ter N (d) = (d) para todo d. Em particular, N (p 1) =


(p 1) > 0, logo existem razes primitivas m
odulo p.
Corol
ario 1.39. Seja p um primo. Para cada d | p 1, existem exatamente (d) elementos em (Z/pZ) com ordem d. Em particular, p
possui exatamente (p 1) razes primitivas.
Com isto, encerramos a demonstracao do teorema 1.30. Vejamos
algumas aplicacoes.
Exemplo 1.40. Determine uma raiz primitiva m
odulo 419.
o: Sabemos que (419) = 418 = 2 11 19, e os divisores
Solu
ca
proprios de 418 s
ao 1, 2, 11, 19, 22, 38 e 209. Assim, precisamos encontrar um a que nao tenha nenhum destes n
umeros como ordem. Vejamos
se a = 2 funciona. Para isto observemos que, m
odulo 419,
22 = 4,

211 = 2048 372,

222 3722 114,

219 372 256 119

238 1192 334,

2209 11911 418

logo ord419 2 = 418, e 2 e raiz primitiva m


odulo 419.
Observe que todas as razes primitivas m
odulo 419 podem ser geradas
calculando 2k (mod 419), onde mdc(k, 418) = 1, assim por exemplo
25 = 32 e 29 93 (mod 419) tambem s
ao razes primitivas.
Exemplo 1.41. Mostre que existe n natural tal que os mil u
ltimos dgitos de 2n pertencem a {1, 2}.
o: Observamos inicialmente que para todo k N existe um
Solu
ca
n
umero mk de k algarismos, todos 1 ou 2, divisvel por 2k . De fato,
m1 = 2 e m2 = 12 satisfazem o enunciado. Seja mk = 2k rk , rk N. Se
rk e par, tome mk+1 = 210k + mk = 2k+1 (5k + rk /2), e se rk e mpar,
tome mk+1 = 10k + mk = 2k+1 (5k + rk )/2.
1000
. Portanto,
Como m1000 2 (mod 10), 5 nao divide r1000 = m
21000
1000
como 2 e raiz primitiva m
odulo 5
pela proposicao 1.34, existe k N

80

CAPITULO 1. POTENCIAS
E CONGRUENCIAS

com 2k r1000 (mod 51000 ). Logo 2k = b51000 + r1000 para algum b N


e assim
2k+1000 = b101000 + 21000 r1000 = b101000 + m1000 ,
e as 1000 u
ltimas casas de 2k+1000 s
ao as 1000 casas de m1000 , que
pertencem todas a {1, 2}.
Observa
c
ao 1.42. Um grupo G e chamado de cclico se existe um elemento g tal que G = {g n | n Z}. O fato de pn e 2pn , p primo mpar,
admitirem razes primitivas equivale a dizer que os grupos (Z/pn Z) e
(Z/2pn Z) s
ao cclicos, ou ainda que h
a isomorfismos de grupos (Z/pn Z)
=
n
n
n

ao nos grupos da direita


Z/(p ) e (Z/2p Z) = Z/(2p ) onde a operac
e a adic
ao.
O leitor n
ao deve ter dificuldades para adaptar a prova acima a fim de
mostrar que todo corpo K com um n
umero finito de elementos (tal como
o construdo no exemplo ap
os o teorema 1.14) admite raiz primitiva, isto
e, o seu grupo de unidades K = K \ {0} e um grupo cclico.

Problemas Propostos
1.22. Determine a ordem de 3 m
odulo 200.
1.23. Encontre uma raiz primitiva m
odulo 71.
1.24. Sabendo que ord132 a = 4, ord112 a = 55 e ord72 a = 21, determine
ord10012 a.
1.25. Determine todos os valores inteiros de n tal que n|22n + 1.
1.26. Determine uma raiz primitiva m
odulo 73 .
1.27. Encontrar as ordens de 2 e 5 m
odulo 101. Encontrar tambem
todos os elementos de ordem 20 em (Z/101Z) .
1.28. Demonstrar que 2n | (an + 1) para todo inteiro positivo a.
1.29 (IMO1978). Sejam m e n inteiros positivos com m < n. Se os
tres u
ltimos algarismos de 1978m s
ao os mesmos que os tres u
ltimos
algarismos de 1978n , encontrar m e n tais que m + n assume o menor
valor possvel.
n

1.30. Sejam d e n n
umeros naturais tais que d | 22 + 1. Demonstre que
existe um inteiro k tal que d = k2n+1 + 1.


1.3. RESIDUOS QUADRATICOS
E SIMBOLO DE LEGENDRE 81
1.31. Seja k 2 e n1 , n2 , . . . , nk 1 n
umeros naturais que tem a
propriedade
n2 | (2n1 1),

n3 | (2n2 1), . . . , nk | (2nk1 1) e n1 | (2nk 1)

Demonstrar que n1 = n2 = = nk = 1.
1.32. Mostrar que x3 x + 1 e irredutvel em Z/3Z[x]. Encontrar todas
as razes primitivas do corpo finito (xZ/3Z[x]
3 x+1) .
1.33 (APMO1997). Encontrar um n no conjunto {100, 101, . . . 1997} tal
que n divide 2n + 2.
1.34. Definimos a funcao de Carmichael : N N como o menor inteiro positivo tal que a(n) 1 (mod n) para todo a primo com n. Observe que, pelo teorema 1.30, (pl ) = pl1 (p 1) para todo p primo
mpar. Mostrar que
(a) (2) = 1, (4) = 2 e (2l ) = 2l2 para todo l 3.
(b) Se n = p1 1 . . . pk k e a fatorac
ao em primos de n, ent
ao
(n) = mmc{(p1 1 ), . . . , (pk k )}.
1.35 (IMO2000). Existe um inteiro N divisvel por exatamente 2000
primos diferentes e tal que N divide 2N + 1?
Dica: Tente construir indutivamente inteiros Nk divisveis por exatamente k primos distintos e tais que Nk | 2Nk + 1.
1.36 (IMO1990). Encontrar todos os n
umeros naturais n tais que
n2 | 2n + 1.
1.37 (IMO1999). Encontrar todos os pares (n, p) de inteiros positivos
tais que p e primo, n 2p e (p 1)n + 1 e divisvel por np1 .
1.38 (Banco-IMO2000). Determine todas as triplas (a, m, n) de inteiros
positivos tais que am + 1 | (a + 1)n .

1.3

Resduos Quadr
aticos e Smbolo de Legendre

Seja p > 2 um n
umero primo e a, b, c Z com a nao divisvel por p.
Resolver a equacao quadratica
ax2 + bx + c 0 (mod p)

82

CAPITULO 1. POTENCIAS
E CONGRUENCIAS

e o mesmo que resolver (completando quadrados)


(2ax + b)2 b2 4ac

(mod p)

(note que 2 e a s
ao invertveis m
odulo p). Assim, estamos interessados
em encontrar criterios de existencia de solucoes da equacao
X2 d

(mod p).

Se a equacao acima admite solucao (i.e. se d e um quadrado perfeito


em Z/pZ) entao dizemos que d e um resduo ou resto quadr
atico m
odulo
p. H
a exatamente (p + 1)/2 resduos quadraticos m
odulo p, a saber
2

0 ,1 ,2 ,3 ,...,

p1
2

2

mod p

ja que todo inteiro x e congruente a i mod p para algum i tal que


0 i (p 1)/2, de modo que x2 e congruente a um dos n
umeros da
lista acima. Note que m
odulo p estes n
umeros s
ao todos distintos: de
fato, temos que
i2 j 2

(mod p) = p | (i j)(i + j)

p | i j ou p | i + j
i j

(mod p)

Mas como 0 i, j (p 1)/2 = 0 < i + j p 1 ou i = j = 0,


temos que a u
nica possibilidade e i j (mod p).
Embora saibamos a lista completa dos resduos quadraticos, na pratica pode ser difcil reconhecer se um n
umero e ou nao resduo quadratico. Por exemplo, voce sabe dizer se 2 e resduo quadratico m
odulo
1019? Veremos a seguir o teorema da reciprocidade quadratica, que
permite responder estas quest
oes de maneira bastante eficiente.
Seja p > 2 um n
umero primo e a um inteiro qualquer. Para simplificar calculos e notacoes definiremos o chamado smbolo de Legendre:

odulo p
 
1 se p a e a e um resduo quadratico m
a
=
0 se p | a

1 caso contrario
Proposi
c
ao 1.43 (Criterio de Euler). Seja p > 2 um primo e a um
inteiro qualquer. Ent
ao
 
a
a(p1)/2 (mod p).
p


1.3. RESIDUOS QUADRATICOS
E SIMBOLO DE LEGENDRE 83
o: Para a 0 (mod p) o resultado e claro, de modo
Demonstra
ca
que podemos supor p a. Pelo teorema de Fermat temos que ap1 1
(mod p), donde
(a

p1
2

1)(a

p1
2

+ 1) 0 (mod p) p | a
a

p1
2

p1
2

1 ou p | a

p1
2

+1

1 (mod p).

p1

Assim, devemos mostrar que a 2 1 (mod p) se, e s


o se, a e um
resduo quadratico m
odulo p.
Se a e um resduo quadratico, digamos a i2 (mod p), novamente
pelo teorema de Fermat temos que
a

p1
2

ip1 1 (mod p).

2
odulo p s
ao razes do
Assim, os resduos quadraticos 12 , 22 , . . . , ( p1
2 ) m
p1

polin
omio f (x) = x 2 1 em Z/(p)[x]. Mas Z/(p) e corpo, logo f (x)
pode ter no m
aximo deg f = (p 1)/2 razes em Z/(p). Isto mostra que
as razes de f (x) s
ao exatamente os resduos quadraticos nao congruentes
p1
a zero m
odulo p e que, portanto, a 2 1 (mod p) se, e s
o se, a e um
resduo quadratico m
odulo p.
Corol
ario 1.44. O smbolo de Legendre possui as seguintes propriedades:


1. se a b (mod p) ent
ao ap = pb .
2.

a2
p

1
p

= 1 se p a.
p1

= (1) 2 , ou seja, 1 e resduo quadr


atico m
odulo p se, e
s
o se, p 1 (mod 4).

 
a b
4. ab
p = p p .

3.

o: Os itens 1 e 2 s
Demonstra
ca
ao imediatos a partir da definicao


p1
1
=
e 3 segue do criterio de Euler: p (1) 2 (mod p) = 1
p
p1

ao iguais a 1.
(1) 2 ja que p > 2 e ambos os lados da congruencia s
Da mesma forma, aplicando o criterio de Euler temos que
 
  
p1 p1
p1
ab
a
b
(ab) 2 a 2 b 2
(mod p),
p
p p

 
= ap pb , pois novamente ambos os lados da
o que mostra que ab
p
congruencia s
ao iguais a 1.

CAPITULO 1. POTENCIAS
E CONGRUENCIAS

84

Exemplo 1.45. Mostre que o polin


omio f (x) = x4 10x2 + 1 e irredutvel em Z[x], mas e redutvel m
odulo p para todo primo p.
o: Vejamos que f (x) e irredutvel em Z[x]. Observe inicialSolu
ca
mente que as razes de f (x) s
ao todas irracionais: se p, q Z s
ao tais
que mdc(p, q) = 1 e f (p/q) = 0 p4 10p2 q 2 + q 4 = 0, temos da
u
ltima igualdade que q | p4 = q = 1 e p | q 4 = p = 1 ja que p e
q s
ao primos entresi, logo
p/q = 1, nenhuma das quais e raiz de f (x)
(cujos zeros s
ao 2 3).
Logo se f (x) for redutvel ele e o produto de dois polin
omios de grau
2, que podemos supor m
onicos. Como o produto dos coeficientes independentes destes dois fatores deve ser igual ao coeficiente independente
de f (x), que e 1, temos apenas duas possibilidades:
f (x) = (x2 + ax + 1)(x2 + bx + 1)
2

ou

f (x) = (x + ax 1)(x + bx 1)
com a, b Z.
x3 ). Logo, no
ab + 2 = 10
an
alogo.
Agora, para

Em ambos os casos, temos a + b = 0 (coeficiente de


primeiro caso, comparando o coeficiente de x2 temos
a2 = 12, o que e impossvel. O segundo caso e
p = 2 e p = 3 temos

f (x) (x + 1)4

f (x) (x2 + 1)2 (mod 3).





Agora se p > 3 e um primo, temos que ou p2 = 1, ou p3 = 1 ou p6 = 1
 

ja que p2 p3 = p6 . No primeiro caso, se a2 2 (mod p) temos
(mod 2)

f (x) (x2 + 2ax 1)(x2 2ax 1)

(mod p).

Ja no segundo caso, se b2 3 (mod p) temos


f (x) (x2 + 2bx + 1)(x2 2bx + 1)

(mod p).

Finalmente, no u
ltimo caso, se c2 6 (mod p) temos
f (x) (x2 + 2c 5)(x2 2c 5)

(mod p).

Isto mostra que f (x) e redutvel m


odulo p para todo primo p.

1.3.1

Lema de Gau

O criterio de Euler ja nos fornece uma maneira de identificar resduos quadraticos. Veremos agora outro resultado
que fornece varias

interpretacoes para o smbolo de Legendre ap .


1.3. RESIDUOS QUADRATICOS
E SIMBOLO DE LEGENDRE 85
Lema 1.46 (Gau). Sejam p > 2 um n
umero primo e a um inteiro
primo relativo com p. Seja s o n
umero de elementos do conjunto


a, 2a, 3a, . . . , p1
2 a

tais que seu resto m


odulo p e maior do que
 
a
= (1)s .
p

p1
2 .

Ent
ao

Interpretamos aqui o resto de n m


odulo p como sendo o u
nico inteiro

com 0 n < p e n n (mod p). Em particular, mesmo se a for


negativo os restos no enunciado serao maiores ou iguais a 0.
o: A ideia e imitar a prova do teorema de Euler-Fermat.
Demonstra
ca
e um sistema completo de invertComo o conjunto {1, 2, . . . , p1
2 }
veis m
odulo p, para cada j = 1, 2, . . . , p1
2 podemos escrever a j j mj
(mod p) com j {1, 1} e mj {1, 2, . . . , p1
2 }. Temos que se i 6= j
entao mi 6= mj donde {m1 , m2 , . . . , m p1 } = {1, 2, . . . , p1
2 }. De fato, se
2
mi = mj temos a i a j (mod p) ou a i a j (mod p); como a e
invertvel m
odulo p e 0 < i, j (p 1)/2, temos que a primeira possibilidade implica i = j e a segunda e impossvel. Assim, multiplicando as
congruencias a j j mj (mod p), obtemos
n

(a 1)(a 2) (a p1
m1 m2 m p1
2 ) 1 2 p1
2
2




p1
p1
p1
a 2
! 1 2 p1
! (mod p)
2
2
2
 
a

1 2 . . . p1 (mod p),
2
p

(mod p)

donde ( ap ) = 1 2 . . . p1 , pois ambos os lados pertencem a {1, 1}.


2

Assim, ( ap ) = (1)s ja s e o n
umero de elementos j de {1, 2, . . . , p1
2 }
tais que j = 1.
Vejamos uma aplicacao facil deste resultado.
Lema 1.47. Seja p um primo mpar. Ent
ao
(
 
p1
1 se p 1 (mod 4),
1
= (1) 2 =
p
1 se p 3 (mod 4),
e

(
 
p2 1
1
2
= (1) 8 =
p
1

se p 1 (mod 8),
se p 3 (mod 8).

CAPITULO 1. POTENCIAS
E CONGRUENCIAS

86

Note que ja demonstramos a primeira formula na secao anterior;


daremos uma segunda demonstracao.
o: No Lema de Gau, tome a = 1. Temos
Demonstra
ca




a, 2a, 3a, . . . , p1
2 a = (p + 1)/2, . . . , p 3, p 2, p 1

donde s = (p 1)/2 e temos que

 
1
= (1)(p1)/2 .
p
Para a segunda formula, tome a = 2. Se p 1 (mod 4), digamos
p = 4k + 1, temos p1
= 2k. Como 1 2j p1
2
2 para j k e
p1
<
2j

1
para
k
+
1

2k,
temos
2
(
 
1,
2
k
= (1) =
p
1,

se p 1
se p 5

(mod 8),
(mod 8).

Se p 3 (mod 4), digamos p = 4k + 3, temos p1


2 = 2k + 1. Para
p1
1 j k temos 1 2j 2 e para k + 1 j 2k + 1 temos
p1
2 < 2j p 1, donde
(
 
1,
2
k+1
= (1)
=
p
1,

se p 3 (mod 8),
se p 7 (mod 8).

O leitor pode modificar a demonstra



cao acima para descobrir formulas an
alogas para, digamos, p3 e p5 . A Lei (ou o Teorema) de
Reciprocidade Quadratica, que veremos na proxima secao, faz isso de
forma muito geral.
Daremos agora duas reformulacoes do Lema de Gau, a primeira
geometrica e combinat
oria e a segunda mais algebrica, usando funcoes
trigonometricas. Estas reformulacoes ajudam a apreciar melhor o Lema
e serao utilizadas nas proximas secoes.
Lema 1.48. Seja p > 2 primo e a > 0 inteiro, a primo com p. Considere
em R2 o tri
angulo T de vertices (0, 0), (p/2, 0) e (p/2, a). Seja N o
n
umero de pontos de coordenadas inteiras no interior de T . Ent
ao
 
a
= (1)N .
p


1.3. RESIDUOS QUADRATICOS
E SIMBOLO DE LEGENDRE 87

A figura ilustra o resultado acima para p = 7 e a = 5: temos N = 7


 
a
= (1)N = 1,
p

consistentemente com o fato de 5 nao ser quadrado m


odulo 7. Observe
que os pontos (k, 0), 1 k (p 1)/2 estao sobre um lado de T e
portanto nao devem ser contados.

o: Note que a hipotenusa de T esta contida na reta


Demonstra
ca
y = 2ax/p. Alem disso, como a e p s
ao primos entre si nao ha nenhum
ponto de coordenadas inteiras sobre a hipotenusa (fora a origem).
Para k inteiro, 1 k p/2, seja Nk o n
umero de pontos de coordenadas inteiras dentro de T . Afirmamos que Nk e mpar se e somente se
o resto de ak m
odulo p for maior do que p/2. De fato, seja k este resto:
temos ak = lp + k , 0 < k < p. Por outro lado, por definicao, Nk e o
n
umero de inteiros positivos y para os quais y < 2ak/p. Assim,

 

   
2ak
2(lp + k )
2k
2k
Nk =
=
= 2l +

(mod 2)
p
p
p
p
(lembre que x e a parte inteira de x). Mas isto demonstra a afirmacao
pois a u
ltima expressao e igual a 1 se k > p/2 e e igual a 0 caso contrario.
Temos N = N1 + + N(p1)/2 . Pelo par
agrafo anterior temos que
N s (mod 2) (onde s e como no enunciado do Lema de Gau), o que,
pelo Lema de Gau, completa a demonstracao.
Lema 1.49. Seja p > 2 primo e a inteiro, a primo com p. Ent
ao

  


Y
Y
k
a
ak
sen 2
=
.
sen 2
p
p
p
1k(p1)/2

1k(p1)/2

o: Observe que para cada k entre 1 e (p 1)/2 existe


Demonstra
ca
um u
nico k no mesmo intervalo com ak k (mod p): assim os dois

CAPITULO 1. POTENCIAS
E CONGRUENCIAS

88

grandes produtos acima tem o mesmo m


odulo. O segundo produto e
claramente positivo. Por outro lado, temos


ak
sen 2
<0
p
se e somente se o resto de ak m
odulo p for maior do que p/2. Assim, para
s como no Lema de Gau, o n
umero de termos negativos no primeiro
produto e igual a s. Pelo Lema de Gau, isto completa a demonstracao.

Problemas Propostos

1.39. Calcule 72 ,

3
11

5
13

1.40. Quantos pontos de coordenadas inteiras existem no interior do


tri
angulo de vertices (0, 0), (3/2, 0) e (3/2, 2012) ?
1.41.
Seja p > 3 primo. Quantos pontos de coordenadas inteiras existem no
interior do tri
angulo de vertices (0, 0), (p/2, 0) e (p/2, 3) ?
Para quais primos p o polin
omio x2 3 e redutvel m
odulo p ?
n

1.42. a) (Euler) Seja Fn = 22 + 1 o n-esimo n


umero de Fermat. Prove
que todo fator primo de Fn e da forma k2n+1 + 1.
b)(Lucas) Prove que, se n 2, ent
ao todo fator primo de Fn e da
forma k2n+2 + 1.
5
c) Mostre que 22 + 1 e composto.

1.4

Lei de Reciprocidade Quadr


atica

Vejamos agora um resultado mais geral do que os das secoes anteriores e que torna facil calcular se n e um quadrado m
odulo p.
Teorema 1.50 (Reciprocidade Quadratica). Sejam p e q primos mpares distintos. Ent
ao
  
p1 q1
p q
= (1) 2 2
q
p
As identidades no Lema 1.47 s
ao muitas vezes consideradas itens
(mais faceis) da Lei de Reciprocidade Quadratica.
Na matem
atica, alguns teoremas inspiram muitas demonstracoes, `as
vezes usando ideias completamente diferentes: o teorema de Pit
agoras


1.4. LEI DE RECIPROCIDADE QUADRATICA

89

e o teorema fundamental da
algebra s
ao exemplos disso. A Lei da Reciprocidade Quadratica tambem ja tem centenas de demonstracoes diferentes, algumas bastante elementares e outras que usam tecnicas mais
avancadas. Nas subsecoes veremos tres demonstracoes diferentes para
este teorema. Vejamos agora algumas aplicacoes.
Exemplo 1.51. Determinar se 90 e resduo quadr
atico m
odulo 1019
ou n
ao.
o:
Solu
ca


 

 2 

90
1
2
3
5
=
1019
1019 1019 1019 1019


1019
= (1) (1) 1
5
   2
2
4
=
=1
=
5
5

Ou seja, 90 e resduo quadratico m


odulo 1019.
Exemplo 1.52. Seja p um n
umero primo. Mostre que
1. se p e da forma 4n + 1 ent
ao p | nn 1.
2. se p e da forma 4n 1 ent
ao p | nn + (1)n+1 2n.
o:
Solu
ca
obtemos

No primeiro item, 4n 1 (mod p), donde elevando a n


(4n)n = 22n nn (1)n

(mod p).

Por outro lado, pelo criterio de Euler e pela reciprocidade quadratica


temos
22n = 2

p1
2

(1)

p2 1
8

(1)n(2n+1) (1)n

(mod p)

Portanto nn 1 (mod p), como queramos demonstrar.


No segundo item, temos 4n 1 (mod p) e assim
(4n)n = 22n nn 1
p1

p2 1

(mod p)

mas 22n1 = 2 2 (1) 8 = (1)n(2n1) (mod p), donde 22n 2


(1)n (mod p). Conclumos que 2nn (1)n (mod p) e multiplicando
por 2n e utilizando 4n 1 (mod p) obtemos nn 2n (1)n (mod p),
como desejado.

90

1.4.1

CAPITULO 1. POTENCIAS
E CONGRUENCIAS

Uma demonstrac
ao combinat
oria

Nossa primeira demonstracao baseia-se em contar pontos de coordenadas inteiras, como no Lema 1.48; vamos inicialmente enunciar uma
versao um pouco diferente daquele resultado.
Lema 1.53. Seja p > 2 primo e q > 0 inteiro mpar, q primo com p.
Considere em R2 o tri
angulo T de vertices (0, 0), (p/2, 0) e (p/2, q/2).

Seja N o n
umero de pontos de coordenadas inteiras no interior de T.
Ent
ao
 
q

= (1)N .
p
Vale a pena chamar a atencao para as diferencas entre o Lema 1.48
e o Lema 1.53. Neste segundo lema, o inteiro a foi (suspeitamente)
renomeado de q, que agora deve ser mpar. Alem disso, o terceiro vertice
de T nao e um dos vertices de T : na figura (com p = 7 e q = 5)
= 3. Note que neste
mostramos os dois tri
angulos. Temos N = 7 e N
(mod 2); dado o Lema 1.48, e exatamente isso que
exemplo N N
precisamos demonstrar que vale em geral.

o: Seja T o tri
Demonstra
ca
angulo de vertices (0, 0), (p, 0) e (p, q).
O n
umero N (como no Lema 1.48) pode ser interpretado como o n
umero
de pontos em T com coordenadas inteiras e coordenada x par: de fato, a
cada ponto (k, l) no interior de T corresponde o ponto (2k, l) no interior
de T.


1.4. LEI DE RECIPROCIDADE QUADRATICA

91

Podemos girar os pontos como acima de coordenadas x > p/2 de


meia volta ao redor do ponto (p/2, q/2) para obtermos ainda outra interpretacao para N . Consideremos agora o ret
angulo R de vertices (0, 0),
(p/2, 0), (0, q), (p/2, q) cortado pelo segmento (0, 0), (p/2, q/2). Para obtermos N , contamos os pontos de coordenadas inteiras no interior de R
tais que a coordenada x e par se e somente se o ponto esta abaixo do
segmento.

. Os pontos de coorVamos representar nesta figura a soma N + N


se estiverem abaixo
denada x mpar agora s
ao contados uma vez (por N
do segmento e por N se estiverem acima); os pontos de coordenada x
par s
ao contados 0 vezes se estiverem acima do segmento e 2 vezes se
estiverem abaixo (representados por dois crculos).

. Para fins de estudar


Ora, estamos interessados na paridade de N +N
a paridade, contar um pontos duas vezes equivale a nao cont
a-lo. Assim
e congruo m
N +N
odulo 2 ao n
umero de pontos de coordenadas inteiras
no interior de R com coordenada x par. Mas este n
umero e m
ultiplo
de q 1 (temos q 1 pontos por coluna), logo par, completando a
demonstracao.
Vejamos finalmente a prova da Lei de Reciprocidade Quadratica.

CAPITULO 1. POTENCIAS
E CONGRUENCIAS

92

o: Consideremos os tri
Demonstra
ca
angulos T1 e T2 de vertices (0, 0),
(p/2, 0), (p/2, q/2) e (0, 0), (0, q/2) e (p/2, q/2). Sejam N1 e N2 o n
umero
de pontos de coordenadas inteiras no interior de T1 e T2 , respectivamente.

Pela Lema 1.53,


 
q
= (1)N1 ,
p

 
p
= (1)N2 .
q

Mas N1 + N2 = (p 1)(q 1)/4. Assim,


  
(p1)(q1)
q
p
4
,
= (1)
p q
como queramos.

1.4.2

Uma demonstrac
ao trigonom
etrica

Nesta subsecao usaremos as abreviacoes


s(x) = sen(2x),

c(x) = cos(2x).

Assim as funcoes s e c tem perodo 1. Por exemplo, s(x) = 0 se e somente


se x e inteiro ou da forma inteiro mais meio. Primeiro demonstraremos
uma formula para um produto de senos.
Lema 1.54. Seja q > 0 um inteiro mpar. Ent
ao
s(qx) = (4)

q1
2

0l<q

l
s x+
q

Assim, por exemplo,




1
s(3x) = 4s(x)s x +
3

 

2
s x+
.
3


1.4. LEI DE RECIPROCIDADE QUADRATICA

93

H
a muitas maneiras de demonstrar estas identidades: a demonstracao
abaixo usa n
umeros complexos e as formulas de Euler:
e(x) = c(x) + is(x) = exp(2ix),
e(x) e(x)
e(x) + e(x)
s(x) =
, c(x) =
.
2i
2
o: Seja = e(1/q) e uma raiz do polin
Demonstra
ca
omio wq 1 e
temos
Y
wq 1 =
w j.
0j<q

Seja z = e(x); temos

z q z q
,
s(qx) =
2i

l
s x+
q

l z l z 1
.
2i

Substituindo na identidade do enunciado, devemos portanto demonstrar


que

Y 
q
q
l
l 1
z z =
z z
.
0l<q

Multiplicando o lado esquerdo por z q e cada fator do lado direito por z,


devemos mostrar que

Y 
z 2q 1 =
l z 2 l .
0l<q

Ora, colocando l em evidencia, temos




Y 
Y 
l z 2 l = q(q1)/2
z 2 2l .
0l<q

0l<q

Como q e mpar, (q 1)/2 e inteiro e portanto q(q1)/2 = 1; quando l


varia de 0 a q 1 temos que 2l corre um sistema completo de resduos
equivalente portanto a j correr de 0 a q 1. Devemos portanto provar
que
Y

z2 j .
z 2q 1 =
0j<q

Fazendo

z2

= w, esta e a primeira identidade da demonstracao.

Lema 1.55. Seja q > 0 um inteiro mpar. Ent


ao


 

Y
q1
l
l
s(qx)
2
s x+
= (4)
s x
.
s(x)
q
q
1l(q1)/2

o: Segue imediatamente do lema anterior, passando


Demonstra
ca
s(x) para o denominador e juntando os termos correspondentes a l e
q l.

CAPITULO 1. POTENCIAS
E CONGRUENCIAS

94

Lema 1.56. Seja p > 2 primo e q inteiro mpar, q primo com p. Ent
ao
 
 
 

Y
(p1)(q1)
k
q
l
l
k
4
s
+

= (4)
s
.
p
p q
p q
1k(p1)/2,1l(q1)/2

o: Pelo Lema 1.49 temos


Demonstra
ca
 
Y
s(qk/p)
q
.
=
p
s(k/p)
1k(p1)/2

Aplicando o Lema 1.55 a cada termo temos

 
 
 

Y
Y
q1
k
q
l
l
k
(4) 2
.
s
+

=
s
p
p q
p q
1k(p1)/2

1l(q1)/2

Passando as potencias de 4 para fora do produtorio e juntando os dois


produtorios temos a formula do enunciado.

Estamos agora prontos para concluir a segunda demonstracao da Lei


de Reciprocidade Quadratica.
o: Se p e q s
Demonstra
ca
ao primos distintos e maiores do que 2
entao usando duas vezes o Lema 1.56 temos
 


q
s kp + ql s kp ql
Y
(p1)(q1)
p
4
 
 = (1)

,
p =
l
l
k
k
q
1k(p1)/2,1l(q1)/2 s p + q s p + q

como queramos.

1.4.3

Uma demonstrac
ao usando corpos finitos

Nas duas subsecoes anteriores apresentamos duas demonstracoes bastante elementares da Lei de Reciprocidade Quadratica. Nossa terceira
demonstracao usa ideias de
algebra em um sentido mais moderno, especialmente corpos finitos.
Ao longo de toda a subsecao, sejam p, q primos mpares distintos.
Desejamos construir um corpo K com Z/(q) K dentro do qual exista
uma raiz p-esima da unidade . Para isso, considere no anel de polin
omios (Z/(q))[Z] o polin
omio
R0 =

Zp 1
= Z p1 + Z p2 + + Z + 1;
Z 1


1.4. LEI DE RECIPROCIDADE QUADRATICA

95

seja R1 um fator irredutvel de R0 . Finalmente, sejam


K=

(Z/(q))[Z]
,
(R1 )

= Z.

(Mais precisamente, e a classe de equivalencia de Z no quociente.)


Assim, por exemplo, se p = 3 e q = 7 temos
R0 = Z 2 + Z + 1 = (Z 2)(Z 4)
e podemos tomar R1 = Z 2, K = Z/(7) e = 2. Por outro lado, se
p = 3 e q = 5 temos que R0 = Z 2 + Z + 1 e irredutvel (verifique!) e
tomamos portanto R1 = R0 : neste caso K e um corpo de 25 elementos:
K = {a + b, a, b Z/(5)},

2 = 1 .

Se p = 5 e q = 7 entao novamente R0 e irredutvel e K tem 74 elementos.


Ja se p = 7 e q = 13 entao R0 e o produto de tres polin
omios irredutveis
de grau 2 (encontre-os!) e K tem 132 elementos.
Definimos agora em K as somas de Gau :
X j 
g=
j.
p
0<j<p



Assim, por exemplo, para p = 3 e q = 7 temos g = 31 + 23 2 = 2 =
24 = 5 Z/(7) K. Para p = 3 e q = 5 temos g = 2 = 1+2 K.
A partir de agora usaremos a notacao
(
 
p,
p 1 (mod 4),
1
p =
p=
p
p, p 1 (mod 4).
Lema 1.57. Sejam p, q, K, e g K como acima. Ent
a o g 2 = p .
o: Temos
Demonstra
ca
g2 =

0<j1 ,j2 <p





X
X
j1 j2 j1 +j2
k(j k) j

=
p
p
0j<p

0k<p

Ora, demonstraremos no Lema 1.58 abaixo que


(

X k(j k)
(p 1) 1
p , j 0 (mod p),

=
p
(1) 1
j 6 0 (mod p).
p ,
0k<p

Assim
 
X
1 2
g = (p 1) +
(1) j = p (1 + + 2 + + p1 ) = p.
p
0<j<p

CAPITULO 1. POTENCIAS
E CONGRUENCIAS

96
Lema 1.58.

(

X k(j k)
(p 1) 1
p ,

=
p
(1) 1
p ,

0k<p

j 0 (mod p),

j 6 0 (mod p).

o: No caso j = 0 temos
Demonstra
ca

  2  
k(j k)
k
1
=
=
p
p
p
para todo k 6= 0, demonstrando a identidade desejada. Para j 6= 0
considere o conjunto P = Z/(p) {} e a funcao m : P P definida
por
jk

k , k 6= 0, ,
m(k) = ,
k = 0,

1, k = .

Observe que m e uma bijecao. Temos




 
X k(j k)
X
X
m(k)
k
=
=
=
p
p
p

0k<p
kP r{0,}
k P r{,1}




 

X
1
1
k

=
.
p
p
p

0k <p

Lema 1.59. Sejamp, q, K e g K como acima. Ent


ao g Z/(q) K
p
se e somente se q = 1.

o: Sabemos que g 2 = p . Suponha inicialmente que


Demonstra
ca
p 
ao existe raiz quadrada de p em Z/(q) donde g
/ Z/(q).
q = 1. N
p 
Suponha agora que q = 1. Existem elementos a Z/(q) com
2
a = p . Mas isto significa que em K[X] temos X 2 p = (X a)(X +a),
ou seja, os u
nicos elementos x de K com x2 = p s
ao x = a. Assim
g = a e g Z/(q).
Generalizamos as somas de Gau: para k um inteiro primo com p,
X j 
gk =
kj ;
p
0<j<p

assim g = g1 . Observe que


gk =

X kj  k 
j =
g.
p
p

0<j <p


1.4. LEI DE RECIPROCIDADE QUADRATICA

97

Lema 1.60. Sejam


p, q, K, g K como acima. Ent
ao g Z/(q) se e

somente se pq = 1.

o: Seja f : K K definida por f (x) = xq . Observe


Demonstra
ca
que f (x) = x para todo x Z/(q) (Fermat). Reciprocamente, f (x) = x
implica que x e raiz de X q X, um polin
omio de grau q; ora, os elementos
de Z/(q) nos dao q razes e estas s
ao portanto as u
nicas razes. Em outras
palavras, f (x) = x se e somente se x Z/(q).
Observe que f (xy) = f (x)f (y). Alem disso,
f (x + y) = (x + y)q =
 
 
q q1
q qk k
q
=x +
x y + +
x y + + yq =
1
k
= xq + y q = f (x) + f (y)

pois kq e m
ultiplo de q para 0 < k < q.
Temos

q
 
X j 
X j 
q
j

f (g) =

qj = gq =
g.
=
p
p
p
0<j<p

0<j<p

Isto completa a demonstracao.

Lema 1.61. Sejam p e q primos mpares distintos. Ent


ao
   
q
p
=
.
q
p
o: Sejam p, q, K, g K como acima. Pelo Lema 1.59,
Demonstra
ca

g Z/(q) se e somente se pq = 1. Pelo Lema 1.60, g Z/(q) se e



somente se pq = 1. Assim pq = 1 se e somente se pq = 1, como
queramos.
A Lei da Reciprocidade Quadratica segue diretamente deste u
ltimo
lema:
 
   
p1 
(p1)(q1)
(1) 2
p
q
p q
4
.
=
= (1)
q
p
q
q
p
Podemos tambem considerar este u
ltimo lema como uma reformulacao
da Lei de Reciprocidade Quadratica.

98

CAPITULO 1. POTENCIAS
E CONGRUENCIAS

Problemas Propostos
1.43. Calcular

44
103

60
1019

2010
1019

1.44. Prove que o polin


omio x4 16x2 + 4 e irredutvel em Z[x] mas
n
ao e irredutvel em (Z/(p))[x] para nenhum primo p.
1.45. Sejam p um primo mpar e c um inteiro que n
ao e m
ultiplo de p.
Prove que

p1 
X
a(a + c)
= 1.
p
a=0

1.46. Existem inteiros m e n tais que


5m2 6mn + 7n2 = 1985 ?
1.47. Demonstrar que a congruencia 6x2 + 5x + 1 0 (mod m) tem
soluc
ao para todo valor natural de m.
1.48. Demonstrar que existem infinitos primos da forma 3k +1 e 3k 1.
1.49. Demonstrar que se mdc(a, b) = 1 o n
umero a2 + b2 n
ao pode ter
fatores primos da forma 4k 1 e se alem disso mdc(a, 3) = 1 ent
ao o
n
umero a2 + 3b2 n
ao pode ter fatores da forma 3k 1. Que podemos
dizer sobre os fatores primos de a2 + pb2 onde p e um primo?
1.50. Demonstrar que, para p = 1093,
2

p1
2

1 (mod p2 )

1.51 (IMO1996). Sejam a, b inteiros positivos tais que 15a + 16b e 16a
15b sejam quadrados perfeitos. Encontrar o menor valor que pode tomar
o menor destes quadrados.
1.52. Seja p um n
umero primo mpar. Mostrar que o menor n
ao resto

quadr
atico positivo de p e menor que p + 1.
1.53. Sejam M um n
umero inteiro e p um n
umero primo maior do que

25. Mostrar que a sequencia M, M + 1, , M + 3 p 1 contem um


resto n
ao quadr
atico m
odulo p.
1.54 (Putnam 1991). Seja p um primo mpar. Quantos elementos tem
o conjunto
{x2 | x Z/pZ} {y 2 + 1 | y Z/pZ}?
1.55 (IMO2008). Prove que existe um n
umero infinito de inteirosposi2
tivos n tais que n + 1 tem um divisor primo maior do que 2n + 2n.

Captulo 2

Func
oes Multiplicativas e
as f
ormulas de invers
ao de
M
obius
Neste captulo estudaremos algumas funcoes aritmeticas importantes. Muitas de tais funcoes satisfazem a propriedade de serem determinadas por seus valores em um subconjunto adequado dos n
umeros
naturais, por exemplo os primos, ou as potencias de primo. Uma classe
importante de tais funcoes e a das chamadas func
oes multiplicativas.
Uma funcao f : N C e dita multiplicativa se f (mn) = f (m) + f (n)
para quaisquer m, n primos entre si. No caso que a propriedade anterior
e verdadeira para quaisquer inteiros positivos m e n dizemos que f e
uma func
ao totalmente multiplicativa.
Alguns exemplos de funcoes totalmente multiplicativas s
ao as funcoes
fa (n) = na , onde a e uma constante. De fato, estas funcoes s
ao as
u
nicas que possuem as propriedades de serem monotonas e totalmente
multiplicativas, como mostra a seguinte proposicao
Proposi
c
ao 2.1. Seja f : N R uma func
ao totalmente multiplicativa e mon
otona, ent
ao existe R tal que f (n) = n .
o: Trocando f por 1/f , podemos supor sem perda de
Demonstra
ca
generalidade que f e crescente, e seja = log2 f (2). Vejamos que f (n) =
n . Para isto observemos que, aplicando f , para todo m N temos
2m log2 n nm < 2m log2 n+1

= 2m log2 n (f (n))m < 2(m log2 n+1)


Assim,
2

m log2 n
m

f (n) < 2

(m log2 n+1)
m

99

para todo m N .

100CAPITULO 2. FUNC
OES
MULTIPLICATIVAS E AS FORMULAS
DE INVERSA
Mas
lim

(m log2 n + 1)
m log2 n
= lim
= log2 n,
m
m
m

donde conclumos que f (n) = 2 log2 n = n .


Para uma extensao desse resultado para funcoes multiplicativas veja
o exerccio 2.27
Uma funcao multiplicativa que nao e totalmente multiplicativa e por
exemplo f (n) = 2(n) , onde (n) e o n
umero de divisores primos de n.
O seguinte teorema nos mostra uma forma usual de construir funcoes
multiplicativas.
Teorema 2.2. Se f e uma func
ao multiplicativa ent
ao a func
ao
X
f (d)
F (n) =
d|n

e tambem multiplicativa.
o: Sejam a e b inteiros tais que mdc(a, b) = 1 entao
Demonstra
ca
X
X
X
f (d1 )f (d2 )
f (d1 d2 ) =
f (d) =
F (ab) =
d1 |a,d2 |b

d|ab

XX

f (d1 )f (d2 ) =

d1 |a d2 |b

X
d1 |a

d1 |a,d2 |b

f (d1 )

f (d2 )

d2 |b

= F (a)F (b).
Segue que F tambem e multiplicativa.
Exemplo 2.3. Seja f : N N tal que f (1) = 1 e para todo n > 1,
f (n) = p1 pk onde {p1 , . . . , pk } e o conjunto dos primos que dividem
n, isto e, f (n) e o maior inteiro livre de quadrados
P que divide n. Mostre
que f e uma func
ao multiplicativa e calcule
d|n f (d) em termos da
fatorac
ao prima de n.
o: Observe que se n = ab onde mdc(a, b) = 1 e a, b > 1 , entao
Solu
ca
l+1
ps s onde pi 6= pj para todo i 6= j, logo
a = p1 1 pll e b = pl+1
f (a) = p1 pl , f (b) = pl+1 ps e
f (ab) = p1 pl pl+1 ps = f (a)f (b)
e portanto a funcao e multiplicativa. Para a segunda parte do problema,
pelo teorema anterior basta calcular a soma para potencias de primos.
Temos
k vezes
X
z }| {
f (d) = 1 + p + + p = kp + 1,
d|pk

101
e portanto

X
d|n

f (d) = (1 p1 + 1) (s ps + 1).

Exemplo 2.4. Determine quantas soluc


oes distintas possui a congruencia
x2 800x 0 (mod 10!).
o: Primeiro observemos que m := 10! = 28 34 52 7, assim
Solu
ca
para cada x solucao da equacao inicial, existe r divisor de 10!, tal que x
e solucao do sistema de equacoes lineares
x

0 (mod r)

x 800 (mod

m
).
r

Por outro lado, para cada r|m, temos que o sistema anterior possui
solucao se, e somente se, mdc(r, m
em disso, caso o sistema seja
r )|800. Al
sol
uvel, a solucao e u
nica m
odulo m/ mdc(r, m
a mdc(r, m
r ) (e portanto h
r)
solucoes m
odulo m). Se denotamos por fm (d) o n
umero de inteiros
umero
positivos r divisores de mP
tais que mdc(r, m
r ) = d, temos que o n
de solucoes da equacao e d|800 d.fm (d).
Para calcular fm (d) observe que se d|r e d| m
ao d2 |m, e neste
r ent
2
casso fm (d) = fm/d2 (1), e no caso que d m temos que fm (d) = 0.
Alem disso, fk (1) = 2(k) onde (k) e o n
umero de primos distintos
que dividem k, pois precisamos distribuir os divisores primos de k em
dois fatores r e kr sem que eles possuam fator comum. Usando o fato
anterior temos que o n
umero de solucoes da equacao inicial e
X
X
d.fm (d) =
d.fm/d2 (1)
d2 |800

d|800

d.2(m/d

2)

d|20

= 2(m) + 2.2( 22 ) + 4.2( 42 ) + 5.2( 52 ) + 10.2( 102 ) + 20.2( 202 )


= 24 + 2.24 + 4.24 + 5.23 + 10.23 + 20.23
= 392

Nas seguintes secoes mostraremos algumas funcoes multiplicativas


importantes em aritmetica.

102CAPITULO 2. FUNC
OES
MULTIPLICATIVAS E AS FORMULAS
DE INVERSA

2.1

As func
oes d, e

Para todo inteiro positivo n definimos d(n) como o n


umero de divi
sores positivos de n. E facil calcular d(n) para valores pequenos de n.
Por exemplo d(1) = 1, d(2) = 2, d(3) = 2, d(4) = 3, d(5) = 2, d(6) = 4 e
d(7) = 2. Em particular, para todo n
umero primo p, como seus u
nicos
divisores s
ao 1 e p, temos que d(p) = 2.
Mais geralmente, para toda potencia de um primo pk , vemos que os
u
nicos n
umeros que a dividem s
ao as potencia de p com expoente menor
ou igual a k, isto e, 1, p, p2 , . . . , pk . Assim, d(pk ) = k + 1.
Para calcular d(n) onde n e um n
umero arbitr
ario precisamos do
teorema fundamental da aritmetica, isto e, todo n
umero natural n maior
que 1 pode ser escrito de forma u
nica como produto de primos, por
3
exemplo 91 = 7 17, 1000 = 2 53 , 1001 = 7 11 13 etc. Em geral
todo n
umero n pode ser escrito como
n = pk11 pk22 . . . pks s ,
onde p1 , . . . , ps s
ao n
umeros primos distintos, e k1 , . . . , ks s
ao inteiros
positivos. Agora observemos que se um n
umero m e um divisor de n
entao m nao pode ter em sua fatoracao prima primos distintos aos que
aparecem na fatoracao de n. Alem disso, o expoente de cada primo na
fatoracao de m tem que ser menor ou igual ao expoente deste primo na
fatoracao de n. Assim todo divisor de n e da forma
pl11 pl22 . . . plss
onde 0 lj kj para todo j = 1, . . . , s. Isto quer dizer que l1 pode
asumir qualquer dos valores 0, 1, . . . , k1 , l2 pode asumir qualquer dos
valores 0, 1, . . . , k2 e assim por diante. Assim ha k1 + 1 valores possveis
para l1 , k2 + 1 valores possveis para l2 , e assim por diante, ate ls , que
tem ks + 1 valores possveis. Pelo principio multiplicativo da contagem,
conclumos que
d(n) = (k1 + 1)(k2 + 1) (ks + 1).
Alternativamente, temos d(n) =

d|n 1,

e como a funcao constante igual


k

a 1 e multiplicativa, d e multiplicativa pelo teorema 2.2, e, como d(pj j ) =


kj + 1, d(n) = d(pk11 pk22 . . . pks s ) = (k1 + 1)(k2 + 1) (ks + 1).
Por exemplo, 1000 tem d(1000) = 4 4 = 16 divisores: 1, 2, 4, 5, 8,
10, 20, 25, 40, 50, 100, 125, 200, 250, 500 e 1000.
facil ver que a funcao d, apesar de ser multiplicativa, nao e totalE
mente multiplicativa. Por exemplo, d(4) = 3 6= 2.2 = d(2).d(2).


2.1. AS FUNC
OES
D, E

103

Exemplo 2.5. Determine todos os n


umeros menores que 2012 que tem
exatamente 15 divisores.
o: Dado que 15 = 3 5, temos que os n
Solu
ca
umeros procurados
tem 1 ou 2 fatores primos. No caso em que somente haja um fator
primo p, este primo deve satisfazer 214 p14 < 2012, o que e impossvel.
Assim os n
umeros procurados tem exatamente 2 fatores primos. Isto
e, precisamos encontrar dois n
umeros primos distintosqp e q tais que

2
4
2
p q < 2012, portanto pq < 2012 < 45 e assim q 44
2 < 4. Segue
que q e 2 ou 3. No caso em que q = 2, temos que p 11, donde obtemos
os n
umeros 24 32 = 144, 24 52 = 400, 24 72 = 784, 24 112 = 1936, que
possuem 15 divisores. No caso que q = 3 temos p < 5, logo p = 3, e
obtemos o n
umero 22 34 = 324, que tem 15 divisores.
Uma funcao da mesma natureza que d e a funcao soma dos divisores
de n, que denotaremos por (n). Seguindo as ideias anteriores, temos
que se n = pk11 pk22 . . . pks s , entao a soma dos divisores de n e igual a
X

p1l1 pl22

. . . plss

k1
X

l1 =0

0lj kj
1js

pl11

k2
 X

pl22

l2 =0

ks
X

ls =0


plss .

Como cada somatorio entre chaves e uma serie geometrica, conclumos


que
pk1 +1 1 pk22 +1 1
pks +1 1
(n) = 1

s
.
p1 1
p2 1
ps 1
Em geral, se denotamos por m a soma das m-esimas potencias dos
divisores de n, por um processo identico se prova que
m (n) =

X
d|n

(k1 +1)m

dm =

p1

(k +1)m

(k +1)m

1
ps s
1
1 p2 2

.
m
m
m
p1 1
p2 1
ps 1

Os detalhes da prova da igualdade anterior, assim como a prova do fato


de que cada uma das funcoes m e multiplicativa, s
ao deixados como
exerccio para o leitor.
Exemplo 2.6. Determine um valor de n tal que (n) = 307.
o: Como 307 e primo, entao os u
Solu
ca
nicos possveis valores para n
k+1
s
ao as potencias de primo. Suponhamos que n = pk , assim p p11 = 307.
A equacao anterior e equivalente a p(307 pk ) = 306, assim p e um
divisor primo de 306 = 232 17, isto e, p pode ser 2, 3 ou 17. Observemos
que se p = 2 teramos 2k = 154, e no caso p = 3 obteramos 3k = 205,
que nao geram solucao. No caso p = 17 temos 17k = 289 = 172 , e assim
temos que n = 289 e a u
nica solucao do problema.

104CAPITULO 2. FUNC
OES
MULTIPLICATIVAS E AS FORMULAS
DE INVERSA
Observemos que, para todo n maior que 1, (n) n + 1, e vale a
igualdade unicamente quando n e primo. No seguinte exemplo, encontramos uma limitacao melhor que a anterior, e que relaciona as funcoes
d e .
Exemplo 2.7. Sejam n um n
umero natural maior que 1. Mostrar que

(n) n + 1 + (d(n) 2) n,
e que vale a igualdade se, e somente se, n e primo ou n e o quadrado de
um n
umero primo.
o: Suponhamos que n nao e primo nem quadrado de n
Solu
ca
umero
primo, pois nestes casos se verifica facilmente a igualdade, e vejamos
que neste caso se tem a desigualdade estrita. Observemos que para
todo divisor nao trivial d de n se tem que nd tambem e divisor de n e

d + nd 2 n, onde a desigualdade e estrita se d 6= nd . Assim, somando


sobre todos os divisores nao triviais de n temos que
X
X
X

n
2 n = 2(d(n) 2) n.
>
d+
d=
2
d
d|n
d|n
d|n
d6=1,n

d6=1,n

d6=1,n

Dividindo a desigualdade por 2, e somando n + 1 aos dois lados da


igualdade, obtemos a desigualdade desejada.
Podemos encontrar exemplos em que a soma dos divisores proprios
de um n
umero (isto e, sem incluir ele mesmo), pode ser menor do que
n, maior do que n e igual a n. Os n
umeros cuja soma dos divisores
proprios e n, ou, equivalentemente, com (n) = 2n s
ao chamados de
n
umeros perfeitos, nome que vem desde os gregos antigos. Observemos
que no tempo de Euclides ja eram conhecidos os n
umeros perfeitos
6 = 2 3,

28 = 4 7,

496 = 16 31

e 8128 = 64 127

Todos estes n
umeros s
ao da forma 2k1 (2k 1) onde 2k 1, e primo. De
fato, foi mostrado por Euler que todos os n
umeros perfeitos pares s
ao
desta forma como se mostra a seguir
Proposi
c
ao 2.8. Se n um n
umero par, ent
ao (n) = 2n se, e somente
se, existe um inteiro k tal que Mk := 2k 1 e primo e n = 2k1 Mk .
Os n
umeros Mk s
ao chamados de N
umeros de Mersenne em homenagem ao fil
osofo, te
ologo e matem
atico frances Marin Mersenne que
estudou estes n
umeros, e conjecturou para que valores de k < 257 os
n
umeros eram primos.


2.1. AS FUNC
OES
D, E

105

o: Se Mp = 2p 1 e um n
Demonstra
ca
umero primo entao
(2p1 Mp ) = (2p1 ) (Mp ) = (2p 1)(Mp + 1) = 2 2p1 Mp .
Por outro lado, seja n = 2k b, com k > 0 e b mpar, um n
umero perfeito
par. Temos
(n) = 2n = (2k )(b)

donde 2k+1 b = (2k+1 1)(b).

Como mdc(2k+1 1, 2k+1 ) = 1, temos que 2k+1 1 e um divisor de b,


logo b = (2k+1 1)c para algum inteiro mpar c e assim (b) = 2k+1 c.
Mas 1, 2k+1 1, c, b s
ao divisores de b = (2k+1 1)c; se c 6= 1 entao
2k+1 c = (b) 1 + b + c = 1 + 2k+1 c,
o que e contradit
orio. Logo c = 1 e b = 2k+1 1 e primo pois temos
(b) = 2k+1 = b + 1.
Deixamos para o leitor mostrar que se Mk e primo entao k tem que
ser primo. Na atualidade unicamente s
ao conhecidos 47 valores de k pra
os quais Mk e primo. O maior deles e k = 43112609, descoberto em 2008
(para mais informacoes ver o site http://www.mersenne.org).
Dado um n
umero n, lembramos que (n) denota o n
umero de n
umeros naturais menores que n e primos relativos com n. Por exemplo,
se p e um n
umero primo, entao todo n
umero menor que p nao tem fator
comum com p, e assim (p) = p 1. Em geral, (pk ) = pk pk1
pois mdc(a, pk ) = 1, se e somente se, a nao e m
ultiplo de p e ha pk1
m
ultiplos de p no intervalo 1 a pk . Sabemos que a funcao e
multiplicativa, isto e, (nm) = (n)(m) para quaisquer m e n primos
entre si
Assim, se n = p1 1 pk k e a fatoracao de n em potencias de primos
distintos pi , temos que

Y
Y
Y 
1
i
i 1
i
.
(n) =
(pi ) =
(pi pi
)=n
1
pi
1ik

1ik

1ik

Exemplo 2.9. Determine todas as soluc


oes de (n) = 27 .
o: Suponhamos que n = 2a m onde m = p1 1 pk k e mpar.
Solu
ca
Temos (n) = (2a )(m), e (m) = p1 1 1 pk k 1 (p1 1) (pk 1),
mas como (n) e uma potencia de 2, segue que j = 1 e pj 1 e
uma potencia de 2 para todo j. Conclumos que os possveis primos na
fatoracao de m s
ao 21 + 1 = 3, 22 + 1 = 5, 24 + 1 = 17 (o proximo
primo da forma 2n + 1 e 257 = 28 + 1 > 27 + 1). Portanto os possveis
valores de n s
ao 28 = 256, 27 3 = 384, 26 5 = 320, 25 15 = 480,
4
2 17 = 272, 23 51 = 408, 22 85 = 340, 2 255 = 510 e 255.

106CAPITULO 2. FUNC
OES
MULTIPLICATIVAS E AS FORMULAS
DE INVERSA
Exemplo 2.10. Mostrar que (n)

log 2
log(2n) n

para todo n > 2

o: Seja n = p1 1 pk k e a fatoracao de n. Pela identidade


Solu
ca
anterior temos que


Y 
Y 
1
1
1
(n)
=
1
=
.

1
n
pi
i
k+1
1ik

2ik+1

Como n p1 p2 pk 2k , assim 2n 2k+1 e portanto


como queramos mostrar.

1
k+1

log 2
log(2n) ,

Exemplo 2.11. Encontrar todos os inteiros n para os quais (n) =


d(n).
o: Se p 3 e um primo, temos que
Solu
ca


(p ) = (p 1)p1 2(1 + 2)1 2 1 + 2( 1) + 1 = d(p ),

onde a igualdade s
o se da quando p = 3 e = 1. Portanto, pela
multiplicatividade das funcoes (n) e d(n), os u
nicos mpares que satisfazem (n) = d(n) s
ao n = 1 e n = 3. Por outro lado, se > 3
temos (2 ) = 21 > + 1 = d(2 ); para = 3 obtemos as solucoes
n = 1 8 = 8 e n = 3 8 = 24.
Assim, s
o nos falta resolver os casos (2n) = d(2n) (n) =
2d(n) e (4n) = d(4n) 2(n) = 3d(n) onde n e mpar. Temos
(5) = 4 = 2d(5), (15) = 8 = 2d(15) e (9) = 6 = 2d(9), donde
2 5 = 10, 2 9 = 18 e 2 15 = 30 tambem s
ao solucoes da equacao
inicial. Demonstremos agora que nao existem mais solucoes. Se n = p
e potencia de um primo mpar p entao para p = 3 e 3, ou para para
p = 5 e 2, ou para p 7, temos como acima que
3
(n) = p1 (p 1) > 2 + 2 = 2d(n) > d(n).
2
Por outro lado, ja sabemos que (n) d(n) para todo n mpar. Assim,
da multiplicatividade das funcoes (n) e d(n), obtemos que se n e divisvel por 33 , 52 ou por algum primo p 7, entao (n) > 2d(n) > 23 d(n)
e analisando os casos restantes obtemos apenas as solucoes apresentadas
anteriormente.
Em conclusao, as u
nicas solucoes de (n) = d(n) s
ao 1, 3, 8, 10, 18,
24 e 30.
A seguinte proposicao relaciona as funcoes e .
Proposi
c
ao 2.12. Seja n um n
umero inteiro maior que 1. Mostrar que
(n)(n) < n2 .


2.1. AS FUNC
OES
D, E

107

o: Seja n = pk11 pk22 . . . pks s a fatoracao em primos de n.


Demonstra
ca
Observemos que
(n) =
<
=
=

pk11 +1 1 pk22 +1 1
pks +1 1

s
p1 1
p2 1
ps 1
pk11 +1 pk22 +1 pks s +1
(p1 1)(p2 1) (ps 1)
np1 p2 ps
(p1 1)(p2 1) (ps 1)
n2
,
(n)

como queramos mostrar.


Podemos usar a proposicao anterior para mostrar o seguinte resultado sobre n
umeros perfeitos e sobre os n
umeros chamados de abundantes, que s
ao os n
umeros menores que a soma dos seus divisores proprios,
isto e, com (n) > 2n.
Exemplo 2.13. Seja n um n
umero mpar perfeito ou abundante. Ent
ao
n tem 3 ou mais fatores primos distintos.
o: Pela proposicao anterior temos que
Solu
ca
2n(n) (n)(n) < n2 ,
assim, dividindo por 2n2 obtemos a desigualdade


 

(n)
1
1
1
1
= 1
1
1
.
n
p1
p2
ps
2
Observe que se
umero
n tiver dois o menos fatores primos, entao
 o n

(n)
1
1
8
1 5 = 15
> 12 , o que e contradit
orio. Portanto, n
n 1 3
tem tres ou mais fatores primos.
Ate o momento nao e conhecido nenhum n
umero perfeito mpar. Se
conjectura que eles nao existam, mas caso existam, eles s
ao maiores que
300
10
e tem no mnimo 9 fatores primos distintos.
Existem algumas condicoes necessarias para que um n
umero mpar
seja perfeito (caso exista algum). O seguinte resultado e devido a Euler
Proposi
c
ao 2.14. Seja n um n
umero perfeito mpar. Ent
ao existem p
um n
umero primo, k e m inteiros tais que n = pk m2 , onde p e k s
ao
congruentes a 1 m
odulo 4.

108CAPITULO 2. FUNC
OES
MULTIPLICATIVAS E AS FORMULAS
DE INVERSA
o: Seja n = pk11 pks s a fatoracao prima de n. Temos
Demonstra
ca
2n = (n) = (pk11 ) (pks s ).
k

Assim, somente um dos n


umeros (pj j ) com 1 j s pode ser par.
Podemos supor que (pk11 ) e par (mas nao divisvel por 4), e os outros
k
n
umeros (pj j ) com 2 j s s
ao mpares. Como
k

(pj j ) = 1 + pj + p2j + + pj j kj + 1 (mod 2),


temos que kj tem que ser par para todo j 2 assim pk22 pks s = m2 .
Observemos que m2 1 (mod 4) logo (pk11 ) 2 (mod 4), e portanto
k1 e mpar. No caso que p1 3 (mod 4) teramos que (pk11 ) 0
(mod 4), o que e contradit
orio, daqui que p1 1 (mod 4) e
(pk11 ) k1 + 1 2 (mod 4),
o que conclui a prova.

Problemas Propostos
2.1. Calcule (1001) e (1001).
2.2. Determine o n
umero de divisores de 2008, 2009, 2010 e 2011.
2.3. Determine as soluc
oes de (n) = 2801.
2.4. Mostre que as func
oes m s
ao multiplicativas para todo m 6= 0.
Observe que para todo n > 1 se tem que lim m (n) = d(n).
m0

2.5. Determine as soluc


oes do sistema

(n) = 8784
d(n) = 12

2.6. Seja f uma func


ao multiplicativa tal que para todo n
umero primo
p,
(
1 se k = 2s para algum s N
k
f (p ) =
.
0 caso contr
ario
Mostre que f e uma func
ao tal que f (n2 ) = f (n)2 para todo n N, mas
n
ao e totalmente multiplicativa.
2.7. Construa um exemplo de uma func
ao multiplicativa f , que n
ao
k
k
seja totalmente multiplicativa, tal que f (n ) = f (n) para todo k 10
e todo n N. Generalize o resultado anterior para k N (em lugar de
k 10), com N inteiro fixo.


2.1. AS FUNC
OES
D, E

109

2.8. Mostrar que para todo inteiro positivo n 5 temos que


n

d2 (n)
+ (n).
4

Para que valores de n se tem a igualdade?


2.9. Quantos n
umeros com 35 divisores existem, tais que todos seus
fatores primos sejam menores que 20. Qual e o maior e o menor de tais
n
umeros?
2.10. Seja n um n
umero composto. Mostre que 2n 1 tambem e composto.
2.11. Mostre que um n
umero n tem um n
umero mpar de divisores se,
e somente se, n e um quadrado perfeito.
2.12. Determine a menor soluc
ao da equac
ao 2d(n2 ) = 17d(n).
2.13. Encontre infinitos valores de n para os quais d(n) e um divisor de
n.
2.14. Denotemos por f (n) a soma dos divisores de n que s
ao quadrados
perfeitos. Mostre que f e uma func
ao multiplicativa, e determine uma
f
ormula fechada para esta func
ao.
2.15. Determine todas as soluc
oes de (n) = 24.
2.16. Denotemos por n# o produto de todos os primos menores ou iguais
a n. Mostre que (n#) divide n! e usando este resultado mostre que a
equac
ao (x) = n! sempre possui soluc
ao x N.
2.17. Determine todos os valores de k N tais que n = k(n) possui
soluc
ao.
2.18. Mostre que para todo n existe um k tal que (x) = kn possui
soluc
ao.
2.19. Determinar todas as soluc
oes da equac
ao (n) = 2d(n).
2.20. Determinar todos os n
umeros inteiros positivos n tais que
2
n = d(n) .
2.21. Dois n
umeros a e b s
ao amig
aveis se (a) = b e (b) = a. Por
exemplo 1184 e 1210 s
ao amig
aveis (verificar!). Encontrar outra dupla
de n
umeros amig
aveis.
2.22. Determine todas as soluc
oes de ((n)) = 213 32 54 .

110CAPITULO 2. FUNC
OES
MULTIPLICATIVAS E AS FORMULAS
DE INVERSA
2.23. Mostre que para todo m e n inteiros vale a identidade
(mn) (mdc(m, n)) = mdc(m, n) (n) (m).
2.24. Mostre que 945 e o menor n
umero abundante mpar.
2.25. Mostre que todo m
ultiplo de um n
umero abundante e abundante.
2.26. Determine qual e o n
umero mnimo de divisores primos que um
n
umero deve ter se ele e
abundante ou perfeito,
n
ao divisvel por 2, nem por 3.
2.27. Seja f : N+ R+ uma func
ao multiplicativa e crescente.
(a) Prove que, para todo inteiro M > 1 e todo inteiro positivo n,
f (M n+1 1) f (M n 1)f (M ) e f (M n+1 + 1) f (M n + 1)f (M ).
Conclua que
lim

p
n
f (M n ) = f (M ).

(b) Utilize o item anterior para M potencia de primo para concluir que
f (pk ) = f (p)k para todo primo p.
(c) Conclua que f e totalmente multiplicativa, e portanto existe > 0
tal que f (n) = n para todo inteiro positivo n.

2.2

Func
ao de M
obius e F
ormula de Invers
ao

Definimos a funcao de M
obius : N>0 Z por

se n = 1
1
(n) = 0
se a2 | n para algum a > 1

(1)k se n e produto de k primos distintos.

Facilmente se comprova que a funcao de M


obius e multiplicativa. Alem
disso
Lema 2.15. Para todo inteiro positivo n temos
(
X
1 se n = 1
(d) =
0 se n > 1.
d|n

DE MOBIUS

2.2. FUNC
AO
E FORMULA
DE INVERSAO

111

Demonstra
P cao: No caso n = 1 nao temos nada para provar. Como a
funcao d|n (d) e multiplicativa pelo teorema 2.2, basta mostra o lema
para n = pk onde p e um n
umero primo. De fato,
X

(d) =

k
X
j=0

d|pk

(pj ) = 1 1 = 0

como queramos demonstrar.


Teorema 2.16 (F
ormula de inversaoPde Mobius). Seja f (n) uma func
ao
ao para todo n inteiro
sobre os inteiros positivos e F (n) = d|n f (d), ent
positivo,
n
X
.
f (n) =
(d)F
d
d|n

o: Vejamos que
Demonstra
ca
n X
X
X
(d)F
(d)
=
f (d1 )
d
n
d|n
d1 | d
d|n
XX
(d)f (d1 )
=
d|n d1 | n
d

XX

d1 |n

(d)f (d1 )

d| dn
1

f (d1 )

(d) = f (n)(1) = f (n),

d| dn

d1 |n

como queramos demonstrar.


P
Exemplo 2.17. Como vimos no lema 1.36, d|n (d) = n, n N.
Portanto, pelo teorema anterior, para todo inteiro positivo n, (n) =
P
P
(n)
n
segue que d|n (d)
d|n (d) d . Da
d = n .

Exemplo 2.18. Uma pulseira e formada por pedras coloridas, de mesmo


tamanho, pregadas em volta de um crculo de modo a ficarem igualmente
espacadas. Duas pulseiras s
ao consideradas iguais se, e s
o se, suas configurac
oes de pedras coincidem por uma rotac
ao. Se h
a pedras disponveis
de k 1 cores distintas, mostre que o n
umero de pulseiras diferentes
possveis com n pedras e dado pela express
ao
1X
(d) k n/d .
n
d|n

112CAPITULO 2. FUNC
OES
MULTIPLICATIVAS E AS FORMULAS
DE INVERSA
o: No que segue o n
Solu
ca
umero k de cores de pedras estara sempre
fixo. A cada pulseira podemos associar um perodo, que e definido como o
menor divisor positivo d de n tal que a sequencia das n pedras da pulseira
e obtida a partir de uma sequencia de d pedras repetida n/d vezes.
Se o problema fosse contar pulseiras fixas, sem indentificar pulseiras
que coincidem por uma rotacao, a resposta seria claramente k n . Ao
considerarmos as n rotacoes de uma pulseira de perodo d, obtemos d
pulseiras fixas distintas (i.e., distintas como pulseiras fixas, mas iguais a
menos de rotacao). Dizemos que uma pulseira com n pedras e primitiva
se seu perodo e n. Se denotarmos por g(n) o n
umero de pulseiras
primitivas com n pedras, temos que, para cada divisor d de n, o n
umero
de pulseiras com n pedras e perodo d e g(d) (se o perodo e d, podemos
tomar d pedras consecutivas e unir as pontas criando uma pulseira com
d pedras, que sera primitiva), e elas dao origem
fixas.
P a d.g(d) pulseiras
n , donde,
Assim, temos, para todo inteiro positivo n,
d.g(d)
=
k
d|n
P
pelo teorema anterior, n.g(n) = d|n (d)k n/d .
O n
umero de pulseiras que queremos contar, como no enunciado, e
X1X
X
g(d) =
(s)k d/s .
d
d|n

d|n

s|d

Fazendo t = d/s na u
ltima expressao, temos d = st, e d | n equivale a
s | n/t. Assim, podemos escrever a u
ltima expressao como
XX 1
X k t X (s)
(s)k t =
,
st
t
s
t|n s|n/t

t|n

s|n/t

P
t
que, pelo exemplo anterior, e igual a t|n t nt (n/t) = t|n kn (n/t),
P
que, por sua vez (fazendo r = n/t), e igual a n1 r|n (r) k n/r .
P

kt

Agora, observemos que para todo n


umero natural m, f e F definidas
como antes,
m
X

F (n) =

f (d) =

n=1 d|n

n=1

Como f (d) e somado

m X
X

m
d
m
X

n=1

m
X
X
d=1

f (d)

d|n
1nm

vezes, entao
F (n) =

m
X

f (d)

d=1

jmk
d

No caso particular em que f (n) = (n) temos que F (n) = n pelo


lema 1.36 e assim
m
jmk
m(m + 1) X
(n)
=
.
2
n
n=1

DE MOBIUS

2.2. FUNC
AO
E FORMULA
DE INVERSAO

113

Se f (n) = (n), entao F (n) = 0 se n > 1 e F (1) = 1 pelo lema 2.15,


portanto
m
jmk
X
.
(n)
1=
n
n=1

A igualdade anterior nos permite resolver o seguinte

Exemplo 2.19. Demonstrar que, para todo inteiro m > 1,




m
X
(k)


< 1.

k
k=1

m
  m
o: Como 1 < (k) m
Solu
ca
k k < 1 e
k
k = 1, m, entao


m
m
X
jmk
X
(k)

m
(k)

<m1

k
k

m
k

= 0 quando

k=1

k=1

Usando a identidade acima provada temos que




m

X
(k)

< m 1,
1 m

k
k=1

P


(k)
logo m m
amos
k=1 k < m e simplificando m obtemos o que quer
conhecido (Mangoldt 1897) que se m tende para infinito,
demonstrar. E
entao a soma anterior converge para 0.
Teorema 2.20 (Segunda formula de inversao de Mobius). Sejam f, g
func
oes reais com domnio (0, +) tais que

x
X
f
g(x) =
k
k=1

para todo x, ent


ao
f (x) =

(k)g

k=1

x
k

o: Observemos que
Demonstra
ca
f (x) =

X
k=1

(k)

X
 x  X
X
 x
f
=
= f (x),
(k) f
kr
m
r=1

como queramos demonstrar.

m=1 k|m

114CAPITULO 2. FUNC
OES
MULTIPLICATIVAS E AS FORMULAS
DE INVERSA
A seguinte e uma das formulacoes da famosa hip
otese de Riemann,
um dos problemas em aberto mais importantes da Matem
atica. O Clay
Mathematics Institute oferece um premio de 1 milh
ao de dolares para
a a primeira demonstracao da Hipotese de Riemann (ver a p
agina web
http://www.claymath.org/millennium/).
Conjetura 2.21 (Hipotese de Riemann). Se > 1/2, ent
ao
n
1 X
(m) = 0.
n n

lim

m=1

Podemos reenunciar esta conjectura assim: seja f : (0, +) R


definida por
(
f (t) = 0
se t < 1
P
k=1 f (t/k) = 1 se t 1.
Entao, para todo > 1/2,

lim

f (t)
= 0.
t

De fato, pela segunda formula de inversao de Mobius, temos


f (t) =

t
X

(m).

m=1

Problemas Propostos
2.28. Encontre formulas fechadas para as somas
X
(r)d(n/r)

r|n

(d)(n/d)

d|n

(d)m (n/d)

d|n

2.29. Seja f uma func


ao multiplicativa e n
ao identicamente nula e n =
p1 1 pk k . Mostre que
X
d|n

(d)f (d) =

k
Y

(1 f (pj )).

j=1

DE MOBIUS

2.2. FUNC
AO
E FORMULA
DE INVERSAO

115

2.30. Encontre formulas fechadas para as somas


X
(r)d(r)

r|n

X (d)

(d)(d)

d|n

(d)(d)

d|n

d|n

2.31. Seja r o n
umero de fatores primos diferentes de n, demonstrar
que
X
|(d)| = 2r .
d|n

2.32. Seja n um inteiro positivo que n


ao e primo e tal que (n) | n 1.
Demonstrar que n possui ao menos quatro fatores primos distintos.
2.33. Dados dois n
umeros reais e tais que 0 < 1, demonstrar que existe um n
umero natural m tal que
<

(m)
< .
m

2.34. Seja m um inteiro positivo. Dizemos que um inteiro m 1 e


superabundante se
k {1, 2, . . . , m 1}

(k)
(m)
>
.
m
k

Demonstrar que existe um n


umero infinito de n
umeros superabundantes.
2.35. Demonstrar que

(n)
n.
d(n)

2.36. Encontrar todos os valores de n para os quais (n) | n.


2.37. Demonstrar que m | (mn 1) para todo n se, e s
o se, m = 2, 3,
4, 6, 8, 12 ou 24.
2.38. Demonstrar que
1
1
(n!)
> 1 + + + .
n!
2
n

116CAPITULO 2. FUNC
OES
MULTIPLICATIVAS E AS FORMULAS
DE INVERSA
2.39. Demonstrar que existem infinitos n
umeros naturais n para os
quais (x) = n n
ao tem soluc
ao.
2.40. Demonstrar que para todo m > 1
m

X (k) 2



< .

k 3
k=1

2.41. Encontrar todos os inteiros positivos n tais que


n = d26 + d27 1,
onde 1 = d1 < d2 < < dk = n s
ao todos os divisores positivos do
n
umero n.
2.42 (IMO1998). Para cada inteiro positivo n, d(n) denota o n
umero de
divisores de n. Determine todos os inteiros positivos k tais que d(n2 ) =
kd(n) para algum n.

2.43. Se n e composto, mostre que (n) n n.


2.44. Mostrar que (n) + (n) 2n para todo inteiro positivo n.
2.45. Seja f : (0, +) R tal que f (x) = 0 se x (0, 1) e f (x) =
P
P
x kx (k)
k1 f (x/k) = x, x 1.
k , x 1. Prove que

2.46. Demonstrar que

(n!)
1
1
> 1 + + + .
n!
2
n
2.47. Dadas duas func
oes f, g : N>0 C, definimos o produto de Dirichlet (ou convoluc
ao de Dirichlet) f g : N>0 C de f e g por
n
X
X
def
f (d)g
f g(n) =
f (d1 )g(d2 ).
=
d
d1 d2 =n

d|n

(a) Prove que, se s R (ou s C) e as series

n1

convergem absolutamente ent


ao
X f (n) X g(n) X f g(n)

=
.
ns
ns
ns
n1

n1

f (n)
ns

n1

g(n)
ns

n1

(b) Prove que, para quaisquer func


oes f, g, h : N>0 C, temos f g =
g f e f (g h) = (f g) h (isto e, o produto de Dirichlet
e comutativo(e associativo), e que a func
ao I : N>0 C dada
1 se n = 1
por I(n) =
e o elemento neutro do produto , i.e.,
0 se n > 1
I f = f I = f , f : N>0 C.

DE MOBIUS

2.2. FUNC
AO
E FORMULA
DE INVERSAO

117

(c) Prove que se f e g s


ao multiplicativas ent
ao f g e multiplicativa.
(d) Prove que, se f : N>0 C e tal que f (1) 6= 0, ent
ao existe uma
(1)
(1)
(1)
u
nica func
ao f
: N>0 C tal que f f
=f
f = I, a
qual e dada recursivamente por f (1) (1) = 1/f (1) e, para n > 1,
f (1) (n) =

X n
1
f
f (1) (d).
f (1)
d
d|n,d<n

(e) Prove que, se f e multiplicativa, ent


ao a func
ao f (1) definida no
item anterior tambem e multiplicativa.

Captulo 3

Fra
co
es Contnuas
A teoria de fracoes contnuas e um dos mais belos assuntos da Matematica elementar, sendo ainda hoje tema de pesquisa.
Nas inclus
oes N Z Q R, a passagem de Q para R e sem d
uvida
a mais complicada conceitualmente e a representacao de um n
umero real
esta diretamente ligada `
a propria nocao de n
umero real.
De fato, o conceito de n
umero natural e quase um conceito primitivo.
Ja um n
umero inteiro e um n
umero natural com um sinal que pode ser
+ ou , e um n
umero racional e a razao entre um n
umero inteiro e
um natural nao nulo. Por outro lado, dizer o que e um n
umero real e
tarefa bem mais complicada, mas ha coisas que podemos dizer sobre eles.
Uma propriedade essencial de R e que todo n
umero real pode ser bem
aproximado por n
umeros racionais. Efetivamente, dado x R, existe
k = x Z tal que 0 x k < 1. Podemos escrever a representacao
decimal de
x k = 0, a1 a2 . . . an . . . ,

ai {0, 1, . . . , 9},

o que significa que se rn = an + 10 an1 + 100 an2 + + 10n1 a1 ,


rn
rn +1
rn
entao 10
e uma boa aproximacao
n x k < 10n , e portanto k + 10n
rn

racional de x, no sentido de que o erro x k + 10
e menor do que
n
1
,
que

e
um
n
u
mero
bem
pequeno
se
n
for
grande.
A
representacao
n
10
decimal de um n
umero real fornece pois uma sequencia de aproximacoes
por racionais cujos denominadores s
ao potencias de 10.
Dado qualquer x R e q natural nao nulo existe p Z tal que


p
x < p+1
(basta tomar p = qx), e portanto x pq < 1q e
q
q


p+1
x q 1q . Em particular ha aproximacoes de x por racionais com
denominador q com erro menor do que 1q . A representacao decimal de x
equivale a dar essas aproximacoes para os denominadores q que s
ao potencias de 10, e tem meritos como sua praticidade para efetuar calculos

119
que a fazem a mais popular das representacoes dos n
umeros reais. Por
outro lado, envolve a escolha arbitr
aria da base 10, e oculta frequentemente aproximacoes racionais de x muito mais eficientes do que as que
exibe. Por exemplo, = 3, 141592653589793... e excepcionalmente bem
355
aproximado por 22
7 = 3, 142857142857... e 113 = 3, 14159292035398....
De fato,














355
1
1
314
3141592
22
e
<

<
<
<





7
700
100
113
3000000
1000000

355
mostram que 22
ao melhores aproximacoes de que aproximacoes
7 e 113 s
decimais com denominadores muito maiores.

O objetivo desta secao e apresentar uma outra maneira de representar n


umeros reais, a representacao por frac
oes contnuas, que sempre
fornece aproximacoes racionais surpreendentemente boas, e de fato fornece todas as aproximacoes excepcionalmente boas, alem de ser natural
e conceitualmente simples.
Definimos recursivamente
an = n
1
para todo n N.
=
n a n

0 = x,
e, se n
/ Z,

n+1

Se, para algum n, n = an temos


1

def

x = 0 = [a0 ; a1 , a2 , . . . , an ] = a0 +
a1 +

1
a2 + . .
.

1
an

Se nao denotamos
def

x = [a0 ; a1 , a2 , . . . ] = a0 +

1
1
a1 +
a2 + . .
.

O sentido dessa u
ltima notacao ficara claro mais tarde. A representacao
acima se chama representac
ao por frac
oes contnuas de x.


CAPITULO 3. FRAC
OES
CONTINUAS

120

A figura da uma interpretacao geometrica para a representacao de


um n
umero por fracoes contnuas. Enchemos um ret
angulo 1 x com
quadrados de forma gulosa, isto e, sempre colocando o maior quadrado
possvel dentro do espaco ainda livre. Os coeficientes a0 , a1 , a2 , . . . indicam o n
umero de quadrados de cada tamanho. Na figura, se os lados do
retangulo s
ao c < d entao
d/c = [1; 2, 2, 1, ...]
pois temos a0 = 1 quadrado grande, a1 = 2 quadrados menores, a2 = 2
quadrados ainda menores, a3 = 1 quadrados ainda ainda menores, e um
n
umero grande nao desenhado de quadrados ainda ainda ainda menores
(a4 e grande). Deixamos a verificacao de que esta descricao geometrica
corresponde `a descricao algebrica acima a cargo do leitor.
Note que, se a representacao por fracoes contnuas de x for finita
entao x e claramente racional.
Reciprocamente, se x Q, sua representacao sera finita, e seus coeficientes an vem do algoritmo de Euclides: se x = p/q (com q > 0)
temos
p = a 0 q + r1
q = a 1 r1 + r2
r1 = a 2 r2 + r3
..
.

0 r1 < q

0 r2 < r1

0 r3 < r2
..
.

rn1 = an rn
Temos entao
x = p/q = a0 + r1 /q = a0 +

1
= a0 +
a1 + r2 /r1

1
a1 +

1
a2 + r3 /r2

121
= = a0 +

1
a1 +

= [a0 ; a1 , a2 , . . . , an ].

1
a2 + . .
.

1
an
Isso ja e uma vantagem da representacao por fracoes contnuas (alem
de nao depender de escolhas artificiais de base), pois o reconhecimento
de racionais e mais simples que na representacao decimal.
A representacao decimal de n
umeros reais esta intimamente ligada
`a funcao f : [0, 1) [0, 1) dada por f (x) = {10x} = 10x 10x, mais
precisamente, `
a din
amica da funcao f . Por din
amica da funcao f queremos dizer o estudo de suas composicoes sucessivas: para cada ponto
x [0, 1), estamos interessados na sequencia x, f (x), f (f (x)), [0, 1),
cujos termos s
ao os chamados iterados sucessivos da f . De fato, se
x [0, 1) tem representacao decimal 0, a1 a2 a3 . . . , entao a1 = 10x e
f (x) = 0, a2 a3 a4 . . . . Assim, definindo f 1 = f e f n+1 = f f n , temos
f n (x) = 0, an+1 an+2 an+3 . . . para todo n 1. Assim, por exemplo, se
x = 1/3 = 0, 333 . . . , temos f (x) = 0, 333 = x (nesse caso, dizemos que x = 1/3 e um ponto fixo de f ); se x = 4/33 = 0, 121212 . . . ,
temos f (x) = 0, 212121 . . . e f (f (x)) = 0, 121212 = x (nesse caso
dizemos que x = 4/33 e um ponto peri
odico de perodo 2 de f ) e, se
x [0, 1] e irracional, os seus iterados por f serao todos distintos, pois
sua representacao decimal nao sera peri
odica a partir de nenhum dgito.
Ja a representacao em fracoes contnuas esta intimamenteligada

`a din
amica
da funcao g : (0, 1) [0, 1), dada por g(x) = x1 =


1
1
em conhecida como transformac
ao de Gauss: se =
x x , tamb
1
[0; a1 , a2 , a3 , . . . ] (0, 1), entao a1 = e g() = [0; a2 , a3 , a4 , . . . ].
Assim, definindo, como antes g 1 = g e g n+1 = g g n para todo n 1,
temos g n () = [0; an+1 , an+2 , an+3 , . . . ], para todo n 1.
Mais informacoes sobre a relacao entre fracoes contnuas e a din
amica
da transformacao de Gauss pode ser encontrada em [6].
Representamos abaixo os gr
aficos de f (x) = {10x} e g(x) = { x1 }.
+

y = g(x) =
y = f (x) = {10x}

1
x


CAPITULO 3. FRAC
OES
CONTINUAS

122

Seja x = [a0 ; a1 , a2 , . . . ]. Sejam pn Z, qn N>0 primos entre si


tais que pqnn = [a0 ; a1 , a2 , . . . , an ], n 0. Esta fracao pqnn e chamada de
n-esima reduzida ou convergente da fracao contnua de x. O seguinte
resultado sera fundamental no que seguir
a.
Proposi
c
ao 3.1. Dada uma sequencia (finita ou infinita) t0 , t1 , t2 ,
R tal que tk > 0, para todo k 1, definimos sequencias (xm ) e (ym ) por
x0 = t0 , y0 = 1, x1 = t0 t1 + 1, y1 = t1 , xm+2 = tm+2 xm+1 + xm ,
ym+2 = tm+2 ym+1 + ym , para todo m 0. Temos ent
ao
1

[t0 ; t1 , t2 , . . . , tn ] = t0 +
t1 +

1
t2 + . .
.

xn
, n 0.
yn

1
tn

Alem disso, xn+1 yn xn yn+1 = (1)n , para todo n 0.

o: A prova sera por inducao em n. Para n = 0 temos


Demonstra
ca
[t0 ] = t0 = t0 /1 = x0 /y0 . Para n = 1, temos [t0 ; t1 ] = t0 + 1/t1 =
t0 t1 +1
= x1 /y1 e, para n = 2, temos
t1
[t0 ; t1 , t2 ] = t0 +

1
t2
t0 t1 t2 + t0 + t2
= t0 +
=
t1 + 1/t2
t1 t2 + 1
t1 t2 + 1

t2 x 1 + x 0
x2
t2 (t0 t1 + 1) + t0
=
=
.
t2 t1 + 1
t 2 y1 + y0
y2
Suponha que a afirmacao seja valida para n. Para n + 1 em lugar de n
temos
1
]
[t0 ; t1 , t2 , . . . , tn , tn+1 ] = [t0 ; t1 , t2 , . . . , tn +
tn+1

1
tn + tn+1
xn1 + xn2

=
1
tn + tn+1
yn1 + yn2
=

tn+1 (tn xn1 + xn2 ) + xn1


tn+1 (tn yn1 + yn2 ) + yn1
tn+1 xn + xn1
xn+1
=
=

tn+1 yn + yn1
yn+1

Vamos agora mostrar, por inducao, a segunda afirmacao. Temos


x1 y0 x0 y1 = (t0 t1 + 1) t0 t1 = 1 = (1)0

e, se xn+1 yn xn yn+1 = (1)n para algum valor de n, entao


xn+2 yn+1 xn+1 yn+2 = (tn+2 xn+1 + xn )yn+1 (tn+2 yn+1 + yn )xn+1
= (xn+1 yn xn yn+1 ) = (1)n = (1)n+1 .

123
Nos proximos resultados, x = [a0 ; a1 , a2 , a3 , . . . ] sera um n
umero
real, e ( pqnn )nN , pqnn = [a0 ; a1 , a2 , . . . , an ] sera a sequencia de reduzidas da
fracao contnua de x.
Corol
ario 3.2. As sequencias (pn ) e (qn ) satisfazem as recorrencias
pn+2 = an+2 pn+1 + pn

qn+2 = an+2 qn+1 + qn

para todo n 0, com p0 = a0 , p1 = a0 a1 + 1, q0 = 1 e q1 = a1 . Alem


disso,
pn+1 qn pn qn+1 = (1)n
para todo n 0.
o: As sequencias (pn ) e (qn ) definidas pelas recorrencias
Demonstra
ca
acima satisfazem, pela proposicao anterior, as igualdades
pn
= [a0 ; a1 , a2 , . . . , an ] e pn+1 qn pn qn+1 = (1)n , n 0.
qn
Como pn+1 qn pn qn+1 = (1)n , para todo n N, temos que os pn , qn
dados pelas recorrencias acima s
ao primos entre si. Alem disso, tambem
segue da recorrencia que qn > 0, n 0. Esses fatos implicam que
( pqnn )nN e a sequencia de reduzidas da fracao contnua de x.
Corol
ario 3.3. Temos, para todo n N,
x=

n pn1 + pn2
n qn1 + qn2

n =

pn2 qn2 x
qn1 x pn1

o: A primeira igualdade segue da proposicao anterior


Demonstra
ca
pois x = [a0 ; a1 , a2 , . . . , an1 , n ] e a segunda e consequencia direta da
primeira.
Proposi
c
ao 3.4. Temos
x
onde
n+1 =

(1)n
pn
=
qn
(n+1 + n+1 )qn2

qn1
= [0; an , an1 , an2 , . . . , a1 ].
qn

Em particular,



1
pn
1
1

< x =
<
.
2
2
(an+1 + 2)qn
qn
(n+1 + n+1 )qn
an+1 qn2


CAPITULO 3. FRAC
OES
CONTINUAS

124

o: Pelo corolario anterior temos


Demonstra
ca
x

n+1 pn + pn1 pn pn1 qn pn qn1


pn
=
=
qn
n+1 qn + qn1
qn (n+1 qn + qn1 )qn
(pn qn1 pn1 qn )
(1)n1
=
=
(n+1 qn + qn1 )qn (n+1 qn + qn1 )qn
(1)n
(1)n
(1)n
=
=
.
=
(n+1 qn + qn1 )qn (n+1 + qn1 /qn )qn2 (n+1 + n+1 )qn2

Em particular,





1
x pn =
,


qn
(n+1 + n+1 )qn2

e, como n+1 = an+1 e 0 < n+1 < 1, segue que an+1 < n+1 +n+1 <
an+1 + 2, o que implica a u
ltima afirmacao.
A expans
ao de n+1 como fracao contnua segue de
qn1
qn1
qn1
1
=
=
=
qn
an qn1 + qn2
qn
an + qqn2
n1
aplicado recursivamente.
Observa
c
ao 3.5. Como limn qn = + (pois (qn ) e estritamente
crescente), segue desta proposic
ao que
lim

pn
= x,
qn

o que permite recuperar x a partir de a0 , a1 , a2 , . . . , e d


a sentido `
a
igualdade x = [a0 ; a1 , a2 , . . . ] quando a frac
ao contnua de x e infinita
(i.e., quando x e irracional).
Observa
c
ao 3.6. A proposic
ao anterior implica que, para todo irracional, a desigualdade | p/q| < 1/q 2 tem infinitas soluc
oes racionais
p/q. Este fato e conhecido como o Teorema de Dirichlet.
interessante notar que, se = r/s Q, a desigualdade | p/q| <
E
2
1/q tem apenas um n
umero finito de soluc
oes racionais p/q. De fato,
|r/s p/q| < 1/q 2 equivale a |qr ps| < s/q, o que implica que q s.
A seguinte proposicao mostra que os convergentes pares formam uma
sequencia crescente, e que os convergentes mpares formam uma sequencia decrescente. Alem disso todos os convergentes mpares s
ao maiores
do que todos os convergentes pares.
Proposi
c
ao 3.7. Para todo k 0, temos

p2k
p2k+2
p2k+3
p2k+1

.
q2k
q2k+2
q2k+3
q2k+1

125
o: O resultado segue dos seguintes fatos gerais. Para
Demonstra
ca
todo n 0, temos que
pn+2 pn
an+2 pn+1 + pn pn

qn+2
qn
an+2 qn+1 + qn
qn
(1)n an+2
an+2 (pn+1 qn pn qn+1 )
=
=
qn (an+2 qn+1 + qn )
qn+2 qn

e positivo para n par e negativo para nn mpar. Alem disso, para todo
e positivo para n par e
n 0, temos que x pqnn = (n+1 q(1)
n +qn1 )qn
negativo para n mpar.
Proposi
c
ao 3.8. Sejam a0 , a1 , . . . , an inteiros com ak > 0, k 1,
e seja (pk /qk )k0 a sequencia de reduzidas da frac
ao contnua
[a0 ; a1 , a2 , . . . , an ]. Ent
ao o conjunto dos n
umeros reais cuja representac
ao por frac
oes contnuas comeca com a0 , a1 , . . . , an e o intervalo
 
pn
I(a0 , a1 , . . . , an ) =
{[a0 , a1 , . . . , an , ], > 1}
q
hn

pn , pn +pn1
se n e par
q
q +q
i
=  pn +pn n1
n n1 , pn
se n e mpar.
qn +qn1 qn

Alem disso, a func


ao G : (1, +) I(a0 , a1 , . . . , an ) dada por G() =
[a0 ; a1 , a2 , . . . , an , ] e mon
otona, sendo crescente para n mpar e decrescente para n par.

suficiente notar que, como na prova do corolario


o: E
Demonstra
ca
(1)n
pn
n +pn1
anterior, G() = [a0 ; a1 , a2 , . . . , an , ] = p
qn +qn1 = qn + (qn +qn1 )qn ,
e portanto G e crescente para n mpar e decrescente para n par. Assim,
+pn1
como G(1) = pqnn +q
e lim G() = pqnn , temos
n1
+

G((1, +)) =

+pn1
( pqnn , pqnn +q
)
n1
n1 pn
( pqnn +p
+qn1 , qn )

se n e par
se n e mpar.

Portanto,



pn
I(a0 , a1 , . . . , an ) =
{[a0 , a1 , . . . , an , ], > 1}
qn
 
pn
G((1, +))
=
qn
h

pn , pn +pn1
se n e par
q
q +q
i
=  pn +pn n1
n n1 , pn
se n e mpar.
qn +qn1 qn


CAPITULO 3. FRAC
OES
CONTINUAS

126

Proposi
c
ao 3.9. Dados inteiros a0 , a1 , a2 , . . . , com ak > 0, k 1,
existe um u
nico n
umero real (que e irracional) cuja representac
ao por
frac
oes contnuas e [a0 ; a1 , a2 , . . . ].
o:
Demonstra
ca
recorrencias

Considere as sequencias (pn ) e (qn ) definidas pelas

pn+2 = an+2 pn+1 + pn

qn+2 = an+2 qn+1 + qn

para todo n 0, com p0 = a0 , p1 = a0 a1 + 1, q0 = 1 e q1 = a1 . Temos,


como na proposicao 3.7,
p2k
p2k+2
p2k+3
p2k+1

, k 0.
q2k
q2k+2
q2k+3
q2k+1
Assim, considerando os intervalos fechados


p2k p2k+1
,
,
Ik =
q2k q2k+1
temos Ik+1 Ik , k 0, e portanto, como
|Ik | =

p2k+1 q2k p2k q2k+1


(1)2k
1
p2k+1 p2k

=
=
=
q2k+1
q2k
q2k+1 q2k
q2k+1 q2k
q2k+1 q2k

tende a 0 quando k tende a infinito, existe R tal que


\
Ik = {}.
k0

Como, para todo k 0,


[a0 ; a1 , a2 , . . . , a2k ] =

p2k
p2k+1

= [a0 ; a1 , a2 , . . . , a2k , a2k+1 ]


q2k
q2k+1

e, da proposicao anterior, [a0 ; a1 , a2 , . . . , a2k ] e [a0 ; a1 , a2 , . . . , a2k , a2k+1 ]


pertencem a I(a0 ; a1 , a2 , . . . , a2k ), que e um intervalo, segue que
I(a0 ; a1 , a2 , . . . , a2k ), e portanto a fracao contnua de comeca
com a0 , a1 , . . . , a2k , para todo k 0, donde a representacao por fracoes
contnuas de e [a0 ; a1 , a2 , . . . ].
Note que, como a representacao por fracoes contnuas de e infinita,
e irracional.
Exemplo 3.10. Temos
= [3; 7, 15, 1, 292, 1, 1, 1, 2, 1, 3, 1, 14, 2, 1, . . . ], portanto
p0
= 3,
q0

p1
22
= ,
q1
7

p2
333
=
,
q2
106

p3
355
=
...
q3
113


3.1. REDUZIDAS E BOAS APROXIMAC
OES

127

e = [2; 1, 2, 1, 1, 4, 1, 1, 6, 1, 1, 8, . . . , 1, 1, 2n, . . . ]

2 = [1; 2, 2, 2, . . . ] pois

2=1+

1+ 5
2

1
=1+
2+1

1
2+

= [1; 1, 1, 1, . . . ] pois

1+ 5
=1+
2

1+ 5
2

=1+
2+

2+1

2+

=1+
1+

1+ 5
2

1
1
2+1

Isto prova em particular que 2 e 1+2 5 s


ao irracionais, pois suas frac
oes

1
=
[0;
2,
2,
2
.
.
.
]e
cont
nuas
s
a
o
infinitas.
Da

segue
tamb
e
m
que

51
= [0; 1, 1, 1, . . . ] s
ao pontos fixos da transformac
ao de Gauss g.
2

3.1

Reduzidas e Boas Aproximac


oes

Teorema 3.11. Temos, para todo n N,






1
1
x pn
< 2


qn
qn qn+1
qn

Alem disso,





x pn < 1

qn 2qn2

ou





x pn+1 < 1 .

2
qn+1 2qn+1

o: O n
Demonstra
ca
umero x sempre pertence ao segmento de extrepn+1
pn
mos qn e qn+1 cujo comprimento e






pn+1 pn (1)n
1
1
pn
1





qn+1 qn = qn qn+1 = qn qn+1 = x qn qn qn+1 < q 2
n
Alem disso, se




x pn 1

qn 2qn2
entao





x pn+1 1 ,

2
qn+1 2qn+1





pn+1
1
pn
1
1

2 + 2
= qn+1 = qn ,
= x + x

qn qn+1
qn
qn+1
2qn 2qn+1

absurdo.


CAPITULO 3. FRAC
OES
CONTINUAS

128



Observa
c
ao 3.12. De fato x pqnn < qn q1n+1 <
for an+1 melhor ser
a a aproximac
ao pqnn de x.

1
2.
an+1 qn

Quanto maior

Teorema 3.13 (Hurwitz, Markov). Para todo irracional e todo inteiro


n 1, temos




p
< 1

q
5q 2
para pelo menos um racional
p


pn1 pn pn+1
, ,
.
qn1 qn qn+1

Em particular, a desigualdade acima tem infinitas soluc


oes racionais
p/q.
o: Suponha que o teorema seja falso.
Demonstra
ca
Entao, pela proposi
n 1 com n + n 5, n+1 + n+1

cao 3.4, existe irracional,


5 e n+2 + n+2 5. Devemos portanto ter an+1 = an+2 = 1 ja
que claramente ak 2 para k = n, n + 1, n +2 e se algum ak = 2 com
k = n + 1, n + 2, teramos ak + k 2 + 31 > 5, absurdo.
Sejam x = 1/n+2 e y = n+1 . As desigualdades acima se traduzem
em
1
1
+ 5,
1+x y

1+x+y

1
1
+
5.
x 1+y

Temos
1+x+y

5 = 1 + x

5y

5
1
1
1
1
+
=
+ =
1+x y
5y y
y( 5 y)

e portanto y( 5 y) 1 = y
x

5 1 y =

51
2 .

Por outro lado temos

1
1
1
1
+

+
x 1+y
51y 1+y

=
(1 + y)( 5 1 y)

e portanto (1 + y)( 5 1 y) 1 = y

ter y =

51
2 ,

o que e absurdo pois y =

51
2 , e portanto
n+1 = qn1
qn Q.

devemos


REDUZIDAS
3.2. BOAS APROXIMAC
OES
SAO

129



Observa
c
ao 3.14. Em particular provamos que pq <
p
q,

1
2
5q

para todo irracional. O n


umero
infinitas soluc
oes racionais
maior com essa propriedade. De fato, se




p
1
1+ 5

<
e
,
> 0, =

2
q
( 5 + )q 2

tem
5 e o

temos

ou seja,



1 + 5 

1


p <
q

( 5 + )q
2

15 p

1 5 
2 q
1 + 5 



p q
p <
,
= q



2
2
5+


1 + 5 p 


5 ( 5 + ).
|p pq q | <
2

q
2

Se q e grande, 1/q 2 e pequeno, e

1+ 5
2

pq e muito pr
oximo de 0, donde

5
< 1, absurdo,
o lado direito da desigualdade e muito pr
oximo de 5+
2
2
2
2
pois |p pq q | 1, de fato se p pq q = 0 teramos



 2  
1+ 5 1 5
p
p
p

1 = 0 =
,
q
q
q
2
2

o que e absurdo, pois

p
q

Q.





1
Outra maneira de ver que, para todo > 0, 1+2 5 pq < (5+)q
2
tem apenas um n
umero finito de soluc
oes pq Q e observar que as me

lhores aproximac
oes racionais de 1+2 5 s
ao as reduzidas pqnn de sua frac
ao contnua
oxima sec
ao), para as quais temos
[1; 1, 1, 1, . . . ] (ver pr
1+5 pn
1
2 qn = (n+1 +n+1 )q2 , com n+1 + n+1 se aproximando cada vez
n
mais de

1+ 5
51
[1; 1, 1, 1, . . . ] + [0; 1, 1, 1, . . . ] =
+
= 5.
2
2

3.2

Boas Aproximac
oes s
ao Reduzidas

O proximo teorema (e seu corolario 3.17) caracteriza as reduzidas


em termos do erro reduzido da aproximacao de x por p/q, o qual e, por
definicao, |qxp|, a razao entre |xp/q| e o erro m
aximo da aproximacao
por falta com denominador q, que e 1/q.

130

CAPITULO 3. FRAC
OES
CONTINUAS

Teorema 3.15. Para todo p, q Z, com 0 < q < qn+1 temos


|qn x pn | |qx p|.
Alem disso, se 0 < q < qn a desigualdade acima e estrita.
o: Como mdc(pn , qn ) = 1, temos que se pq = pqn entao
Demonstra
ca
n
p = kpn e q = kqn para algum inteiro k 6= 0 e neste caso o resultado e
claro. Assim, podemos supor que pq 6= pqnn de modo que


p pn
1
1 >

q
qn
qqn
qn qn+1
ja que q < qn+1 . Assim, pq esta fora do intervalo de extremos
e portanto










x p min p pn , p pn+1 1





q
q
qn
q
qn+1
qqn+1
o que implica

|qx p|

qn+1

pn
qn

pn+1
qn+1

|qn x pn |.

n+1
, donde an+1 2,
Alem disso, a igualdade s
o pode ocorrer se x = pqn+1
e qn+1 > 2qn , pois numa fracao contnua finita, como no algoritmo de
Euclides, o u
ltimo coeficiente an e sempre maior que 1. Nesse caso, se
q < qn , teremos








x p p pn pn+1 pn

q q
qn qn+1
qn
1
1
qn+1 q
1

=
>

qqn qn qn+1
qqn qn+1
qqn+1

o que implica
|qx p| >

1
qn+1

|qn x pn |.

Corol
ario 3.16. Para todo q < qn ,




x pn < x

qn


p
q

Corol
ario 3.17. Se |qx p| < |q x p |, para todo p e q q tais que

p
p
ao p/q e uma reduzida da frac
ao contnua de x.
q 6= q , ent
o: Tome n tal que qn q < qn+1 . Pelo teorema,
Demonstra
ca
|qn x pn | |qx p|, e portanto p/q = pn /qn .


REDUZIDAS
3.2. BOAS APROXIMAC
OES
SAO


Teorema 3.18. Se x pq <
contnua de x.

1
2q 2

ent
ao

p
q

131

e uma reduzida da frac


ao

o: Seja n tal que qn q < qn+1 . Suponha que pq 6= pqn .


Demonstra
ca
n


1
e assim pq
Como na demonstracao do teorema anterior, x pq qqn+1
n+1
. Temos duas possibilidades:
esta fora do intervalo de extremos pqnn e pqn+1
(a) Se q

qn+1
2

(b) Se q <

qn+1
2 ,



entao x pq


x

1
qqn+1

1
,
2q 2

absurdo.




p pn p pn+1 pn



q qn
q
qn+1
qn
1
1
qn+1 q

=
qqn qn qn+1
qqn qn+1
1
1
>
2
2qqn
2q

o que tambem e um absurdo.

Exemplo 3.19. Seja =[a0 ; a1 , a2 , . . . ]R. Prove que, se qn q <


qn+1 , mdc(p, q) = 1 e p/q 6= pn /qn ent
ao | p/q| | pn /qn | se,
pn+1 rpn
p
e somente se, q = qn+1 rqn , onde r N e tal que 0 < r < an+1 /2 ou
(r = an+1 /2 e n+2 n+1 1).


o: Como antes, pq pqn qq1 > q q1
, e portanto pq esta
Solu
ca
n
n
n n+1
n+1
. Se | pq | | pn /qn |,
fora do intervalo de extremos pqnn e pqn+1
p
n+1
a do mesmo lado que pqn+1
em relacao a , e portanto pq pqnn =
q est
n

n+1
tem o mesmo sinal de pqn+1
pqnn = q(1)
; por outro lado, se
n qn+1
p
p
tivessemos |pqn qpn | 2, teramos | q | | q pqnn | | pqnn |
1
1
2
qqn qn qn+1 > qn qn+1 | pn /qn |, absurdo. Assim, devemos ter
pqn qpn = (1)n = pn+1 qn qn+1 pn , e portanto, como q < qn+1 ,
devemos ter p = pn+1 rpn = (an+1 r)pn + pn1 e q = qn+1 rqn =
(an+1 r)qn + qn1 , para algum inteiro positivo r.
Temos entao






p = n+1 pn + pn1 (an+1 r)pn + pn1 =

q n+1 qn + qn1
(an+1 r)qn + qn1

pqn qpn
qqn

n+1 an+1 + r
,
(n+1 qn + qn1 )((an+1 r)qn + qn1 )


CAPITULO 3. FRAC
OES
CONTINUAS

132
pn
qn |

1
(n+1 qn +qn1 )qn , temos | p/q| | pn /qn | se, e
n +qn1
somente se, n+1 an+1 + r (an+1 r)q
= an+1 r + n+1 . Se 0 <
qn
r < an+1 /2, temos n+1 an+1 + r < 1 + r an+1 r < an+1 r + n+1 .

e como |

Se r > an+1 /2, n+1 an+1 + r r an+1 + 1 r > an+1 r + n+1 .


Finalmente, se r = an+1 /2, n+1 an+1 + r an+1 r + n+1 equivale
1
a n+2
= n+1 an+1 n+1 , ou seja, a n+2 n+1 1.
Exemplo 3.20. Seja =[a0 ; a1 , a2 , . . . ]R. Prove que, se qn q <
qn+1 , mdc(p, q) = 1 e p/q 6= pn /qn ent
ao | p/q| < 1/q 2 se, e somente
n+1 pn
se, (an+1 2, pq = pqn+1
qn e an+1 2 + n+1 < n+2 ) ou (an+1 2,
p
q

pn +pn1
qn +qn1

e (n+1 2)n+1 < 1).






Como antes, pq pqnn

p
1
1
a
qqn > qn qn+1 , e portanto q est
pn+1
pn
1
fora do intervalo de extremos qn e qn+1 . Se | p/q| q2 , como
1
n+1
em relacao a , e portanto
qq1n , pq esta do mesmo lado que pqn+1
q2
n
p
p
pqn qpn
pn
n+1
tem o mesmo sinal de qn+1
pqnn = q(1)
; por outro
q qn =
qqn
n qn+1
p
lado, se tivessemos |pqn qpn | 2, teramos | q | | pq pqnn |
| pqnn | qq2n qn q1n+1 > qq1n q12 , absurdo. Assim, devemos ter
pqn qpn = (1)n = pn+1 qn qn+1 pn , e portanto, como q < qn+1 ,

o:
Solu
ca

devemos ter p = pn+1 rpn = (an+1 r)pn + pn1 e q = qn+1 rqn =


(an+1 r)qn + qn1 , para algum inteiro positivo r.
Temos entao, como antes,




n+1 an+1 + r
p =
,

q (n+1 qn + qn1 )((an+1 r)qn + qn1 )
donde | p/q| <

1
q2

1
((an+1 r)qn +qn1 )2

se, e somente se,

(n+1 an+1 + r)((an+1 r)qn + qn1 ) < n+1 qn + qn1 .


1
1
Se essa desigualdade ocorre, devemos ter n+2
+ r(an+1 r) (n+2
+
r)(an+1 r) (n+1 an+1 + r)((an+1 r)qn + qn1 )/qn qn1 /qn <
1
(n+1 qn +qn1 )/qn qn+1 /qn = n+2
+an+1 , donde r = 1 ou r = an+1 1
(caso contrario, teramos an+1 4 e r(an+1 r) 2(an+1 2) an+1 ,
absurdo).
Se r = 1 (e an+1 2), temos (n+1 an+1 +r)((an+1 r)qn +qn1 ) <
n+1 qn +qn1 equivalente a (n+1 an+1 +1)(an+1 1+n+1 ) < n+1 +
1
n+1 , que, escrevendo n+1 = an+1 + n+2
, equivale a an+1 2 + n+1 <
n+2 . Finalmente, se r = an+1 1, temos (n+1 an+1 +r)((an+1 r)qn +
qn1 ) < n+1 qn +qn1 equivalente a (n+1 1)(1+n+1 ) < n+1 +n+1 ,
que equivale a (n+1 2)n+1 < 1.


REDUZIDAS
3.2. BOAS APROXIMAC
OES
SAO

133

Dado R, definimos a ordem de como o u


nico n
umero real
ord = > 0 (se existir) tal que, para todo > 0

p
1
p


< tem infinitas solucoes Q e
q
q
q
1
p
umero finito de solucoes racionais p/q
| | < + tem apenas um n
q
q

p
1
(caso nao exista um tal , i.e., caso | | < tenha infinitas solucoes
q
q
racionais p/q para todo > 0, definimos ord = +).
Observemos que, se = r/s Q, com r e s inteiros, S > 0, entao
ord = 1. De fato, para todo racional p/q 6= , temos
r p
|qr ps|
1
c
1
p

= , onde c = > 0, e portanto


| | = | | =
q
s q
sq
sq
q
s
1
p
umero finito de solucoes racionais p/q.
| | < 1+ tem apenas um n
q
q
Por outro lado, pelo teorema 3.11, para todo irracional, temos
ord 2. A ordem de todo n
umero irracional pode ser calculada (por
causa de 3.18 e usando a proposicao 3.4) a partir de sua fracao contnua.
Vejamos por exemplo a seguinte
Proposi
c
ao 3.21. Para todo [2, +] existe irracional com ord =
.
o: Sabemos que as melhores aproximacoes por racioDemonstra
ca
nais s
ao dadas pelos convergentes da fracao contnua. Se < +, tomamos = [0; a1 , a2 , a3 , ...], onde a1 = 2 e, para cada n 1, an+1 = qn2
e o menor inteiro que e maior ou igual a qn2 . Temos entao, para todo
n 1, pela proposicao 3.4,
1
1
1
1
1
pn
1
<
2
< | | <
2 2 = ,
2
2
2
4qn
(a
+
2)q
q
a
q
q
(qn + 3)qn
qn qn
n+1
n
n+1 n
n
n
o que claramente implica ord = .
Por outro lado, no caso = +, podemos tomar = [0; a1 , a2 , a3 , ...],
onde a1 = 2 e, para cada n 1, an+1 = qnn2 . Temos entao, para todo
n 1, pela proposicao 3.4,
|

1
1
1
pn
|<
n2 2 = n ,
2
qn
an+1 qn
qn
qn qn

e portanto ord = +.


CAPITULO 3. FRAC
OES
CONTINUAS

134

3.3

Frac
oes Contnuas Peri
odicas

Nesta secao provaremos que os n


umeros reais com fracao contnua
peri
odica s
ao exatamente as razes de equacoes do segundo grau com
coeficientes inteiros.
Lembramos que na representacao de x por fracao contnua, an , n
s
ao definidos por recurs
ao por
0 = x,
e temos
n =

an = n ,

n+1 =

pn2 qn2 x
,
qn1 x pn1

1
n a n

n N.

Isso da uma prova explcita do fato de que se a fracao contnua de


x e peri
odica, entao x e raiz de uma equacao do segundo grau com
coeficientes inteiros. De fato, se n+k = n , n N, k N>0 segue que
pn2 qn2 x
pn+k2 qn+k2 x
=
,
qn1 x pn1
qn+k1 x pn+k1
entao Ax2 + Bx + C = 0, onde
A = qn1 qn+k2 qn2 qn+k1

B = pn+k1 qn2 + pn2 qn+k1 pn+k2 qn1 pn1 qn+k2


C = pn1 pn+k2 pn2 pn+k1 .

n1
e uma fracao irredutvel
Note que o coeficiente de x2 e nao-nulo, pois qqn2
q
de denominador qn2 , pois pn1 qn2 pn2 qn1 = (1)n , e qn+k1
e uma
n+k2
qn+k1
qn1
fracao irredutvel de denominador qn+k2 > qn2 , donde qn2 6= qn+k2 ,
logo qn1 qn+k2 qn2 qn+k1 6= 0.
Vamos provar agora um resultado devido a Lagrange segundo o qual
se x e uma irracionalidade quadr
atica, isto e, se x e um irracional do

odica,
tipo r + s, r, s Q, s > 0, entao a fracao contnua de x e peri
i.e., existem n N e k N>0 com n+k = n . Neste caso, existem
a,

b, c inteiros tais que ax2 + bx + c = 0, com b2 4ac > 0 e b2 4ac


n +pn2
, temos
irracional. Como x = pqn1
n1 n +qn2

ax2 + bx + c = 0




pn1 n + pn2 2
pn1 n + pn2
= a
+b
+c=0
qn1 n + qn2
qn1 n + qn2

= An n2 + Bn n + Cn = 0,

135

3.4. O ESPECTRO DE LAGRANGE


onde
2
An = ap2n1 + bpn1 qn1 + cqn1

Bn = 2apn1 pn2 + b(pn1 qn2 + pn2 qn1 ) + 2cqn1 qn2


2
Cn = ap2n2 + bpn2 qn2 + cqn2
.

Note que Cn = An1 . Vamos provar que existe M > 0 tal que
0 < |An | M para todo n N, e portanto 0 < |Cn | M , n N:



pn1
pn1
2
2
2
An = apn1 + bpn1 qn1 + cqn1 = aqn1 x
x

,
qn1
qn1
onde x e x
s
ao as razes de aX 2 + bX + c = 0, mas









pn1
2
x pn1 x
x pn1 < 1 1 = |An | = aqn1



2
qn1 qn1
qn1
qn1




pn1
a |
x x| + x
qn1
def

M = a(|
x x| + 1).

Notemos agora que, para qualquer n N,


Bn2 4An Cn = (pn1 qn2 pn2 qn1 )2 (b2 4ac) = b2 4ac.
Portanto
Bn2 4An Cn + b2 4ac 4M 2 + b2 4ac
p
def
= Bn M = 4M 2 + b2 4ac.

Provamos assim que An , Bn e Cn estao uniformemente limitados,


donde ha apenas um n
umero finito de possveis equacoes An X 2 +Bn X +
Cn = 0, e portanto de possveis valores de n . Assim, necessariamente
n+k = n para alguma escolha de n N, k N>0 .

3.4

O espectro de Lagrange

Seja um n
umero irracional. Definimos k() como o u
nico k > 0
1
p
tem infinitas
(se existir) tal que, para todo > 0, | | <
q
(k + )q 2
p
1
solucoes racionais p/q e | | <
tem apenas um n
umero finito
q
(k )q 2
p
de solucoes racionais p/q (caso nao exista um tal k, i.e., caso | | <
q


CAPITULO 3. FRAC
OES
CONTINUAS

136

1
tenha infinitas solucoes racionais p/q para todo k > 0, definimos
kq 2

k() = +). Pelo Teorema de Hurwitz-Markov, temos


k() 5,

R \ Q. Por outro lado, e possvel provar que k( 1+2 5 ) = 5.


Estaremos interessados nos R tais que k() < +, e, mais
particularmente, na imagem da funcao k, isto e, no conjunto L = {k() |
R\Q e k() < +}. Este conjunto e conhecido como o espectro de
Lagrange.
O conjunto L encodifica uma serie de propriedades diofantinas (isto
e, relativas `as boas aproximacoes por n
umeros racionais) de n
umeros
reais, e vem sendo estudado ha bastante tempo. Talvez o primeiro resultado nao-trivial sobre ele se deva a Markov, que provou em 1879 que
(
)

221
L (, 3) = k1 = 5 < k2 = 2 2 < k3 =
< ,
5
onde (kn ) e uma sequencia convergente a 3 tal que kn
/ Q mas kn2 Q
para todo n. Assim, o comeco do espectro de Lagrange e discreto. Essa
afirmacao nao e verdadeira para todo o conjunto L. Marshall Hall provou
em 1947 que L contem toda uma semi-reta (por exemplo [6, +)), e G.
Freiman
que esta contida em L,
" determinou em 1975a maior semi-reta
!
2221564096 + 283748 462
que e
, + .
491993569
Uma apresentacao detalhada destes e de outros resultados sobre o
espectro de Lagrange pode ser encontrada em [3].
Na referencia [9] s
ao provados resultados sobre propriedades geometricas (relativas a geometria fractal) dos espectros de Markov e Lagrange,
que envolvem resultados delicados sobre somas de conjuntos de Cantor
regulares.

Problemas Propostos

3.1. Determine a frac


ao contnua de 7. Mostre que ela e peri
odica a
partir de um certo ponto, e determine o perodo.

umeros reais
3.2. Escreva na forma r + s, com r, s Q, s 0, os n
cujas representac
oes em frac
oes contnuas s
ao as seguintes:
(a) [0; 3, 6, 3, 6, 3, 6, . . . ].
(b) [0; k, k, k, . . . ], onde k e um inteiro positivo dado.
(c) [0; 1, 1, 2, 2, 1, 1, 2, 2, 1, 1, 2, 2, . . . ].

137

3.4. O ESPECTRO DE LAGRANGE

3.3. (a) Sabendo que 3, 14 < x < 3, 15, determine o maior natural n e
inteiros a0 , a1 , . . . , an para os quais e possvel garantir que a representac
ao em frac
oes contnuas de x comeca por [a0 ; a1 , . . . , an ].
(b) Sabendo que 3, 141592 < x < 3, 141593, determine o maior natural
n e inteiros a0 , a1 , . . . , an para os quais e possvel garantir que a
representac
ao em frac
oes contnuas de x comeca por [a0 ; a1 , . . . , an ].
(c) Sabendo que 3, 1415926 < x < 3, 1415927, determine o maior natural n e inteiros a0 , a1 , . . . , an para os quais e possvel garantir que a
representac
ao em frac
oes contnuas de x comeca por [a0 ; a1 , . . . , an ].
3.4. (a) Determine as primeiros 6 reduzidas da frac
ao continua de

5.

(b) Definimos a sequencia an = n 5 n 5. Determine os valores de


n 2011 tais que an seja respectivamente m
aximo e mnimo.
3.5. Demonstrar que, para todo inteiro positivo a, temos as seguintes
expans
oes em frac
oes contnuas peri
odicas:
(a)

(b)

(c)

(d)

a2 + 1 = [a, 2a].
a2 1 = [a 1, 1, 2a 2].
a2 2 = [a 1, 1, a 2, 1, 2a 2].
a2 a = [a 1, 2, 2a 2].

3.6. Encontrar as frac


oes contnuas de

a2 + 4 e

a2 4.

3.7.
Prove que, para quaisquer inteiros p, q com q > 0, temos
| 2 pq | > 3q12 . Determine todos os pares de inteiros (p, q) com q > 0

tais que | 2 pq | < q13 .


3.8. Prove que, para qualquer R \ Q, e quaisquer s, t R com s < t,
existem inteiros m, n com n > 0 tais que s < n + m < t.
3.9. Seja
pn
=
qn

1
1+

12
32
2+
52
2+
(2n 3)2
..
.2+
2


CAPITULO 3. FRAC
OES
CONTINUAS

138

a n-esima convergente da frac


ao contnua
1
1+

Demonstrar que

pn
qn

=1

1
3

12
32
2+
52
2+
72
2+
..
.

1
5

1
7

1
+ + (1)n1 2n1
.

3.10. Dizemos que dois n


umeros irracionais e s
ao GL2 (Z)-equivalentes se existem inteiros a, b, c, d com |ad bc| = 1 tais que = a+b
c+d .
Mostre que, se as frac
oes contnuas de e s
ao = [a0 ; a1 , a2 , . . . ]
e = [b0 ; b1 , b2 , . . . ] ent
ao e s
ao GL2 (Z)-equivalentes se, e somente
se, existem r Z e n0 N tais que bn = an+r , n n0 .

Captulo 4

Equa
c
oes diofantinas n
ao
lineares
4.1

Teorema de Pit
agoras e triplas Pitag
oricas

Um dos teoremas basicos e elementares da geometria euclidiana e o


conhecido Teorema de Pit
agoras, que relaciona o comprimento dos lados
de um tri
angulo quando um dos
angulos e reto. Nesta secao estudaremos
os tri
angulos ret
angulos com a restricao adicional que o comprimento dos
lados s
ao n
umeros inteiros. Antes de impor tal restricao vejamos uma
prova, entre as muitas conhecidas, deste teorema.
Teorema 4.1 (Pit
agoras). Sejam a, b e c os comprimentos dos lados
de um tri
angulo. O
angulo oposto ao lado c e reto se e somente se
a 2 + b2 = c 2 .
o: Nossa prova se baseia no seguinte desenho, no qual
Demonstra
ca
temos colocado quatro tri
angulos ret
angulos de catetos a e b e hipotenusa
c ao redor de um quadrado de lado c.
b
a

a
c

c
a

b
139


LINEARES
140 CAPITULO 4. EQUAC
OES
DIOFANTINAS NAO
Observe que a figura externa formada tambem e um quadrado (comprove!) de lado a + b. A
area deste quadrado pode ser calculada de duas
formas diferentes: uma e a forma natural, elevando o comprimento do
lado ao quadrado, isto e (a + b)2 , e a outra calculando a area como a
soma das areas dos quatro tri
angulos mais a area do quadrado de lado
c, isto e,
 
ab
2
(a + b) = 4
+ c2 = 2ab + c2 .
2
Subtraindo 2ab dos dois lados da igualdade obtemos a relacao desejada.

Claramente esta relacao mostra a dependencia de um lado com respeito aos outros dois. Desta forma e facil encontrar exemplos de tri
angulos que tem
exatamente
dois
lados
de
comprimento
inteiro,
como
por

exemplo (1, 1, 2), (1, 2, 5), (3, 2 10, 7) etc. Menos evidente e encontrar tri
angulos com todos seus lados inteiros, fato que pode ser ilustrado
no seguinte
Exemplo 4.2. Encontre todos os tri
angulos ret
angulos com lados inteiros e um cateto igual a 30.
o: Precisamos encontrar inteiros positivos b e c tais que 900 +
Solu
ca
2
2
b = c . Esta igualdade pode ser reescrita da seguinte forma
22 32 52 = (c + b)(c b).
Observe que os n
umeros c + b e c b tem a mesma paridade, logo os dois
s
ao pares, e como c + b e maior que c b, as u
nicas formas de distribuir
os fatores de 900 entre estes fatores s
ao
(
(
(
(
c + b = 450
c + b = 150
c + b = 90
c + b = 50
e
cb=2
cb=6
c b = 10
c b = 18
que gera as solucoes (30, 224, 226), (30, 72, 78), (30, 40, 50) e (30, 16, 34).

N
ao temos um metodo an
alogo para determinar os lados de um tri
angulo dado o comprimento da hipotenusa, mas eles podem ser encontrados
por uma procura ordenada como e mostrado a seguir
Exemplo 4.3. Encontre todos os tri
angulos ret
angulos com lados inteiros e a hipotenusa igual a 65.
Neste caso, precisamos encontrar dois n
umeros inteiros a e b, que
podemos supor, sem perda de generalidade, satisfazerem a < b, tais que

4.2. TRIANGULOS
RETANGULOS
DE PITAGORAS
E PLATAO141
a2 +b2 = 652 = 4225. Como b2 +(b1)2 4225, temos que 64 b 47.
Assim temos 18 possveis valores de b que precisamos testar se verificam
que 4225 b2 e um quadrado perfeito. De fato, este n
umero e quadrado
perfeito quando b e igual a 52, 56, 60 e 63, que gera os tri
angulos com
lados de comprimento (39, 52, 65), (33, 56, 65), (25, 60, 65) e (16, 63, 65).
O caso geral do problema anterior sera resolvido por outro metodo
na secao 4.12.
No caso em que os tres n
umeros (a, b, c) sejam inteiros e sejam lados
de um tri
angulo ret
angulo, diremos que estes tres n
umeros formam uma
Tripla Pitag
orica, que sera o objeto de estudo nas seguintes secoes.

Problemas Propostos
4.1. Determine todos os tri
angulos ret
angulos com lados inteiros e um
de seus catetos com comprimento igual a
a) 60
b) 825
4.2. Determine todos os tri
angulos ret
angulos com lados inteiros e hipotenusa de comprimento 105.
4.3. a) Mostre que o quadrado de um n
umero mpar sempre deixa resto
1 quando dividido por 8.
b) Existe algum tri
angulo ret
angulo com lados inteiros e catetos mpares?

4.4. Observem
que
usando
o
tri
a
ngulo
ret
a
ngulo
(1,
1,
2), e possvel

construir 2 com regua e compasso. Mostrar que para todo inteiro po


sitivo n, o n
umero n e construtvel com regua e compasso.
4.5. Existe algum tri
angulo ret
angulo com lados inteiros e permetro
igual a uma potencia de 2?

4.2

Tri
angulos ret
angulos de Pit
agoras e Plat
ao

Observemos que podemos gerar infinitas triplas pit


agoricas a partir
2
2
2
de uma. De fato, como 3 + 4 = 5 , temos que (3, 4, 5) e uma tripla
Pitagorica, e a partir dela podemos gerar infinitas multiplicando por um
inteiro positivo:
(6, 8, 10), (3, 12, 15), . . . , (3k, 4k, 5k).


LINEARES
142 CAPITULO 4. EQUAC
OES
DIOFANTINAS NAO
primitivas,
As triplas nesta lista s
ao chamadas triplas pitagoricas NAO
porque s
ao obtidas multiplicado (3, 4, 5) por um n
umero inteiro maior
que 1, enquanto (3, 4, 5) e chamada tripla pitagorica primitiva, isto e,
uma tripla pitagorica (a, b, c) e chamada primitiva se ela nao e obtida a
partir de outra multiplicando por uma constante inteira maior ou igual
a 2.
Pit
agoras observou que existe uma famlia infinita de triplas pitagoricas primitivas: (5, 12, 13), (7, 24, 25) (9, 40, 41), etc. Em todos os casos
estudados por ele, um dos catetos e a hipotenusa s
ao inteiros consecutivos. Para gerar uma formula para todas as triplas do tipo (a, b, b + 1)
(chamadas triplas pitagoricas cl
assicas de primeiro tipo) e suficiente observar que
a2 = (b + 1)2 b2 = 2b + 1,
logo a e um n
umero mpar, portanto a = 2k + 1 com k um n
umero
2
natural, assim substituindo na formula anterior temos que 4k +4k +1 =
2b + 1, e assim b = 2k 2 + 2k e c = 2k 2 + 2k + 1, isto e, obtemos a famlia
de triplas pitagoricas (2k + 1, 2k 2 + 2k, 2k 2 + 2k + 1).
Por outro lado, Platao observou outra famlia de triplas primitivas
em que a diferenca entre a hipotenusa e um cateto e 2 (chamadas triplas
pitagoricas cl
assicas de segundo tipo), isto e, triplas da forma (a, b, b+2).
Seguindo o argumento anterior temos que a2 = (b + 2)2 b2 = 4b + 4.
Assim a e par. Escrevendo a = 2s e substituindo na equacao anterior
obtemos que s2 = b + 1, como estamos interessados somente em triplas
Pitagoricas primitivas, entao b nao pode ser par, assim s nao pode ser
mpar, assim s = 2k com k inteiro, e neste caso obtemos a famlia de
triplas pitagoricas (4k, 4k 2 1, 4k 2 + 1).
A seguinte tabela mostra as triplas obtidas a partir das equacoes de
Pit
agoras e Platao
k
1
2
3
4
5
6
7

(2k + 1, 2k 2 + 2k, 2k 2 + 2k + 1)
(3, 4, 5)
(5, 12, 13)
(7, 24, 25)
(9, 40, 41)
(11, 60, 61)
(13, 84, 85)
(15, 112, 113)

(4k, 4k 2 1, 4k 2 + 1)
(4, 3, 5)
(8, 15, 17)
(12, 35, 37)
(16, 63, 65)
(20, 99, 101)
(24, 143, 145)
(28, 195, 197)

Problemas Propostos
4.6. Existem triplas pitag
oricas primitivas, tais que a diferenca entre a
hipotenusa e um cateto seja 3, 4, 5, 6, 7 ou 8? Caso existam em algum
dos casos, determine a f
ormula geral delas.


4.3. TRIPLAS PITAGORICAS
PRIMITIVAS

143

4.7. Dado r natural, encontre a famlia de todas as triplas pitag


oricas
primitivas tais que a diferenca entre a hipotenusa e um cateto seja r.
4.8. Encontre todas as triplas pitag
oricas tais que os lados est
ao em
progress
ao aritmetica.

4.3

Triplas Pitag
oricas Primitivas

Como falamos anteriormente, se a, b e c s


ao os comprimentos dos
lados de um tri
angulo ret
angulos e queremos caracterizar os tri
angulos
ret
angulos com lados inteiros, o problema e equivalente a encontrar todas
as triplas de n
umeros inteiros (a, b, c) tais que se verifique a relacao
a2 + b2 = c2 . Como ja foi observado anteriormente, tais triplas s
ao
denominadas triplas pitagoricas.
Para isto, suponhamos que temos um tripla pitagorica (a, b, c). Podemos supor que os tres n
umeros nao tem fatores em comum dois a dois,
porque, por exemplo, no caso em que a e b tem um fator comum d entao
d2 divide a2 + b2 = c2 , portanto d tambem divide c, assim ( ad , db , dc ) tambem e tripla pitagorica. Uma tripla pitagorica na qual seus termos nao
tem fator em comum dois a dois se denomina tripla pitag
orica primitiva.
Observemos que a e b nao podem ser simultaneamente mpares, ja
que se temos um n
umero mpar 2k + 1, ao elev
a-lo ao quadrado temos
que
(2k + 1)2 = 4k 2 + 4k + 1 = 4k(k + 1) + 1,
e logo o quadrado de um n
umero mpar e sempre congruente a 1 m
odulo
4. Assim, se somamos dois quadrados de n
umeros mpares obteremos um
n
umero que e congruente a 2 m
odulo 4, mas o quadrado de um n
umero
par e sempre divisvel por por 4. Portanto, temos que a e b nao podem
ser simultaneamente mpares. Podemos supor entao que a e mpar e b e
par, e, entao, c e mpar. Temos entao
b2 = c2 a2 = (c a)(c + a).
ca
ao podem
mas c e a nao tem fator em comum, donde u = c+a
2 e v = 2 n
ter fator em comum (caso contrario, se d divide simultaneamente u e v,
entao d tambem dividira a u + v = c e u v = a). Daqui segue que
b2 = uv, isto e, o produto uv e o quadrado de um n
umero inteiro. Mas
como u e v nao tem fator em comum, entao cada um deles tem que ser
o quadrado de um n
umero inteiro. Assim

c+a
= u = m2 ,
2

ca
= v = n2 ,
2

b = 2mn,


LINEARES
144 CAPITULO 4. EQUAC
OES
DIOFANTINAS NAO
onde m e n nao tem fator comum. Conclumos entao que todas as triplas
pitagoricas primitivas estao dadas pelas formulas
a = m2 n2 ,

b = 2mn

c = m2 + n2

com m e n sem fator comum e m + n mpar e todas as triplas pitagoricas


se encontram a partir de uma tripla pitagorica primitiva, multiplicando
por uma constante.
Observemos que o resultado encontrado na secao anterior, isto e, as
triplas pitagoricas encontradas por Pit
agoras, correspondem ao caso em
que m = k + 1 e n = k, e as triplas pitagoricos encontradas por Platao
se reduzem ao caso em que m = 2k e n = 1. A seguinte tabela ilustra
algumas solucoes novas nao encontradas pelo metodo anterior.
m
5
7
7
8
8
9
9

n
2
2
4
3
5
2
4

(a, b, c)
(21, 20, 25)
(45, 28, 53)
(33, 56, 65)
(55, 48, 73)
(39, 80, 89)
(77, 36, 85)
(65, 72, 97)

Como uma aplicacao do resultado anterior, consideremos o seguinte


Exemplo 4.4. Encontrar todas as triplas de n
umeros (a, b, c) tais que
a2 , b2 e c2 est
ao em progress
ao aritmetica.
Observemos que a tripla (1, 5, 7) e solucao do problema. Sabemos
que, em uma progress
ao aritmetica, a diferenca de dois termos consecutivos e constante; assim, o problema se reduz a encontrar todas as
triplas de n
umeros (a, b, c) tais que
b2 a 2 = c 2 b2 ,

isto e, a2 + c2 = 2b2 . Daqui segue que a e c tem a mesma paridade e,


portanto, existem inteiros r e s tais que c = r + s e a = r s (basta
ca
fazer r = c+a
2 e s = 2 ). Deste modo, substituindo teremos que
2b2 = a2 + c2 = (r s)2 + (r + s)2 = 2(r2 + s2 )
donde (r, s, b) e uma tripla pitagorica, portanto existem inteiros m e n
tais que r = m2 n2 , s = 2mn e b = m2 + n2 , e se conclui que
a = |m2 n2 2mn|,

b = m2 + n2 ,

c = m2 n2 + 2mn,

com m > n e m+n mpar, e uma tripla que satisfaz o pedido. Alem disso
todas as triplas primitivas s
ao desta forma. A seguinte tabela mostra os
primeiros valores de a, b e c com a2 , b2 e c2 em progress
ao aritmetica


4.3. TRIPLAS PITAGORICAS
PRIMITIVAS
m
2
3
4
4

n
1
2
1
3

145

(a, b, c)
(1, 5, 7)
(7, 13, 17)
(7, 17, 23)
(17, 25, 31)

Usando a mesma tecnica podemos resolver o seguinte


Exemplo 4.5. Determine todas as ternas (a, b, c) de inteiros positivos
tais que a2 = 2b + c4 .
Soluc
ao. Seja (a, b, c) uma de tais solucoes. Dado que
(a c2 )(a + c2 ) = 2b ,
existem dois naturais m > n tais que a c2 = 2n e a + c2 = 2m .
Substraindo tais equacoes, obtemos que 2c2 = 2m 2n , assim c2 =
2n1 (2mn 1). Segue que n 1 e par e que 2mn 1 = k 2 , mas
k 2 + 1 2 (mod 4), portanto m n = 1. Assim, todas as solucoes s
ao
2r
r
da forma (3 2 , 4r + 3, 2 ) com r N.
Exemplo 4.6. Encontrar todas as soluc
oes inteiras de x2 +y 2 +z 2 = t2 .

Soluc
ao. Observemos inicialmente (considerando congruencias m
odulo 4) que pelo menos dois de tais n
umeros tem que ser pares. Sejam
y = 2l e z = 2m. Como t > x, podemos supor que t = x + 2n, com n
inteiro positivo. Substituindo, teremos que (x + 2n)2 = x2 + 4l2 + 4m2 ,
2
2
2
isto e, nx + n2 = l2 + m2 . Obtemos assim que x = l +mn n . Como x
e inteiro
positivo, escolhemos n como sendo um divisor de l2 + m2 menor que l2 + m2 . Conclumos que todas as solucoes da equacao estao
dadas por
l2 + m2
l2 + m2
n, y = 2l, z = 2m, t =
+ n,
n
n

onde n e um divisor de l2 + m2 menor que l2 + m2 .


Outra representacao das solucoes desta equacao e proposta no problema 4.35.
x=

Problemas Propostos
4.9. Determine todas as soluc
oes inteiras da equac
ao 2x2 + y 2 = z 2 .
4.10. Existe algum tri
angulo ret
angulo com lados inteiros e permetro
2pk com p um n
umero primo e k inteiro?
4.11. Determine todos os tri
angulos ret
angulos com lados inteiros e permetro 120.


LINEARES
146 CAPITULO 4. EQUAC
OES
DIOFANTINAS NAO

4.4

Tri
angulos pitag
oricos e o m
etodo geom
etrico

Nesta secao mostraremos outra forma de encontrar a forma geral das


triplas pitagoricas com o uso de um pouco de geometria analtica. Para
isto basta observar que a2 + b2 = c2 e equivalente `a equacao ( ac )2 +
( cb )2 = 1, isto e, o ponto ( ac , cb ) e um ponto com coordenadas racionais
da circunferencia C com centro na origem e raio 1, ou, equivalentemente,
( ac , cb ) e uma solucao da equacao x2 + y 2 = 1 com coordenadas que s
ao
n
umeros racionais.
Note que os pontos (1, 0), (0, 1) s
ao pontos que pertencem `a circunferencia. Agora, se temos outro ponto sobre a circunferencia que tem
coordenadas racionais, digamos (r, s), entao a reta L que passa pelos
pontos (0, 1) e (r, s) corta a circunferencia exatamente nesses pontos e
tem inclinacao s+1
e um n
umero racional.
r , que
Agora pensemos ao contrario, peguemos uma reta L que passa por
a a circunferencia C no
(0, 1) e tem inclinacao racional m
n . Ela cortar
ponto (0, 1), em algum
outro
ponto
que
pode
ser
calculado resolvendo
(
2
2
x +y =1
o sistema de equacoes
y=m
n x 1,
y

y=

m
nx

( ac , cb )
x

(0, 1)
Assim, substituindo y por m
n x 1, obtemos
2
m
m2
m
x + 1 = x2 + 2 x2 2 x + 1.
1 = x2 +
n
n
n
A equacao anterior possui uma solucao esperada x = 0, A segunda
2
solucao e dada por (1 + m
e, x = m2mn
)x = 2 m
2 +n2 , e portanto
n , isto
n2
a
2mn
= 2
c
m + n2

b
m 2mn
m2 n2
=

1
=
,
c
n m2 + n2
m2 + n2

4.4. TRIANGULOS
PITAGORICOS
E O METODO
GEOMETRICO147
agora como as fracoes s
ao iguais, supondo que mdc(m, n) = 1, segue-se
que, no caso em que m + n e mpar, existe um inteiro k tal que
a = 2mnk,

b = (m2 n2 )k,

c = (m2 + n2 )k.

E, no caso em que m + n e par, existe um inteiro k tal que a = mnk,


2
2
2
2
b = (m 2n ) k e c = (m 2+n ) k.
O interessante deste metodo geometrico, e que permite resolver equacoes mais gerais de tipo quadratico quando conhecemos um ponto, como
se mostra no seguinte exemplo
Exemplo 4.7. Determinar todas as soluc
oes inteiras da equac
ao a 2 +
2b2 = 11c2 .
Dividindo por c2 obtemos ( ac )2 + 2( cb )2 = 11, assim queremos encontrar as solucoes racionais da equacao x2 + 2y 2 = 11 que e uma elipse
centrada na origem do plano cartesiano. Por inspecao direta temos que
os pontos (3, 1) s
ao solucoes. A equacao da reta que passa pelo ponto
m
3mn
ey=m
(3, 1) e tem inclinacao m
n
n (x + 3) 1 = n x +
n .
y=

m
nx

3mn
n

( ac , cb )

(3, 1)

Os pontos de interseccao da reta com a elipse, s


ao as solucoes obtidas
quando resolvemos o sistema de equacoes
(
x2 + 2y 2 = 11
3mn
y=m
nx+
n ,
agora, substituindo y na equacao da elipse obtemos que


m
3m n 2
2
11 = x + 2
x+
n
n
2
2
n + 2m 2
m(3m n)
9m2 6mn + n2
=
x
+
4
x
+
2
,
n2
n2
n2


LINEARES
148 CAPITULO 4. EQUAC
OES
DIOFANTINAS NAO
assim, x e solucao da equacao quadratica
x2 + 4

m(3m n)
3n2 + 4mn 6m2
x

3
= 0.
n2 + 2m2
n2 + 2m2

Como (3, 1) e um dos pontos de interseccao da elipse com a reta, o


leitor pode comprovar diretamente que x = 3 deve ser solucao desta
equacao. Agora usando o fato que o coeficiente independente de uma
equacao quadratica e o produto das razes, temos que a outra raiz e
3n2 + 4mn 6m2
a
=x=
,
c
n2 + 2m2
e assim
b
m
=y=
c
n

3n2 + 4mn 6m2


n2 + 2m2

2m2 + 6mn n2
3m n
=
.
n
n2 + 2m2

Como as fracoes que aparecem em cada igualdade s


ao iguais, entao existir
a um inteiro k tal que
a=

k
(3n2 +4mn6m2 ),
d

b=

k
(2m2 +6mnn2 )
d

e c=

k 2
(n +2m2 )
d

(onde d = mdc(3n2 + 4mn 6m2 , 2m2 + 6mn n2 , n2 + 2m2 ), que


pertence a {1, 2, 11}, caso mdc(m, n) = 1) e usando a simetria da elipse
podemos considerar os valores positivos de a, b e c. A tabela seguinte
ilustra algumas solucoes obtidas a partir das equacoes anteriores supondo
k=1
m
1
0
1
2
3
4
4

n
-1
1
1
1
1
1
3

(a, b, c)
(7, 5, 3)
(3, 1, 1)
(1, 7, 3)
(13, 19, 9)
(39, 37, 19)
(77, 55, 33)
(21, 95, 25)

Problemas Propostos
4.12. Determine todas as soluc
oes inteiras da equac
ao a2 + 3b2 = 13c2 .
4.13. Mostre que a equac
ao x2 + y 2 = 6z 2 n
ao possui soluc
oes inteiras
positivas.
Dica: Mostre que o quadrado de todo mpar deixa resto 1 quando dividido por 8, assim os quadrados quando divididos por 8 somente podem
deixar resto 0, 1 ou 4.

4.5. TRIANGULOS
COM LADOS INTEIROS E ANGULOS
EM PROGRESSAO
4.14. Demonstre que a equac
ao x2 + y 2 = 3z 2 n
ao tem soluc
oes inteiras
positivas.
4.15. Encontrar todas as soluc
oes inteiras da equac
ao x2 + y 2 = 5z 2 .
4.16. Um bloco retangular e chamado Bloco de Euler se o comprimento
de seus lados e inteiro e o comprimento das diagonais em cada face e
inteira. Um bloco de Euler e chamado primitivo se o mdc dos comprimentos dos lados e 1. Mostrar que um bloco com medidas 44, 117 e 240
e um bloco de Euler.
4.17. Mostrar que se um bloco com medidas a, b e c e de Euler, ent
ao
um bloco com medidas (bc, ab, ac) e de Euler.
4.18. Encontre um bloco de Euler cuja aresta de menor comprimento e
85.
4.19. Mostrar que existem infinitos blocos de Euler primitivos .
um problema aberto a existencia de um bloco de Euler com
Obs.: E
diagonal inteira.

4.5

Tri
angulos com lados inteiros e
angulos em
progress
ao aritm
etica

Como uma aplicacao do resultado da secao anterior, pretendemos


caracterizar todos os tri
angulos ABC com lados de comprimentos inteiros a, b e c que tenham seus
angulos em progress
ao aritmetica. Como
a soma dos angulos internos de um tri
angulo e 180o , se a razao da progress
ao e , entao os
angulos ter
ao medidas 60 , 60 e 60 + .
Suponhamos que o
angulo em B mede 60 e o angulo em A e o maior,
e seja D o pe da altura desde o vertice A. Observemos que o tri
angulo
BAC e de fato, meio tri
angulo equil
atero, assim BD = 2c .
Como os tri
angulos ADB e ADC s
ao ret
angulos, aplicando o
teorema de Pit
agoras temos que
c2

 c 2
2


c 2
= AD2 = b2 a
2

que equivale a
1
3 2
c = b2 a2 + ac c2 ,
4
4


LINEARES
150 CAPITULO 4. EQUAC
OES
DIOFANTINAS NAO
A

60
B

c
2

c
2

ou seja, os tri
angulos que tem os lados em progress
ao aritmetica s
ao os
tri
angulos que tem lados que satisfazem a relacao
a2 ac + c2 = b2 ,
que equivale usando o metodo geometrico, a determinar as solucoes racionais da equacao x2 xy + y 2 = 1, que representa no plano uma elipse
rotacionada. Observemos que esta equacao possui a solucao (0, 1).
Consideremos a intersecao da elipse com a reta y = m
n x 1.
y=

m
nx

( ab , cb )

x2 xy + y 2 = 1
(1, 1)

(0, 1)

A intersecao da reta com a elipse e formada por pontos cuja coordenada x satisfaz
(m2 mn + n2 )x2 (2m n)nx = 0.
Assim,
a
2mm n2
= 2
b
m mn + n2

c
m2 n2
= 2
.
b
m mn + n2

DE ANGULOS

4.6. OUTRA RELAC


AO

151

Conclumos que os comprimentos dos lados dos tri


angulos com um angulo de 60 s
ao
a = k(2mn n2 )

b = k(m2 mn + n2 )

c = k(m2 n2 ), k Z.

Fica como exerccio para o leitor completar a seguinte tabela, considerando k = 1.


m
2
3
3
4
4
5
5

n
1
1
2
2
3
1
2

(a, b, c)
(3, 3, 3)
(5, 7, 8)

Problemas Propostos
4.20. Mostre que n
ao existem tri
angulos com lados inteiros de tal forma
que um de seus
angulos seja 30 , 45 ou 72 .
4.21. Encontre todos os tri
angulos com lados inteiros e com um
angulo
2
tal que cos = 5 .
4.22. Seja 0 < < 180 , tal que cos e um n
umero racional. Mostre
que existem infinitos tri
angulos n
ao semelhantes e com lados inteiros
tais que um de seus
angulos mede .

4.6

Outra relac
ao de
angulos

Nesta secao mostraremos como determinar todos os tri


angulos com
lados inteiros, tais que um
angulo e o dobro do outro. Para isto, suponhamos que temos um tri
angulo ABC com lados de comprimentos a,
b e c e tal que A = 2B = 2. Tracemos a bissetriz desde o vertice A
e denotemos por D o pe da bissetriz no lado BC.
Pelas condicoes acima, segue que o tri
angulo BDA e isosceles, e,
assim, BD = DA = x. Por outro lado, o angulo ADC e angulo externo
ao tri
angulo BDA, logo ele e igual `
a soma dos angulos nao adjacentes,
donde ADC = 2.


LINEARES
152 CAPITULO 4. EQUAC
OES
DIOFANTINAS NAO
C
D

Assim, os tri
angulos ABC e DCA s
ao semelhantes, ja que seus
angulos s
ao iguais, de onde temos as relacoes entre os lados
x
ax
b
= =
.
a
c
b
Da primeira igualdade temos que x =
igualdade obtemos que
bc
a
2
b
a = a bc ,
=
a
b
ab

bc
a

e substituindo na segunda

assim b2 + bc = a2 .

Podemos resolver esta equacao pelo metodo geometrico ou simplesmente,


no caso em que mdc(b, c) = 1, observando que b(b + c) e um quadrado
perfeito, e que b e b + c nao tem fator em comum,
donde b e b + c tem
(
b = n2
que ser quadrados de n
umeros inteiros, logo
com m e n
b + c = m2
sem fator comum. Portanto, o conjunto de tri
angulos procurados e o
dos que satisfazem as relacoes
a = kmn,

b = kn2

e c = k(m2 n2 )

com n e m sem fator comum e k inteiro arbitr


ario.

Problemas Propostos
4.23. Determine todos os tri
angulos ABC com lados inteiros, A =
2B e c = 40.
4.24. Encontre uma f
ormula para o n
umero de tri
angulos ABC com
lados inteiros, A = 2B e lado c um inteiro dado (esta f
ormula deve
depender do n
umero de divisores de c).

4.7. CONTANDO TRIANGULOS


PITAGORICOS
COM UM CATETO DADO1

4.7

Contando tri
angulos pitag
oricos com um cateto dado

Dado um n
umero natural M , uma pergunta que podemos fazer e
quantos tri
angulos ret
angulos com lados inteiros tem M como um de
seus catetos. Para responder esta pergunta basta contar o n
umero de
2
2
formas que podemos escrever M como k(m n ) ou 2kmn satisfazendo
as condicoes: m e n nao tem fator comum e m + n e mpar. Primeiro
vamos supor que k = 1, isto e, contaremos somente triplas pitagoricas
primitivas. Podemos separar o problema em dois casos:
Se M e mpar, entao precisamos encontrar o n
umero de solucoes
de
M = m2 n2 = (m n)(m + n),
isto e, o n
umero de formas de fatorar M como produto de dois
fatores distintos, mas isto e exatamente
(
1
d(M ),
se M nao e um quadrado perfeito
C(M ) = 12
e um quadrado perfeito,
2 (d(M ) 1), se M
onde d e a funcao n
umero de divisores estudada no captulo 2.
Por exemplo se M = 15, temos que d(15) = 4, logo existem dois triangulos pitagoricos primitivos com 15 como lado. Para
( determinar
m + n = 15
estes tri
angulos, precisamos resolver os sistemas
mn=1
(
m+n=5
e
, que tem como solucoes (m, n) = (8, 7) e (m, n) =
mn=3
(4, 1), e que geram os tri
angulos com lados (15, 112, 113) e (15, 8, 17).
Observemos que desta forma nao geramos os tri
angulos com lados
(15, 20, 25) e (15, 36, 39) porque estes tri
angulos nao s
ao primitivos.
Se M e par, precisamos contar de quantas forma podemos escrever
M = 2mn com m e n de paridades diferentes. Assim, se M
e mpar,
2
nao existe nenhum tri
angulo pitagorico primitivo que tenha M
como lado. Escrevamos M = 2a+1 N onde N e mpar e a 1, assim
o n
umero de tri
angulos pitagoricos primitivos com M = 2a+1 N e
(
0,
se a = 0
a+1
C(2 N ) =
d(N ), se a > 0.
Por exemplo se M = 60, entao N = 15 e d(N ) = 4, isto e, 15 =
15 1 = 5 3, assim (m, n) pode tomar os valores (30, 1), (15, 2),


LINEARES
154 CAPITULO 4. EQUAC
OES
DIOFANTINAS NAO
(10, 3), ou (6, 5), que geram os tri
angulos com lados (899, 60, 901),
(221, 60, 229), (91, 60, 109) e (11, 60, 61).
Agora, para contar o n
umero de triplas pitagoricas que existem com um
cateto igual a M , consideraremos dois casos:
Se M e impar, para cada m um divisor de M e para cada tripla
pitagorica primitiva com um cateto igual a m e possvel gerar uma
tripla pitagorica com um cateto igual a M , multiplicando esta
termo da tripla por M
oricas e
m , assim o total de triplas pitag
X
X
X
C(m) +
C(n2 )
C(m) =
m|M

n2 |M

m|M
m6=n2

1 X
1 X
d(m) +
(d(n2 ) 1)
2 m|M
2 2
n |M

m6=n2

1 X
1 X
d(m)
1.
2
2 2
m|M

n |M

No caso que M = p1 1 p2 2 pk k , usando que a funcao d e multiP i


P i
( +2)( +1)
(j + 1) = j 2 j
d(pji ) = j=0
plicativa temos que j=0
e
portanto, o n
umero de tri
angulos pitagoricos com um cateto igual
a M e

k 
k
1  Y (j + 2)(j + 1) Y j + 2 

.
2
2
2
j=1

j=1

Se M e par, isto e, M = 2a+1 N (com a 0) onde N = p1 1 p2 2 pk k


e mpar, o n
umero de triplas pitagoricas com um cateto igual a M
e
a X
X
X
X
C(2j+1 m).
C(m) =
C(m) +
m|M

j=1 m|N

m|N

O primeiro somatorio ja foi calculado no item anterior e o segundo


e igual a
k
a X
Y
X
(j + 2)(j + 1)
,
d(m) = a
2
j=1 m|N

j=1

Portanto, o n
umero de tri
angulos pitagoricos com um cateto igual
a+1
a M = 2 N e

k
k 
1
Y
(j + 2)(j + 1) 1 Y j + 1
+a

.
2
2
2
2
j=1

j=1


SOMAS DE DOIS QUADRADOS
4.8. NUMEROS
QUE SAO

155

Problemas Propostos
4.25. Determine o n
umero de tri
angulos pitag
oricos com um cateto de
comprimento 60. Observe que existem exatamente 4 triplas pitag
oricas
primitivas em que um dos catetos vale 60.
4.26. Quantos tri
angulos pitag
oricos tem 2 3k como um de seus catetos?
4.27. Quantos tri
angulos pitag
oricos tem 3k 5l como um de seus catetos?

4.8

N
umeros que s
ao somas de dois quadrados

Vamos mostrar algumas condicoes sobre os divisores primos de um


n
umero que e soma de dois quadrados. Notemos, inicialmente, que 2 =
12 + 12 . Quanto aos primos mpares, temos a
Proposi
c
ao 4.8. Sejam a e b dois n
umeros inteiros tais que mdc(a, b) =
1 e seja p um n
umero primo mpar tal que p divide a2 +b2 , ent
ao p deixa
resto 1 quando dividido por 4.
o: Primeiro observemos que p nao divide nem a nem
Demonstra
ca
p1
b, e que a2 b2 (mod p). Elevando a` potencia p1
2 temos que a
(1)(p1)/2 bp1 (mod p), e pelo pequeno teorema de Fermat obtemos
e um n
umero par, isto e,
que (1)(p1)/2 1 (mod p). Portanto p1
2
p 1 (mod 4).
A seguinte identidade mostra que para determinar quais n
umeros
s
ao somas de dois quadrados, basta identificar quais primos s
ao soma de
dois quadrados.
Proposi
c
ao 4.9. Sejam m e n n
umeros que s
ao somas de dois quadrados, ent
ao mn tambem e soma de dois quadrados.
o: Suponhamos que m = a2 + b2 e n = c2 + d2 . Entao
Demonstra
ca
mn = (a2 + b2 )(c2 + d2 ) = a2 c2 + a2 d2 + b2 c2 + b2 d2
= (ac)2 + (ad)2 + (bc)2 + (bd)2 + 2abcd 2abcd

= ((ac)2 + 2abcd + (bd)2 ) + ((bc)2 2abcd + (ad)2 )

= (ac + bd)2 + (bc ad)2 .

Portanto, seu produto tambem sera soma de dois quadrados.


LINEARES
156 CAPITULO 4. EQUAC
OES
DIOFANTINAS NAO
Observa
c
ao 4.10. Pelo mesmo processo podemos obter que
mn = (ac + bd)2 + (bc ad)2 = (ac bd)2 + (bc + ad)2 ,
isto e, se m, n 6= 2 ent
ao mn tem no mnimo duas representac
oes como
soma de dois quadrados.
Em resumo, mostramos que todos os n
umeros primos mpares que
2
2
dividem a + b com mdc(a, b) = 1 s
ao da forma 4k + 1 e se dois n
umeros
s
ao soma de dois quadrados entao seu produto tambem e. Assim, seria
interessante provar o recproco desta afirmacao; mas, para isto, precisamos que todos os primos da forma 4k + 1 sejam soma de dois quadrados.
Pesquisando os primeiros casos, vemos que
5 = 22 + 12 , 13 = 32 + 22 , 17 = 42 + 12 , 29 = 52 + 22 ,
37 = 62 + 12 , 41 = 52 + 42 , 53 = 72 + 22 , 61 = 62 + 52 , etc
Assim, mostrando que todo primo da forma 4k+1 e soma de dois primos,
obteremos uma classificacao completa dos n
umeros que s
ao hipotenusa
de um tri
angulo ret
angulo com lados inteiros. Nessa secao mostraremos
usando ferramentas elementares que esta caracterizacao e verdadeira. Na
secao 4.12.1 mostraremos este mesmo fato usando os chamados inteiros
de Gau.
De fato, para mostrar esta afirmacao precisamos antes dos seguintes
resultados
Lema 4.11 (Lema de Thue). Se m e um n
umero natural e a e um
inteiro primo relativo com m, ent
ao existem n
umeros naturais x e y n
ao

nulos menores que m e tais que algum dos n


umeros ax + y ou ax y
e divisvel por m.
o: No caso em que m = 1, para qualquer valor de a
Demonstra
ca
teremos que x = y = 1 satisfazem as condicoes. Suponhamos que m seja

um n
umero natural maior que 1. Seja q = m. Entao q + 1 > m
e portanto (q + 1)2 > m. Consideremos todos os n
umeros da forma
ax y onde x e y tomam os valores 0, 1, . . . , q. Observe que estamos
considerando (q + 1)2 n
umeros. Como s
o existem m restos possveis ao
dividir um n
umero por m, entao existem dois desses n
umeros que tem
o mesmo resto quando divididos por m. Sejam ax1 y1 e ax2 y2 tais
n
umeros. Portanto sua diferenca a(x1 x2 ) (y1 y2 ) e divisvel por
m. Como

0 x1 < m e 0 x2 < m, entao m < x1 x2 < m ,


e da mesma forma

0 y1 < m e 0 y2 < m,

entao

m < y1 y2 <

m.


COMPRIMENTO DA HIPOTENUSA DE UM TRIANGULO

157

Se x1 x2 = 0, entao y1 y2 sera divisvel por m, o que implicara


que y1 = y2 , mas os pares (x1 , y1 ) e (x2 , y2 ) s
ao diferentes, logo isso
nos leva a uma contradicao. Da mesma forma, se y1 y2 = 0, entao
a(x1 x2 ) sera divisvel por m, mas a e m s
ao primos relativos, assim
m divide (x1 x2 ) e teremos x1 = x2 , contradicao. Assim, x1 x2 6= 0 e
y1 y2 6= 0. Podemos supor, sem perda de generalidade, que x1 x2 > 0
e, neste caso, tomamos x = x1 x2 e y = |y1 y2 |, e conclumos a prova
do lema.
Agora sim podemos enunciar e mostrar o teorema central desta secao
Teorema 4.12. Todo primo da forma 4k + 1 pode-se escrever como
soma de dois quadrados de inteiros positivos.
o: Seja p um n
Demonstra
ca
umero primo da forma 4k + 1. Pelo
teorema de Wilson temos que
(p 1)! 1 (mod p),
mas
12
ja que



p1
p1
p+1
(1)(2)
(p1)(p2)
2
2
2
p1
2

(mod p)

e par, portanto

 2
p1
! 1 (mod p).
2

Tomando a = ( 12 (p 1))! entao p divide a2 + 1 e aplicando o lema de

umeros ax y
Thue existem inteiros 0 < x, y < p tais que algum dos n
e divisvel por p, portanto o n
umero (ax + y)(ax y) = a2 x2 y 2 e
divisvel por p. Daqui, segue que
x2 + y 2 = a2 x2 + y 2 0 (mod p),

mas 0 < x, y < p, logo 0 < x2 + y 2 < 2p e portanto se conclui que


p = x2 + y 2 .
Podemos classificar, usando este teorema e a proposicao 4.8, os n
umeros que podem ser comprimento da hipotenusa de um tri
angulo retangulo com lados inteiros. Observemos que para N ser hipotenusa de
um tri
angulo temos que N deve ser da forma N = k(a2 + b2 ) onde a
e b s
ao inteiros sem fator comum e de diferente paridade. Assim, os
n
umeros que podem ser comprimento da hipotenusa de um tri
angulo
ret
angulo com lados inteiros s
ao exatamente os n
umeros que tem algum
fator primo congruente a 1 m
odulo 4.
O seguinte resultado mostra que a decomposicao de um primo como
soma de dois quadrados de fato e u
nica:


LINEARES
158 CAPITULO 4. EQUAC
OES
DIOFANTINAS NAO
Teorema 4.13. Seja n um n
umero primo da forma 4k + 1. Ent
ao n
pode ser escrito de forma u
nica como soma de dois quadrados.
o:
Demonstra
ca
Suponhamos, por outro lado, que n seja primo e que n = a2 +

2
b = c2 + d2 com a, b, c, d < n. Note que necessariamente temos
mdc(a, b) = mdc(c, d) = 1. Afirmamos que, a menos de ordem, estas
somas s
ao iguais. De fato, se a2 b2 (mod n) e d2 c2 (mod n),
entao (ad)2 (bc)2 (mod n), e como n e primo, temos que n divide
ad bc ou ad + bc.
No primeiro caso, sabemos que |ad bc| max{ad, bc} < n. Portanto, adbc = 0, isto e, ab = dc ; mas como as duas fracoes s
ao reduzidas,
temos que a = c e b = d. No caso em que n divide ad + bc, observemos
que n2 divide
(a2 + b2 )(c2 + d2 ) = (ad + bc)2 + (ac bd)2 .
Portanto n divide ac bd e, pelo mesmo argumento, obtemos que a = d
e b = c.
Como conclusao final desta secao, temos que os u
nicos n
umeros que
podem ser expressos como soma de dois quadrados de inteiros s
ao os da
s
2
forma n = 2 d l onde s e d s
ao naturais e l e um produto de primos da
forma 4k + 1.

Problemas Propostos
4.28. Escrever 73, 89 e 97 como somas de dois quadrados.
4.29. Escrever 145 e 187 como somas de dois quadrados de duas formas
distintas.
4.30. De quantas formas distintas pode-se escrever 10012 como soma de
dois quadrados?
4.31. Seja p um n
umero primo tal que a congruencia c2 2 mod p
possui soluc
ao. Mostre que existem inteiros a e b tais que p = a2 + 2b2 .
4.32. Demonstre que todo primo da forma 6k + 1 pode se expressar de
forma u
nica como x2 + 3y 2 , com x e y inteiros positivos.
4.33. Mostre que os n
umeros da forma 4k (8n+7) n
ao podem ser escritos
como soma de tres quadrados.
4.34 (Scholz). Prove a seguinte generalizac
ao do lema de Thue: Sejam
n um n
umero natural positivo e r e s n
umeros naturais tais que rs > n
com 1 < r, s < n. Ent
ao, para todo a com (a, n) = 1, a congruencia
ay x (mod n) tem soluc
ao inteira com 0 < x < r e 0 < y < s.

4.9. TRIANGULOS
PITAGORICOS
COM CATETOS CONSECUTIVOS159
4.35. Demonstrar que todas as soluc
oes inteiras de x2 + y 2 + z 2 = t2 ,
est
ao dadas pelas equac
oes
x = d(m2 n2 p2 + q 2 ), y = d(2mn 2pq),
z = d(2mp + 2nq), t = d(m2 + n2 + p2 + q 2 ),
com d, m, n, p, q inteiros.

4.9

Tri
angulos pitag
oricos com catetos consecutivos

O problema de determinar todos os tri


angulos com a hipotenusa e
um cateto consecutivos foi resolvido totalmente por Pit
agoras. Agora
observemos que os tri
angulos com lados (3, 4, 5) e (20, 21, 25) tem seus
catetos consecutivos. O prop
osito desta secao e determinar todos os
tri
angulos com os catetos consecutivos. Se os lados tem comprimentos
a, a + 1 e c, temos a equacao
2a2 + 2a + 1 = c2 ,
a qual, multiplicando por 2 e completando quadrados, equivale a
(2a + 1)2 2c2 = 1.
Assim, o problema e equivalente a encontrar todas as solucoes inteiras
positivas da equacao x2 2y 2 = 1. Antes de encontrar as solucoes
desta equacao estudaremos uma equacao mais geral, conhecida como
equacao de Pell.

4.9.1

Equa
c
ao de Pell

Uma equacao de Pell e uma equacao da forma x2 dy 2 = 1, onde


d e um n
umero natural que nao e um quadrado perfeito. Por exemplo,
algumas solucoes da equacao x2 3y 2 = 1 s
ao
(2, 1), (7, 4), (26, 15), (101, 56), . . . .
As solucoes desta equacao s
ao exatamente os pontos de coordenadas
inteiras da hiperbole x2 3y 2 = 1, como mostra a figura


LINEARES
160 CAPITULO 4. EQUAC
OES
DIOFANTINAS NAO
y

Suponhamos que conhecemos uma solucao(x1 , y1 ) com x1 e y1 inteiros positivos (encontrada testando alguns valores ou pelo metodo apresentado na seguinte secao). Vejamos como gerar infinitas solucoes da
equacao de Pell a partir desta solucao (x1 , y1 ). Como x21 dy12 = 1,
fatorando a parte da direita e elevando `
a potencia n temos
n
n
(x1 y1 d) (x1 + y1 d) = (x21 dy12 )n = (1)n .
Cada um destes fatores pode ser expandido usando o bin
omio de Newton
n  
X
n

n ni
(x1 y1 d) =
x1 (y1 d)i
i
i=0

X n
X n

i
ni
=
x1ni (y1 d)i .
x1 (y1 d)
i
i
0in
0in
i mpar

i par

2 de forma que x + y
d = (x1 +
Portanto,
definindo
(x
,
y
)

Z
n
n
n
n

y1 d)n , ou seja, tomando


n

xn =

2  
X
n
i=0

2i

xn2i
di y12i
1

e yn =


n1
2

X
i=0


n
xn2i1 di y12i+1 ,
2i + 1 1

temos tambem xn yn d = (x1 y1 d)n , e obtemos que x2n dyn2 =


(1)n . Assim, a partir de uma solucao nao trivial da equacao de Pell
podemos gerar infinitas solucoes.
Estas mesmas solucoes podem ser obtidas a partir de uma formula
recursiva. Para chegar a tal formula vejamos que

xn+1 + yn+1 d = (x1 + y1 d)n+1

= (x1 + y1 d)n (x1 + y1 d)

= (xn + yn d)(x1 + y1 d),

4.9. TRIANGULOS
PITAGORICOS
COM CATETOS CONSECUTIVOS161
e igualando parte racional com parte racional e parte irracional com
parte irracional, temos as relacoes
(

xn+1 = x1 xn + dy1 yn
yn+1 = x1 yn + y1 xn .

Por exemplo, a equacao x2 5y 2(= 1 tem como solucao (2, 1), asxn+1 = 2xn + 5yn
sim usando as equacoes recursivas
, obtemos o par
yn+1 = 2yn + xn
(x2 , y2 ) = (9, 4), que e solucao da equacao x2 5y 2 = (1)2 = 1 e o
par (x3 , y3 ) = (38, 17) que e solucao da equacao original. Em outras
palavras, os termos de ordem mpar desta sequencia, (2, 1), (38, 17),
(682, 305), (12238, 5473) etc, s
ao solucoes da equacao x2 5y 2 = 1 e os
termos de ordem par da sequencia, que s
ao (9, 4), (161, 72), (2889, 1292),
(51841, 23184) etc, s
ao solucoes da equacao x2 5y 2 = 1.
Por outro lado, dentre todas as solucoes (x, y) N2 da equacao
de Pell x2 dy 2 = 1 com x + y d > 1, existe uma soluc
ao mnima
ou fundamental, i.e., com x + y d mnimo e como consequencia x e
y mnimos. Denote por (x1 , y1 ) esta solucaomnima. Se,
como antes,
definimos (xn , yn ) N2 pela relacao xn + yn d = (x1 + y1 d)n , temos
que (xn , yn ), n 1, s
ao todas as solucoes inteiras positivas da equacao
2
2
de Pell x dy = 1: de fato, ja vimos que (x
ao solucoes, e se
n , yn ) s
(a, b) e uma outra solucao, entao como x1 + y1 d > 1, existe n 0 tal
que

(x1 + y1 d)n a + b d < (x1 + y1 d)n+1 .

Multiplicando por xn yn d = (x1 + y1 d)n > 0, obtemos

1 (a + b d)(xn yn d) = (axn byn d) + (bxn ayn ) d

< x1 + y1 d.

Como
(axn dbyn )2 d(bxn ayn )2 = (a2 db2 )(x2n dyn2 ) = 1,
temos que (axn dbyn , bxn ayn ) tambem e uma solucao da equacao
de Pell, menor que a solucao mnima. Temos que axn dbyn 0, pois
caso contrario axn dbyn < 0 ab xynn < d, porem
x2n dyn2 = 1 =

 x 2
n

yn

=d+

1
xn
> d
> d =
2
yn
yn


LINEARES
162 CAPITULO 4. EQUAC
OES
DIOFANTINAS NAO

e analogamente ab > d, o que contradiz ab xynn < d. Da mesma forma,


bxn ayn 0, pois caso contrario,
 x 2  a 2
1
a
1
xn
n
<
=d+ 2
<
= d + 2 =
yn
b
yn
yn
b
b
= b < yn = a < xn

o que contradiz o fato de xn + yn d = (x1 + y1 d)n a + b d. Resumindo, temos que (axn dbyn , bxn ayn ) N2 e uma solucao menor
do que a solu
n = 0, ou
cao mnima,
logo axn dbyn =1 e bxn ay
seja, (a + b d)(x1 y1 d)n = 1 a + b d = xn + yn d, donde
(a, b) = (xn , yn ), como queramos.
Voltando ao problema inicial de encontrar todos os tri
angulos ret
angulos com lados inteiros tal que os catetos sejam inteiros consecutivos,
isto e, com lados a, a + 1 e hipotenusa c tinhamos obtido a relacao
(2a + a)2 2c2 = 1. Portanto, este problema se reduz a
estudar a
2
2
equacao x 2y = 1 que tem solucao fundamental 1 + 2, assim
todas as solucoes s
ao da forma
2n+1

(1 + 2)
+ (1 2)2n+1
(1 + 2)2n+1 (1 2)2n+1 )

xn =
e yn =
.
2
2 2
Conclumos que todas as solucoes s
ao da forma

(1 + 2)2n+1 + (1 2)2n+1 2
,
an =
4
bn = an + 1,

(1 + 2)2n+1 (1 2)2n+1 )

cn =
2 2
para todo n N. As primeiras solucoes s
ao mostradas na tabela a seguir
n
1
2
3
4
5
6

an
3
20
119
696
4059
23660

bn
4
21
120
697
4060
23661

cn
5
29
169
985
5741
33461

Exemplo 4.14. Mostre que existem infinitos valores de n tais que a


soma dos primeiros n inteiros positivos e igual ao quadrado de um inteiro.
. O problema se reSoluc
ao. Observemos que 1 + 2 + + n = n(n+1)
2
n(n+1)
2
duz a encontrar inteiros (n, m) tais que 2 = m , o que e equivalente

4.9. TRIANGULOS
PITAGORICOS
COM CATETOS CONSECUTIVOS163
a n(n + 1) = 2m2 . Multiplicando a equacao anterior por 4 e completando quadrados obtemos a equacao (2n + 1)2 8m2 = 1. Assim basta
encontrar as solucoes da equacao de Pell x2 8y 2 = 1. Esta equacao tem
como solucao fundamental (3, 1), e portanto possui infinitas solucoes. As
seguintes tres solucoes podem ser calculadas facilmente, pois

8)2 = 17 + 6 8,

(3 + 8)3 = 99 + 35 8

(3 + 8)2 = 577 + 204 8.


(3 +

Assim 2n + 1 pode ser 3, 17, 99, 577, . . . , logo n = 1, 8, 49, 288, . . . .


A seguinte proposicao mostra uma recorrencia simples de segunda
ordem que gera todas as solucoes da equacao de Pell a partir das duas
primeiras.

Proposi
c
ao 4.15. Seja a + b d a soluc
ao fundamental da equac
ao de
2
2
Pell x dy = 1. Se {(xn , yn )}nN e a sequencia de soluc
oes da equac
ao,
ent
ao
xn+2 = 2axn+1 xn

yn+2 = 2ayn+1 yn ,

para todo n 1.
o:
Demonstra
ca

Como ja foi mostrado (


anteriormente temos que a
xn+1 = axn + dyn
sequencia de solucoes satisfaz a recorrencia
. Asyn+1 = bxn + ayn .
sim temos que
xn+2 = axn+1 + dbyn+1
= axn+1 + db(bxn + ayn ) = axn+1 + db2 xn + a(dbyn )
= axn+1 + db2 xn + a(xn+1 axn ) = 2axn+1 (a2 db2 )xn
= 2axn+1 xn ,

o que prova a primeira recorrencia. A segunda recorrencia e obtida de


modo an
alogo e e deixada como exerccio ao leitor.
Se aplicarmos estas formulas no problema anterior, podemos comprovar diretamente que 577 = 2 3 99 17 e 204 = 2 3 35 6, assim nao
e necessario calcular a expans
ao dos bin
omios para calcular as solucoes
de forma recursiva.


LINEARES
164 CAPITULO 4. EQUAC
OES
DIOFANTINAS NAO

Problemas Propostos
4.36. Observe que 83 73 = 132 . Mostre que existem infinitos pares de
cubos consecutivos tais que sua diferenca e um quadrado perfeito.
4.37. Mostre que a soma de tres quadrados consecutivos n
ao pode ser
igual a um quadrado. Pode a soma de cinco quadrados consecutivos ser
igual a um quadrado?

4.10

Solu
c
ao fundamental da equac
ao de Pell

2
2
Para determinar uma solucaoda equa
cao x Ay = 1, vamos
considerar a fracao contnua de A + A = [a0 ; a1 , a2 , . . .]. Mostraremos que existem duas sequencias de inteiros positivos bi e ci de
modo que

A + ci
= [ai ; ai+1 , ai+2 . . .]
bi
para todo i 0, onde os bi e ci s
ao limitados por uma constante que
depende unicamente de A, assim em algum momento tais sequ
encias
comecam a se repetir. Comecamos definindo b0 = 1 e c0 = A. Em
geral, definimos recursivamente ci+1 = ai bi ci e bi+1 = (A c2i+1 )/bi .
Mostremos inicialmente por inducao que bi e ci s
ao inteiros com
bi 6= 0 e tais que bi | A c2i para todo i. Isto e claramente verdade
para i = 0. Por hip
otese de inducao, temos que bi e ci s
ao inteiros, logo
ci+1 = ai bi ci tambem sera inteiro e A c2i+1 6= 0 ja que A nao e
quadrado perfeito. Alem disso,

A c2i+1 = A (ai bi ci )2 = A c2i bi (a2i bi 2ai ci )


sera m
ultiplo de bi ja que bi | A c2i por hip
otese de inducao. Assim
bi+1 = (A c2i+1 )/bi sera um inteiro nao nulo tal que bi+1 | A c2i+1 .
Desta forma, temos

A + ci
A ci+1
bi+1
1
= ai +
= ai +
= ai +
bi
bi
A + ci+1
A + ci+1
bi+1

i+1
de modo que A+c
= [ai+1 ; ai+2 , ai+3 . . .], e portanto sera valida para
bi+1
todo i. Resta provar que bi e ci s
aopositivos. Para isto, vamos provar
por induca
o que bi > 0 e 0 < ci < A, o que e verdadeiro para i = 0
pois c0 = A e A nao e quadrado perfeito. Alem disso, pela definicao
de ai temos

A + ci
ai <
= [ai ; ai+1 , ai+2 . . .] < ai + 1
bi

FUNDAMENTAL DA EQUAC
DE PELL
4.10. SOLUC
AO
AO
donde obtemos ai bi <
inducao) e portanto

165

A + ci < ai bi + bi (ja que bi > 0 por hip


otese de

ci+1 = ai bi ci < A < ai bi ci + bi = ci+1 + bi

e assim ci+1 < A, o que implica bi+1 = (A c2i+1 )/bi > 0 tambem.
Agora suponha

por absurdo que ci+1 0. Neste caso teramos bi >


A ci+1 A, mas como A > ci por hip
otese de inducao, teramos
bi > ci , donde ci+1 = ai bi ci bi ci > 0, o que e uma contradicao.
Portanto ci+1 > 0, completando
a inducao.

Como 0 < ci < A e bi | A c2i , temos que as sequencias {ci }


e {bi } s
o assumem um n
umero finito de valores. Alem disso, como
bi = (A c2i+1 )/bi+1 e ci = ai bi ci+1 podemos recuperar os valores de
bi e ci a partir dos de bi+1e ci+1 .
Portanto estas duas sequencias, assim
ao peri
odicas
como a fracao contnua A + A = [a0 ; a1 , a2 , . . .], s
puras, digamos de perodo
k. Em particular bk = 1 e ck = a0 .
Noteque como a0 = 2 A, temos que a expans
ao em fracao contnua de A e [a0 /2; a1 , a2 , . . .]. Logo, para i 1, denotando por pi /qi a
i-esima convergente desta fracao contnua, temos

A=

A+ci+1
bi+1 pi

A+ci+1
bi+1 qi

+ pi1

+ qi1

e portanto

Aqi + ci+1 Aqi + Abi+1 qi1 = Api + ci+1 pi + bi+1 pi1 .


Separando parte racional da parte irracional obtemos as equacoes
Aqi = ci+1 pi + bi+1 pi1

pi = ci+1 qi + bi+1 qi1 .

Isolando ci+1 nas equacoes anteriores e igualando obtemos


pi bi+1 qi1
Aqi bi+1 pi1
=
pi
qi
2
2
Aqi bi+1 pi1 qi = pi bi+1 qi1 pi
p2i Aqi2 = bi+1 (pi qi1 pi1 qi )
p2i Aqi2 = (1)i+1 bi+1

donde obtemos uma solucao da equacao x2 Ay 2 = (1)i+1 bi+1 . Se k e


o perodo teremos que bk = 1 e portanto a equacao x2 Ay 2 = 1 tem
solucao se k e mpar, enquanto que x2 Ay 2 = 1 sempre tem solucao
(tomando i + 1 = 2k).
Exemplo 4.16. Determine a soluc
ao mnima da equac
ao x2 13y 2 = 1.


LINEARES
166 CAPITULO 4. EQUAC
OES
DIOFANTINAS NAO

o: Sejam [a0 ; a1 , a2 , a3 , . . . ] a fracao contnua de 0 = 13 + 3.


Solu
ca
Os valores aj podem ser calculados recursivamente usando as relacoes
1
aj = j , j+1 = j a
. Assim neste caso
j
j

aj

j
13 + 3

13+3
4
13+1
3
13+2
3
13+1
4

13 + 3

Observemos que alinha 0 e igual `


a linha 5,
umeros vao se
assim os n
repetir, portanto 13 + 3 = [6; 1, 1, 1, 1] e 13
= [3; 1, 1, 1, 1, 6], assim
podemos calcular os primeiro 5 convergentes de 13 obtendo
p0
3
=
q0
1

p1
4
=
q1
1

p2
7
=
q2
2

p3
11
=
q3
3

p4
18
=
q4
5

logo p24 13q42 = 182 13 52 = 1 e a solucao fundamental da equacao


x2 13y 2 = 1. Para obter a solucao fundamental de x2 13y 2 = 1
basta calcular o decimo convergente, de fato
119
p5
=
q5
33

p6
137
=
q6
38

p7
256
=
q7
71

p8
393
=
q8
109

p9
649
=
,
q9
180

segue que 6492 13 1802 = 1 e a solucao mnima desta equacao.


Observemos
que podemos chegar a este mesmo valor
pegando a solu
cao

18 + 5 13 e elevando ao quadrado, isto e, (18 + 5 13)2 = 649 + 180 13.

Problemas Propostos
4.38. Calcule a a soluc
ao fundamental da equac
ao de Pell x2 dy 2 = 1,
onde d = a2 2 e d = a2 a para a inteiro arbitr
ario.

4.39. Determine a frac


ao contnua de a2 + 2, e determine para que
valores de a a equac
ao x2 (a2 + 2)y 2 = 1 tem soluc
ao.
4.40. Determine a soluc
ao fundamental da equac
ao x2 31y 2 = 1.
4.41. Determine a soluc
ao fundamental da equac
ao x2 41y 2 = 1.


4.11. OUTRAS EQUAC
OES
DO TIPO X 2 AY 2 = C

4.11

167

Outras equac
oes do tipo x2 Ay 2 = c

Foi mostrado que a equacao x2 Ay 2 = 1 sempre possui solucao.


Por outro lado a equac
ao x2 Ay 2 = 1 s
o possui solucao se o perodo
da fracao contnua de A tem comprimento mpar. O seguintes teoremas respondem de forma simples quando esta equacao tem ou nao tem
solucao para alguns valores particulares de A.
Teorema 4.17. Se A tem um divisor primo da forma 4k + 3, ent
ao a
2
2
equac
ao x Ay = 1 n
ao tem soluc
ao inteira positiva.

o: Seja p um n
Demonstra
ca
umero primo da forma 4k + 3 que divide
A. Se esta equacao tiver solucao, entao a congruencia x2 x2 Ay 2
1 (mod p) teria solucao. Mas elevando `a potencia p1
2 , obtemos que
xp1 (1)(p1)/2 1 (mod p), logo pelo teorema de Fermat obtemos que 1 1 (mod p), o que e contradit
orio.
Teorema 4.18. Seja p um primo da forma 4k + 1. Ent
ao a equac
ao
x2 py 2 = 1 sempre possui soluc
ao.

o: Suponhamos que (a, b) e solucao fundamental da


Demonstra
ca
2
equacao x py 2 = 1. Como a2 pb2 a2 b2 1 (mod 4) temos que
a e mpar e b e par, e como a2 1 = pb2 temos que
 2
b
a+1a1
=p
.
2
2
2
Como os n
umeros
dades:

a+1
2

a1
2

s
ao primos entre si, temos duas possibili-

a + 1 = 2f 2 e a 1 = 2pg 2 . Subtraindo estas duas equacoes


obtemos f 2 pg 2 = 1, isto e, o par (f, g) tambem e solucao de
x2 py 2 = 1 menor que a solucao (a, b), o que e impossvel ja que
esta e a solucao fundamental, logo este caso nao pode acontecer.
a + 1 = 2pf 2 e a 1 = 2g 2 . Subtraindo estas duas equacoes
obtemos g 2 pf 2 = 1, como queramos mostrar.
Exemplo 4.19. Sabendo que x2 29y 2 = 1 possui como soluc
ao fundamental x = 9801 e y = 1820, determine a soluc
ao fundamental de
x2 29y 2 = 1.

o: Pela prova do teorema anterior sabemos que a equacao


Solu
ca
2
x 29y 2 = 1 possui uma solucao (g, f ) com 2g 2 q
= 9801 1 = 9800 e

2
169 = 13.
229f = 9801+1, portanto g = 4900 = 70 e f = 9802
58 =


LINEARES
168 CAPITULO 4. EQUAC
OES
DIOFANTINAS NAO
A seguinte proposicao ajuda a reduzir o trabalho necessario para
decidir se a equacao mais geral x2 Ay 2 = c possui solucoes inteiras
positivas.

Proposi
c
ao 4.20. Seja = x1 + y1 A > 1 onde (x1 , y1 ) e a soluc
ao
2
2
mnima de x Ay = 1. Dado c Z n
ao nulo, se existem
x, yp N
com x2 Ay 2 = c, ent
ao existem u, v N com u + v A p |c|
2
e u2 Av
ao 0 u |c| e
p = c (em particular, para esta soluc
0 v |c|/A).

o: Se = r+s A com r, s Q, definimos = rs A


Demonstra
ca
e N () = N (
) = = r2 As2 .
Seja = x+y A > 0, com N () = x2 Ay 2 = c. Entao N ( k ) =
c para
Z tal
p todo k Z. Empparticular podemospescolher um kk p
que |c| < k |c|. No p
caso que |c| < p

|c|
definimos = k e no caso que |c| < k |c|, podemos
1k onde = c {1, 1}, assim
definir = |c|/( k ) =
|c|
p
p
= N () = c e |c| < |c|. Logo, sem perda de
N () = N ()
p
p
generalidade, podemos supor que |c| |c|.
Assim temos que = u + v A com u, v Z e ainda precisamos
verificar que u, v s
ao naturais, mas

c = N () = u2 Av 2 = (u + v A)(u v A).
Temos entao
p

|c|
|c|
p = |c| u + v A.
u+v A
|c|

Temos
assim u v A u + v A, donde v 0 e simultaneamente
u + v A u + v A, e logo u 0.

|u v A| =

Exemplo 4.21. Determine se a equac


ao x2 10y 2 = 39 possui soluc
ao.
o: A equacao x2 10y 2 = 1 possui como
Solu
ca
q solucao fundamental
q

39(19+6 10)
19 + 6 10. Como 39(19 + 6 10) < 39 e
< 13, pela
10
2
proposicao anterior, e do fato que x > 39, temos que uma possvel
solucao satisfaz 7 x 39 com x mpar e y 12, assim s
o precisamos
testar 12 valores para y. De fato com y = 1, obtemos x = 7, e portanto
a equacao possui solucao.
Exemplo 4.22. Mostre que a equac
ao x2 10y 2 = 11 n
ao possui soluc
ao
inteira.

4.12. CONTANDO TRIANGULOS


PITAGORICOS
COM HIPOTENUSA FIXA
o: Pelo mesmo processo anterior temos que
Solu
ca
s

11(19 + 6 10)
11(19 + 6 10) < 21 e
< 7.
10
Considerando a equacao m
odulo 4 obtemos x2 + 2y 2 3 (mod 4), mas
esta relacao somente e possvel quando x e y s
ao mpares, portanto, se
a equacao tem solucao, uma das solucoes deve satisfazer 4 x 20,
y 6 com x e y mpares, assim s
o precisamos testar y = 1, 3, 5, mas
como nenhum dos n
umeros 11 + 10 = 21, 11 + 90 = 101 e 11 + 250 = 261
e um quadrado perfeito, conclumos que a equacao nao possui solucoes
inteiras.

Problemas Propostos
4.42. Determine se a equac
ao x2 19y 2 = 21 possui soluc
ao inteira.
4.43. Determine todos os |c| < 10 tais que x2 17y 2 = c possui soluc
ao
inteira.
4.44. Usando o fato de que a equac
ao x2 91y 2 = 1 tem como soluc
ao
fundamental o par (1574, 165), mostre que a equac
ao 7x2 13y 2 = 1 n
ao
possui soluc
oes inteiras.

4.12

Contando tri
angulos pitag
oricos com hipotenusa fixada

Para todo c n
umero natural, denotemos por S2 (c) o n
umero de pares
de naturais (m, n) com mdc(m, n) = 1, m < n e m + n mpar tais que
c = m2 + n2 (note que, se S2 (c) 6= 0, entao n e mpar) e denotemos
por T2 (c) o n
umero de triplas de naturais (k, m, n) com mdc(m, n) = 1,
1 m < n e m + n mpar tais que c = k(m2 + n2 ), isto e S2 (c) e
o n
umero de triplas pitagoricas primitivas com hipotenusa c e T2 (c) e
o n
umero de triplas pitagoricas com hipotenusa c. Denotemos ainda
por U2 (c) o n
umero de pares de naturais (m, n) com m n tais que
c = m2 + n2 . Pelo que foi mostrado anteriormente, se c possui um
fator primo congruente a 3 (mod 4) entao S2 (k) = 0. Por outro lado,
se p e um primo congruente a 1 (mod 4) pelo teorema 4.13 temos que
T2 (p) = S2 (p) = 1.
Para determinar uma formula fechada para S2 (c) e T2 (c), precisaremos introduzir alguns resultados sobre inteiros de Gau. Antes de fazer
isso, resolvamos o seguinte problema:


LINEARES
170 CAPITULO 4. EQUAC
OES
DIOFANTINAS NAO
Exemplo 4.23. Determine todas as triplas pitag
oricas com hipotenusa
325.
o: Sabemos que cada tripla de inteiros k,m e n com mdc(m, n) =
Solu
ca
1 e m+n mpar gera uma u
nica tripla pitagorica (a, b, c) com a = 2kmn,
2
2
b = k(m n ) e c = k(m2 + n2 ). Assim precisamos encontrar triplas
(k, m, n) tais que k(m2 + n2 ) = 325 = 52 13. Como m2 + n2 e um
divisor de 325 distinto de 1, precisamos encontrar as solucoes de
m2 + n2 {5, 13, 25, 65, 325}.
De fato, 12 +22 = 5, 22 +32 = 13, 32 +42 = 25, 12 +82 = 65, 42 +72 = 65,
62 + 172 = 325 e 12 + 182 = 325, s
ao as u
nicas solucoes, que geram as
triplas pitagoricas
(195, 260, 325), (125, 300, 325), (91, 312, 325),
(80, 315, 325), (165, 280, 325), (204, 253, 325), (36, 323, 325)

4.12.1

Inteiros de Gau

Os inteiros de Gau s
ao os elementos do conjunto
def

Z[i] = {a + bi C | a, b Z}
subconjunto dos n
umeros complexos (onde i2 = 1). Este conjunto e
fechado para as operacoes soma e produto de n
umeros, isto e, soma de
dois interios de Gau e um inteiro de Gau e produto de inteiros de Gauss
e um inteiro de Gau. Alem disso e com respeito a soma, todo elemento
tem inverso aditivo, assim este conjunto junto com estas operacoes e
chamado anel dos inteiros de Gau. Este anel cumpre propriedades
equivalentes ao anel de inteiros Z como mostraremos nesta secao. Em
particular, podemos definir um algoritmo da divisao, e todo n
umero
pode-se fatorar como produto de primos, assim neste anel podemos
desenvolver uma aritmetica similar `
a aritmetica dos n
umeros inteiros.
Definimos a norma de um inteiro de Gau como
N : Z[i] Z

z = a + bi 7 |z|2 = zz = a2 + b2 .
Como foi verificado anteriormente a norma cumpre que N (wz) = N (w)N (z)
para todo w, z Z[i], isto e, N e uma funcao multiplicativa.

4.12. CONTANDO TRIANGULOS


PITAGORICOS
COM HIPOTENUSA FIXA
Inteiros de Gau possuem propriedades aritmeticas muito similares
a`s dos inteiros. Por exemplo, podemos definir divisibilidade da maneira
usual:
| existe Z[i] tal que = .
Assim, por exemplo, temos que 1 + i | 5 + 3i pois 5 + 3i = (1 + i)(4 i).
Note que | = N () | N (). Mais geralmente, temos
Lema 4.24 (Divisao Euclidiana). Sejam , Z[i] com 6= 0. Ent
ao
existem inteiros de Gau q, r Z[i] tais que
= q + r

com

N (r) < N ().

o: Escreva = x + yi com x, y Q. Agora sejam


Demonstra
ca
m, n Z os inteiros mais proximos de x e y, ou seja, m e n s
ao tais que
|x m| 21 e |y n| 12 . Agora basta tomar q = m + ni e r = q,
pois temos
2

1 1


q = |(x m) + (y n)i|2 = (x m)2 + (y n)2 + < 1.

4 4

Multiplicando por ||2 , temos portanto

N (r) = | q|2 < ||2 = N ().

Note que, ao contrario da divisao euclidiana em inteiros, o quociente e


o resto na divisao em Z[i] nem sempre estao unicamente determinados.
Por exemplo, dividindo-se = 5 por = 1 + i, temos mais de uma
possibilidade:
5 = (1 + i)(2 2i) + 1
5 = (1 + i)(2 3i) + i

com 1 = N (1) < N (1 + i) = 2


com 1 = N (i) < N (1 + i) = 2.

Exemplo 4.25. Escreva 21 i como produto de dois inteiros de Gau


n
ao triviais.
o: Queremos encontrar a, b, c e d tais que (a + ib)(c + id) =
Solu
ca
21 i, logo utilizando a funcao norma temos que (a2 + b2 )(c2 + d2 ) =
212 + 12 = 442 = 17 26. Fazendo a2 + b2 = 17 e c2 + d2 = 26, obtemos
solucoes para este problema. Dentre estas solucoes, temos a + ib = 4 i
e c + id = 5 + i que fornecem uma solucao do problema original, pois
21 i = (4 i)(5 + i).


LINEARES
172 CAPITULO 4. EQUAC
OES
DIOFANTINAS NAO
Ja que existe um algoritmo de divisao em Z[i] podemos definir mdc
de dois elementos de Z[i]. Em particular, dizemos que dois n
umeros
em Z[i] s
ao primos entre si se os seus u
nicos divisores comuns s
ao os
elementos invertveis de Z[i]. Em geral temos a seguinte
Defini
c
ao 4.26. Seja A um domnio, isto e, um anel em que (ab =
0 = a = 0 ou b = 0). Dizemos que um elemento u A e uma
unidade (ou um elemento invertvel) se ele possui inverso multiplicativo
em A, isto e, existe v A tal que uv = 1. O conjunto de todas as
unidades de A com a operac
ao de produto e um grupo multiplicativo, o
grupo de unidades de A, que denotamos por A .
Por exemplo, em A = Z, as unidades s
ao 1 e se A = Z[i] os u
nicos
elementos invertveis s
ao {1, i} como se mostrara no lema 4.31.
Por outro lado, dado que em Z[i] temos um algoritmo de divisao,
entao tambem temos um equivalente ao teorema de Bachet-Bezout como
se mostra a seguir.
Teorema 4.27 (Bachet-Bezout). Sejam e dois elementos em Z[i]
primos entre si, isto e, cujos u
nicos divisores comuns s
ao unidades. Ent
ao existem x, y Z[i] tais que
x + y = 1.
o: Consideremos o conjunto C = {x + y|x, y Z[i]}
Demonstra
ca
e sejam = x0 + y0 C nao nulo e com norma mnima (observe
que o conjunto das normas e um subconjunto de N e portanto esta bem
ordenado). Vejamos que divide e . Caso contrario, suponhamos
que nao divide : assim existem , Z[i] tais que = + , com
0 < N () < N (). Portanto
= = (1 x0 ) (y0 )
tambem pertence a C, o que e contradiz a minimalidade de N (), e logo
divide . Simetricamente temos que divide . Como e s
ao
primos entre si, temos que N () = 1, e assim, multiplicando por caso
necessario, podemos supor que = 1, como queriamos mostrar.
Exemplo 4.28. Calcule mdc(17 + 51i, 210 + 214i)

o: Aplicando o algoritmo extendido da divisao para inteiros de


Solu
ca
Gau temos
210 + 214i = (2 + 5i) (17 + 51i) + (11 + 27i)
17 + 51i = (2 + 0i) (11 + 27i) + (5 3i)
11 + 27i = (4 3i) (5 3i).

Poranto mdc(17 + 51i, 210 + 214i) = 5 + 3i.

4.12. CONTANDO TRIANGULOS


PITAGORICOS
COM HIPOTENUSA FIXA

Problemas Propostos
4.45. Calcule um resto da divis
ao de 38 + 65i por 7 5i.
4.46. Determine mdc(232 + 156i, 371 + 223i).
4.47. Use o algarismo extendido da divis
ao para determinar as soluc
oes
x, y Z[i] de (13 + 10i)x + (7 + 20i)y = 1.
Podemos tambem definir congruencias para inteiros de Gau:

(mod ) | .

As mesmas demonstracoes do caso de Z mostram que congruencias m


odulo definem uma relacao de equivalencia em Z[i] compatvel com a
soma, a subtracao e o produto. Podemos portanto formar o anel quociente Z[i]/(), cujos elementos s
ao as classes de congruencia m
odulo
.
Exemplo 4.29. Mostre que (1 + i)2009 + 1 e divisvel por 2 + i em Z[i].
o: Temos que 2 + i 0 (mod 2 + i) i 2 (mod 2 +
Solu
ca
i) 1 + i 1 (mod 2 + i). Logo (1 + i)2009 (1)2009 (mod 2 +
i) (1 + i)2009 + 1 0 (mod 2 + i).
O proximo passo e generalizar o conceito de primo:
Defini
c
ao 4.30. Dizemos que A \ {0} e irredutvel se ele n
ao pode
ser escrito como produto de dois elementos em A \ A . Dois irredutveis
1 e 2 s
ao ditos associados se eles diferem por multiplicac
ao por uma

unidade: 1 = u2 com u A .
Por exemplo, em A = Z, os n
umeros da forma p, onde p e um
n
umero primo s
ao primos associados. Intuitivamente, elementos associados devem ser vistos nao como primos distintos mas como um
unico
primo para efeitos de fatoracao.
Primos em Z nao necessariamente s
ao irredutveis em Z[i]. Por exemplo, temos que 5 = (2 + i)(2 i). Por outro lado, 2 + i e 2 i possuem
norma N (2 i) = 5 prima e logo s
ao irredutveis pelo seguinte
Lema 4.31.
1. Z[i] = {1, i}. Em particular u Z[i]
N (u) = 1.

2. Se Z[i] e tal que N () e um n


umero primo, ent
ao e irredutvel.
3. Se p Z e um primo p 3 (mod 4), ent
ao p e irredutvel em Z[i].


LINEARES
174 CAPITULO 4. EQUAC
OES
DIOFANTINAS NAO
facil verificar que 1, i s
o: E
Demonstra
ca
ao unidades. Por outro

lado, se u Z[i] , entao existe v Z[i] tal que uv = 1, logo N (u)N (v) =
1. Como N (u), N (v) s
ao inteiros positivos, temos N (u) = N (v) = 1.
Escrevendo u = a + bi com a, b Z, temos que N (u) = 1 a2 + b2 =
1 (a, b) = (1, 0) ou (a, b) = (0, 1), ou seja, u {1, i}.
Agora suponha que N () seja primo. Se = com , Z[i]
entao N () = N ()N (). Como N () e primo, ou N () = 1 ou N () =
1, ou seja, ou ou e uma unidade e portanto e irredutvel.
Finalmente, seja p 3 (mod 4). Se p pode ser fatorado como p =
com , Z[i] \ Z[i] , temos p2 = N (p) = N ()N (). Como e nao
s
ao unidades, N () 6= 1 e N () 6= 1, logo N () = N () = p. Porem,
escrevendo = a + bi com a, b Z, temos que a2 + b2 = p 3 (mod 4),
o que e impossvel, visto que um quadrado perfeito e congruente a 0 ou
a 1 m
odulo 4, logo a2 + b2 e congruente a 0, 1 ou 2 m
odulo 4, mas nunca
a 3 m
odulo 4.
Note que, como no caso de Z, o teorema de Bachet-Bezout implica
Lema 4.32. Seja Z[i] um elemento irredutvel. Ent
ao
| = |

ou

para , Z[i].
Como corolario, obtemos a fatoracao u
nica:
Teorema 4.33 (Fatoracao u
nica). Qualquer elemento 6= 0 de Z[i]
admite uma fatorac
ao
= 1 2 . . . n
em elementos irredutveis i . Tal fatorac
ao e u
nica a menos da ordem
dos fatores e de multiplicac
ao por unidades (isto e, a menos de associados).
o: A prova da unicidade da fatoracao e identica `a dos
Demonstra
ca
inteiros, utilizando o lema anterior. A prova da existencia da fatoracao
e tambem similar, mas agora utilizamos inducao em N (): se N () = 2
(base) entao e irredutvel (ver lema) e se e irredutvel, nao ha nada a
fazer; caso contrario, existe uma fatoracao = onde nem nem s
ao
unidades, isto e, N () 6= 1 e N () 6= 1. Como N () = N ()N (), temos
que e possuem norma estritamente menor do que N (). Por hip
otese
de inducao, e podem ser fatorados em irredutveis, e combinando as
duas fatoracoes temos uma fatoracao de .

4.12. CONTANDO TRIANGULOS


PITAGORICOS
COM HIPOTENUSA FIXA
Exemplo 4.34. Escreva 50 como produto de irredutveis em Z[i].
o: Como 50 = 2 52 e ja sabemos fatorar 5 = (2 + i)(2 i)
Solu
ca
em irredutveis, basta agora fatorar 2. Temos que 2 = (1 + i)(1 i) =
i(1 i)2 e 1 i e irredutvel pois sua norma N (1 i) = 2 e prima. Logo
50 = i(1 i)2 (2 + i)2 (2 i)2 e a fatoracao em irredutveis de 50.
Exemplo 4.35. Determine todas as soluc
oes inteiras da equac
ao
x2 + y 2 = z 3 ,

com

mdc(x, y) = 1.

o: Fatorando em inteiros de Gau temos que (x + iy)(x iy) =


Solu
ca
3
z . Alem disso, temos
mdc(x+iy, xiy) = mdc(2x, xiy)| mdc(2x, 2x2iy) = 2 mdc(x, y) = 2.
O mdc destes dois n
umeros nao pode ser 2, porque nesse caso x e y
seriam divisveis por 2. Assim temos dois possveis casos
Se mdc(x+iy, xiy) = 1, entao cada um e o cubo de um inteiro de
Gau, isto, e x+iy = (a+ib)3 , portanto x = a2 3ab2 , y = 3abb3
e z = a2 + b2 com mdc(a, b) = 1 e com a + b mpar (de fato, se
a + b fosse par, entao a + ib e x + iy seriam m
ultiplos de 1 + i, e
teramos 1 + i | mdc(x + iy, x iy), contradicao).
Se mdc(x + iy, x iy) = 1 + i teramos x e y da mesma paridade,
logo x e y seriam mpares, donde x2 + y 2 2 (mod 4), e logo
x2 + y 2 nao pode ser o cubo de um inteiro.

Problemas Propostos
4.48. Determine todas as soluc
oes inteiras da equac
ao x2 + y 2 = 2z 3 .
4.49. Mostre que se p um primo da forma 4k + 1 ent
ao x2 + y 2 = pz 4
possui infinitas soluc
oes com mdc(x, y) = 1.
4.50. Mostre que a equac
ao 25x2 + 14xy + 2y 2 = z 5 possui infinitas
soluc
oes com mdc(x, y) = 1.
Agora sim voltemos a nosso problema original de calcular S2 (c) e
T2 (c).


LINEARES
176 CAPITULO 4. EQUAC
OES
DIOFANTINAS NAO
Teorema 4.36. Sejam c um inteiro positivo e c = p1 1 pk k sua fatorac
ao em fatores primos. Ent
ao

se c e par
0
S2 (c) = 0
se existe j com 1 j k tal que pj 3 (mod 4)

k1
2
caso contr
ario.

o: Se c e par, nao pode ser escrito como soma de quaDemonstra


ca
drados de paridades distintzas. No caso que c possui um divisor primo
da forma 4t + 3 ja foi provado na proposicao 4.8 que c nao pode-se escrever como soma de dois quadrados primos entre si. Assim vamos supor
que todos os divisores primos de c deixam resto 1 quando divididos por
4. Neste caso cada pk pode-se fatorar no anel de inteiros de Gau com
pj = pj pj , onde pj = aj + ibj e pj = aj ibj s
ao primos no anel dos
inteiros de Gau. Assim c se fatora nos inteiros de Gau como
c = p1 1 p1 1 p2 2 p2 2 p1 k p1 k .
Como queremos saber de quantas formas podemos escrever c como m2 +
n2 , isto e c = (m + in)(m in), com mdc(m, n) = 1, entao queremos
contar de quantas formas podemos distribuir os fatores de c em dois
fatores com a propridedade de que um sejam conjugado do outro, assim,
automaticamente no momento que escolhemos um primo para estar no
primeiro fator, seu conjugado tem que estar no segundo fator. Observemos tambem que um primo pj e seu conjugado pj nao podem estar
simultaneamente no mesmo fator, porque isso implicaria que pj dividiria simultaneamente m + in e m in, e logo m e n nao seriam primos
entre si. Assim, o n
umero de formas de escolher os fatores de m + in e
2k . Como a ordem dos fatores m + in e m in nao interessa, entao o
n
umero de formas distintas de escrever c como soma de dois quadrados
e 2k1 .
Agora estamos em condicoes de estudar o casos geral, isto e, contar
todas as triplas pitagoricas com hipotenusa fixa.
Teorema 4.37. Sejam c um n
umero inteiro positivo e
c = 2n p1 1 pk k q11 qll
sua fatorac
ao em fatores primos, onde os fatores pi s
ao congruentes a 1
m
odulo 4 e os fatores qj s
ao congruentes a 3 m
odulo 4. Ent
ao
1
1
T2 (c) = (21 + 1) (2k + 1) .
2
2

4.12. CONTANDO TRIANGULOS


PITAGORICOS
COM HIPOTENUSA FIXA
o: Observemos que a funcao S2 nao e multiplicativa,
Demonstra
ca
mas e facil comprovar que a funcao S definida por
(
1
se n = 1

S(n)
=
2S2 (n) se n > 1
e multiplicativa. Assim
T2 (c) =

X
d|c
d6=c

S2

c
d

1 X  c  1
S
.
2
d
2
d|c

Usando o fato que S e multiplicativa, temos que para todo primo p se


tem que
(
t
X
se p = 2 ou p 3 (mod 4)
j) = 1
S(p
.
2t + 1 se p 1 (mod 4)
j=0
Destes dois fatos obtemos o que queramos mostrar.
No exemplo 4.23 exibimos 7 tri
angulos com hipotenusa 325. De fato,
como 325 = 52 13, aplicando a formula obtida no teorema anterior
temos que T (325) = 21 (2 2 + 1)(2 1 + 1) 12 = 7, que esta em
concord
ancia com o exemplo.
Vamos agora mostrar uma formula para U2 (c):
Teorema 4.38. Sejam c um n
umero inteiro positivo e
c = 2n p1 1 pk k q11 qll
sua fatorac
ao em fatores primos, onde os fatores pi s
ao congruentes a 1
m
odulo 4 e os fatores qj s
ao congruentes a 3 m
odulo 4. Se algum j e
mpar ent
ao U2 (c) = 0. Caso contr
ario, temos duas possibilidades:
Caso algum i seja mpar,
1
U2 (c) = (1 + 1) (k + 1),
2
e caso todos os i sejam pares,
1
U2 (c) = ((1 + 1) (k + 1) + 1).
2
o: Cada pk pode-se fatorar no anel de inteiros de Gau
Demonstra
ca
com pj = pj pj , onde pj = aj + ibj e pj = aj ibj s
ao primos no anel dos
inteiros de Gau. Assim c se fatora nos inteiros de Gau como
c = in (1 i)n p1 1 p1 1 p2 2 p2 2 p1 k p1 k q11 qll .


LINEARES
178 CAPITULO 4. EQUAC
OES
DIOFANTINAS NAO
Queremos saber de quantas formas podemos escrever c como u2 + v 2 ,
isto e c = (u + iv)(u iv), com 0 u v, entao queremos contar
de quantas formas podemos distribuir os fatores de c em dois fatores
com a propridedade de que um sejam conjugado do outro. Para cada
i k, temos as i + 1 escolhas 0, 1, ..., 1 para o expoente 1 de p1 no
primeiro fator (sendo o expoente de p1 no primeiro fator igual a 1 1 ),
o que nos da (1 + 1) (k + 1) escolhas. Mas se permutarmos esses
dois fatores conjugados de c obtemos a mesma solucao. Assim, devemos
dividir o n
umero de escolhas por 2, exceto no caso em que algum fator
seja conjugado dele mesmo (m
odulo multiplicacao por invertvel), mas
isso s
o e possvel quando todos os i s
ao pares, e nesse caso ha um u
nico
fator com essa propriedade, o que nos da a formula do enunciado.

Problemas Propostos
4.51. Encontre todos os tri
angulos ret
angulos com hipotenusa 330.
4.52. Quantos triplas pitag
oricas tem hipotenusa igual a 5525?
4.53. Encontre o menor valor para c de tal forma que existam exatamente 28 tri
angulos com hipotenusa igual a c.

4.13

Descenso Infinito de Fermat

Ate agora foram estudadas essencialmente equacoes que principalmente possuem termos quadraticos. Existem metodos semelhantes ao
metodo geometrico, por exemplo em curvas do tipo y 2 = x3 + ax + b,
as chamadas curvas elpticas, mas, em geral, nao existe um metodo generico para determinar se uma equacao diofantina possui ou nao possui
solucao. De fato, em 1970 o matem
atico russo Yuri Matiyasevich, completando trabalhos de Martin Davis, Julia Robinson e Hilary Putnam,
mostrou que nao pode existir um algoritmo para determinar se uma
dada equacao diofantina possui ou nao possui solucao, dando resposta
negativa ao decimo problema de Hilbert.
Apesar disso, em alguns casos, e possvel mostrar que algumas equacoes diofantinas
f (x1 , . . . , xn ) = 0,
nao possuem solucao. O metodo do descenso infinito (quando aplicavel)
permite mostrar que esta equacao nao possui solucoes inteiras positivas
ou, sob certas condicoes, ate mesmo encontrar todas as suas solucoes
inteiras. Se o conjunto de solucoes de f
A = {(x1 , . . . , xn ) Zn | f (x1 , . . . , xn ) = 0}

4.13. DESCENSO INFINITO DE FERMAT

179

e diferente de vazio, entao gostaramos de considerar a solucao mnima


em certo sentido. Em outras palavras, queremos construir uma funcao
: A N e considerar a solucao (x1 , . . . , xn ) A com (x1 , . . . , xn )
mnimo. O descenso consiste em obter, a partir desta solucao mnima,
uma ainda menor, o que nos conduz claramente a uma contradicao,
provando que A de fato e vazio.
Para ilustrar este metodo consideremos o seguinte
Exemplo 4.39. Sejam a e b inteiros positivos tais que b < 2a + 1.
Mostrar que a equac
ao bx2 2axy y 2 = 0 n
ao possui soluc
oes inteiras
positivas.
o: Suponhamos que a equacao possui solucao nao nula (m, n)
Solu
ca
com m, n N, isto e, bm2 2amn n2 = 0 e escolhemos esta solucao
de tal forma que n seja mnimo. Como n2 + 2amn = bm2 , somando aos
dois lados a2 m2 obtemos que
(n + am)2 = (b + a2 )m2 < (a2 + 2a + 1)m2 = (a + 1)2 m2 ,
e assim n+am < am+m, isto, e n < m. Observemos que m(bm2an) =
n2 , isto e, m e um divisor de n2 , portanto n2 = mq onde 0 < q < n < m
e q + 2an = bm. Por outro lado, multiplicando a equacao original por b
obtemos
0 = bn2 +2an(bm)(bm)2 = bn2 +2an(q+2an)(q+2an)2 = bn2 2anqq 2 ,
logo o par (n, q) e tambem solucao da equacao, o que contradiz a minimalidade da solucao anterior.

Como consequencia imediata do problema anterior, podemos mostrar

que se k e um inteiro positivo que nao e um quadrado perfeito,


ent
ao k

e irracional. De fato, suponhamos por contradicao que k e racional e


seja a sua parte inteira. Logo existem inteiros nao nulos m, n tais que

n
k =a+ m
. Elevando ao quadrado e multiplicando por m2 obtemos
(k a2 )m2 2amn n2 = 0,
isto e, a equacao (k a2 )x2 2axy y 2 = 0 possui uma solucao inteira
positiva, mas k < (a + 1)2 , portanto k a2 < 2a + 1, mas pelo problema

anterior a equacao nao possui solucoes inteiras positivas, portanto k


nao pode ser racional.
O seguinte e um exemplo dado por Fermat para aplicacao de seu
metodo, que em particular tem como consequencia que a equacao de
Fermat nao possui solucoes nao triviais para expoentes m
ultiplos de 4.


LINEARES
180 CAPITULO 4. EQUAC
OES
DIOFANTINAS NAO
Exemplo 4.40 (Fermat). Demonstrar que a equac
ao x4 + y 4 = z 2 n
ao
possui soluc
oes inteiras positivas, isto e, n
ao existem tripas pitag
oricas
em que os dois catetos sejam quadrados perfeitos.
o: Suponhamos que x4 + y 4 = z 2 possui uma solucao inteira
Solu
ca
com x, y, z > 0. Logo existe uma tal solucao (a, b, c) na qual c e mnimo. Em particular, temos que a e b s
ao primos entre si, pois se
d = mdc(a, b) > 1 poderamos substituir (a, b, c) por ( ad , db , dc2 ) e obter uma solucao com c menor. De (a2 )2 + (b2 )2 = c2 temos portanto
que (a2 , b2 , c) e uma tripla pitagorica primitiva e assim existem inteiros
positivos m e n primos relativos tais que
a2 = m2 n2 ,

b2 = 2mn

c = m2 + n2 .

Da primeira equacao temos que (a, n, m) e uma tripla pitagorica primitiva e portanto m e mpar. Assim, de b2 = 2mn conclumos que b, e
portanto n, e par. Observando ainda que b2 = (2n)m e um quadrado
perfeito e mdc(2n, m) = 1, conclumos que tanto 2n como m s
ao quadrados perfeitos, donde podemos encontrar inteiros positivos s e t tais
que
2n = 4s2
e
m = t2 .
Por outro lado, dado que a2 + n2 = m2 , entao existir
ao inteiros positivos
i e j, primos entre si, tais que
a = i2 j 2 ,

n = 2ij

m = i2 + j 2 .

Portanto s2 = n2 = ij, logo i e j serao quadrados perfeitos, digamos


i = u2 e j = v 2 .
Logo temos que m = i2 + j 2 , i = u2 , j = v 2 e m = t2 , assim
t2 = u 4 + v 4 ,
isto e, (u, v, t) e outra solucao da equacao original. Porem
t t2 = m m2 < m2 + n2 = c
e t 6= 0 porque m e diferente de 0. Isto contradiz a minimalidade de c,
o que conclui a demonstracao.
Observemos alem disso que, uma vez que esta equacao nao possui solucoes inteiras positivas, entao a equacao x4 +y 4 = z 4 e, mais geralmente
x4n + y 4n = z 4n , nao possuem solucoes inteiras positivas.
Exemplo 4.41. Mostre que a equac
ao x4 2y 2 = 1 somente possui
soluc
oes triviais.

181

4.13. DESCENSO INFINITO DE FERMAT

o: Suponhamos que (a, b) e uma solucao da equacao, assim


Solu
ca
a4 1
2
b = 2 . Elevando ao quadrado obtemos
4

b =

a4 1
2

2

a8 + 2a4 + 1 4a4
=
=
4

a4 + 1
2

2

a4 ,

 4 2
logo a4 + b4 = a 2+1 e solucao da equacao x4 + y 4 = z 2 , que como foi
mostrado anteriormente, somente possui solucoes com xyz = 0, assim
a = 1 e b = 0 e a u
nica solucao.
No seguinte exemplo e usado o metodo do descenso de Fermat para
achar todas as solucoes a partir de uma solucao mnima.
Exemplo 4.42. Determine todos os pares de inteiros positivos (m, n)
tais que m|n2 + 1 e n|m2 + 1.
o: Observemos que qualquer solucao satisfaz mdc(m, n) =
Solu
ca
mdc(n2 + 1, n) = 1, e assim a u
nica solucao com m = n e a solucao
(1, 1). Suponhamos que temos outra solucao com m 6= n e pela simetria
das condicoes do problema, podemos supor que m > n. Como m e n
2
2 +1
dividem m2 + n2 + 1 e nao tem fator comum, entao m +n
= k N.
mn
2
2
Assim temos a relacao m + n + 1 kmn = 0, isto e, m e raiz do polin
omio quadratico X 2 knX + (n2 + 1) = 0. A outra solucao desta equacao
quadratica pode ser calculada de duas formas distintas, j
a que kn e a
soma das razes e n2 + 1 e o produto das razes. Assim esta segunda raiz
m por um lado e igual a kn m, e portanto inteira, e por outro lado
2
tambem e igual a n m+1 o que implica que e positiva. Observe alem disso
que
n2 + 1
2
n2 + 1

=n1+
n.
m =
m
n+1
n+1
Por tanto se o par (m, n) e solucao do problema com m > n, entao (n, m )
2 +n2 +1
e solucao de m nm
= k com n m , onde esta desigualdade e estrita

se n > 1. Assim, neste caso poderamos de novo aplicar este processo


para de novo encontrar uma solucao menor. Este processo s
o nao pode
ser aplicado novamente quando n = 1 = m, e neste caso k = 3. Para
gerar todas as solucoes basta fazer o processo inverso. Assim, partindo
do par (1, 1) podemos construir todas as solucoes usando os termos da
recorrencia x1 = 1, x2 = 1 e xj+1 = 3xj xj1 , onde os pares
(1, 1), (2, 1), (5, 2), (13, 5), (34, 13), . . . , (xj+1 , xj ), . . .
s
ao todas as solucoes do problema.


LINEARES
182 CAPITULO 4. EQUAC
OES
DIOFANTINAS NAO
O metodo do descenso tambem pode ser usado para encontrar as
solucoes da equacao de Markov.
Exemplo 4.43. Mostrar que a equac
ao diofantina
x2 + y 2 + z 2 = 3xyz.
possui infinitas soluc
oes inteiras positivas, e descreve-las.
Esta equac
ao e conhecida como equac
ao de Markov.
o: Por verificacao direta temos que (1, 1, 1) e (1, 1, 2) s
Solu
ca
ao solucoes da equacao. Alem disso, como a equacao e simetrica, podemos
considerar, sem perda de generalidade, somente as solucoes com as coordenadas x y z ordenadas de forma nao decrescente. Se z = y,
teremos x2 = 3xy 2 2y 2 = (3x 2)y 2 y 2 x2 , valendo as igualdades
se e s
o se z = y = x = 1.
Assim suponhamos que (x, y, z) e uma solucao com x y < z e
portanto com z > 1. O polin
omio quadratico
T 2 3xyT + (x2 + y 2 ) = 0
possui duas solucoes, e uma dela e z. Assim a outra e
z = 3xy z =

x2 + y 2
Z \ {0}.
z

Vejamos que z y < z, e assim (z , x, y) e tambem solucao (menor)


da equacao de Markov. Para isto, suponhamos por contradicao que
x2 +y 2
= z > y, isto e, yz < x2 + y 2 2y 2 , e em particular z < 2y.
z
Segue que
5y 2 > y 2 + z 2 = 3xyz x2 = x(3yz x) xy(3z 1),
e portanto 5y > x(3z 1). Observemos que se x 2, entao 5y
2
z,
2(3z 1) 5z, o que contradiz a hip
otese y > z. Logo x = 1 e 1+y
y
assim y1 + y z > y. Portanto devemos ter temos y1 + y = z, e neste
caso y = 1 e z = 2, o que nos da z = 3xy z = 1 y, contradicao.
Do fato anterior, temos que dada uma solucao da equacao de Markov (x, y, z) com z 2 ,e sempre possvel encontrar uma solucao menor (z , x, y) e este processo somente para quando chegamos `a solucao
(1, 1, 1), isto e, estamos gerando uma
arvore de solucoes da seguinte
forma:

183

4.13. DESCENSO INFINITO DE FERMAT

(1, 1, 1)

(1, 1, 2)

(1, 2, 5)

(1, 5, 13)

(1, 13, 34)


(1, 34, 89) (13, 34, 1385)
.
.
..
..

(2, 5, 29)

(5, 13, 194)

(2, 29, 169)


..
.

(5, 29, 433)


..
.

(5, 194, 2897) (13, 194, 7561)


.
.
.
.
.
.

onde de cada no (x, y, z), com x y z, saem duas novas solucoes


(x, z, 3xz y) e (y, z, 3yz x).

Problemas Propostos
4.54. Seja p um n
umero primo e n um inteiro maior do que 1. Usar o

umero irraciometodo do descenso infinito para mostrar que n p e um n


nal.
4.55. Seja p um n
umero primo. Mostrar que n
ao existem inteiros positivos a, b e c tais que a3 + pb3 + p2 c3 = 0.
4.56. Pode um tri
angulo ret
angulo com lados inteiros ter
area que seja
o quadrado de um inteiro?
4.57. Mostre que a equac
ao x2 + y 2 + z 2 = x2 y 2 n
ao possui soluc
oes
inteiras positivas.
4.58. Mostre que n
ao existem
ao nulos x, y, z, w tais que cum( inteiros n
x2 + y 2 = z 2
prem o sistema de equac
oes
.
x2 y 2 = w 2
4.59. Mostrar usando o metodo do descenso infinito que a equac
ao x 4 +
4
2
y = 2z n
ao tem soluc
oes n
ao triviais.


LINEARES
184 CAPITULO 4. EQUAC
OES
DIOFANTINAS NAO
4.60. Sejam a e b inteiros postitivos tais que ab divide a2 + b2 + 2.
2
2 +2
= 4.
Mostrar que a +b
ab
4.61. Sejam a e b inteiros positivos tais que ab+1 divide a2 +b2 . Mostrar
a 2 + b2
que o n
umero
e um quadrado perfeito.
ab + 1
4.62. Seja k um n
umero inteiro distinto de 1 e 3. Mostrar que a equac
ao
2
2
2
x + y + z = kxyz n
ao possui soluc
oes inteiras positivas.
4.63. Determine todas as soluc
oes inteiras de x4 2y 2 = 1.

Bibliografia
[1] F. E. Brochero Martinez, C. G. Moreira, N. C. Saldanha, E. Tengan
- Teoria dos N
umeros: um passeio com primos e outros n
umeros
familiares pelo mundo inteiro - Projeto Euclides, IMPA, 2010.
[2] A. Caminha. Equacoes diofantinas, Revista Eureka! No. 7, pp. 3948.
[3] T. W. Cusick e M. E. Flahive, The Markoff and Lagrange spectra,
Math. Surveys and Monographs, no. 30, A.M.S. (1989).
[4] S. C. Coutinho, N
umeros inteiros e criptografia RSA, Colecao Computacao e Matem
atica, SBM e IMPA (2000).
[5] H. Cramer, On the order of magnitude of the difference between
consecutive prime numbers, Acta Arithmetica 2: 2346 (1936).
[6] Lorenzo J. Daz, Danielle de Rezende Jorge, Uma introduc
ao aos
o
Sistemas Din
amicos via Frac
oes Contnuas, 26 Coloquio Brasileiro
de Matem
atica, IMPA (2007).
[7] A. Hefez. Elementos de Aritmetica, 2a. edicao. Textos Universitarios, SBM (2005).
[8] C. G. Moreira, O teorema de Ramsey, Revista Eureka! 6, 2329.
[9] C. G. Moreira, Geometric properties of the Markov and Lagrange
spectra. Preprint-IMPA-2009.
[10] D. H. J. Polymath, Deterministic methods to find primes, preprint,
http://polymathprojects.files.wordpress.com/2010/07/
polymath.pdf; veja tambem http://polymathprojects.org/
2009/08/09/research-thread-ii-deterministic-way-tofind-primes/ e http://michaelnielsen.org/polymath1/
index.php? title=Finding_primes
185

186

BIBLIOGRAFIA

[11] A. Politi, J. C. F. Matthews, J. L. OBrien, Shors Quantum Factoring Algorithm on a Photonic Chip, Science 4 September 2009:
Vol. 325. no. 5945, p. 1221.
[12] P. Ribenboim, Selling primes, Math. Mag. 68 (1995), 175182. Traduzido como Vendendo primos, Rev. Mat. Univ. 22/23 (1997), 113.
[13] J.P.O. Santos. Introducao `
a Teoria dos N
umeros, 3a. edicao. Colecao Matem
atica Universit
aria, IMPA (2010).
[14] J.P. Serre, On a theorem of Jordan, Bull. Amer. Math. Soc. (N.S.)
40 (2003), no. 4, 429440.
[15] A. Shen e N. K. Vereshchagin, Basic Set Theory, AMS, 2002.
[16] P. W. Shor, Polynomial-Time Algorithms for Prime Factorization and Discrete Logarithms on a Quantum Computer, SIAM
J. Comput. 26 (5), 1484-1509 (1997). Tambem em arXiv:quantph/9508027v2.

S-ar putea să vă placă și